Text
                    МАТЕМАТИКА
ВСЕРОССИЙСКИЕ
ОЛИМПИАДЫ
ПРОСВЕЩЕНИЕ
ИЗДАТЕЛЬСТВО


МАТЕМАТИКА ВСЕРОССИЙСКИЕ ОЛИМПИАДЫ Выпуск 1 Москва «Просвещение» 2008
УДК 372.8:51 ББК 74.262.21 М34 Серия «Пять колец» основана в 2007 г. Авторы: Н. X. Агаханов, И. И. Богданов, П. А. Кожевников, О. К. Подлипский, Д. А. Терешин Под общей редакцией С. И. Демидовой и И. И. Колисниченко Математика. Всероссийские олимпиады. Вып. 1 / М34 [Н. X. Агаханов, И. И. Богданов, П. А. Кожевников и др.; под общ. ред. С. И. Демидовой, И. И. Колисниченко]. — М. : Просвещение, 2008. — 192 с. ил. — (Пять колец). — ISBN 978-5-09-017182-3. В книге описаны структура Всероссийской олимпиады школьников по математике, особенности проведения различных этапов, в нее включены практические советы по организации олимпиад. В книге приведены комплекты заданий Всероссийской математической олимпиады школьников различных этапов в 2005/2006 и 2006/2007 гг. К задачам даются подробные решения. УДК 372.8:51 ББК 74.262.21 Учебное издание Серия «Пять колец» Агаханов Назар Хангельдыевич, Богданов Илья Игоревич, Кожевников Павел Александрович и др. МАТЕМАТИКА Всероссийские олимпиады Выпуск 1 Зав. редакцией Т. А. Бурмистрова. Редактор Т. Ю. Акимова. Младшие редакторы Е. А. Андреенкова, С. В.Дубова. Дизайн обложки С.Ю.Бири- чев. Художественный редактор О. П. Богомолова. Технический редактор и верстальщик Е.В.Саватеева. Корректоры Ю. Б. Григорьева, Л. С. Румянцева Налоговая льгота — Общероссийский классификатор продукции ОК 005- 93—953000. Изд. лиц. Серия ИД № 05824 от 12.09.01. Подписано в печать 24.09.07. Формат 6OX9OVie- Бумага офсетная. Гарнитура Школьная. Печать офсетная. Уч.-изд. л. 11,83. Тираж 10000 экз. Заказ № 19944. Открытое акционерное общество «Издательство «Просвещение». 127521, Москва, 3-й проезд Марьиной рощи, 41. Отпечатано в ОАО «Саратовский полиграфкомбинат». ч 410004, г. Саратов, ул. Чернышевского, 59. www.sarpk.ru у ISBN 978-5-09-017182-3 © Издательство «Просвещение», 2008 © Художественное оформление. Издательство «Просвещение», 2008 Все права защищены
ГЛАВЛЕНИЕ Введение 5 ГЛАВА 1. О математических олимпиадах Что такое математическая олимпиада 8 История математических олимпиад 11 ГЛАВА 2. Структура олимпиады Школьные олимпиады 14 Районные (городские) олимпиады 15 Региональные (областные, республиканские) олимпиады 15 Федеральные окружные олимпиады 16 Заключительный этап олимпиады 17 Методическая комиссия и Жюри олимпиады.... — глава 3. Подготовка и проведение математических олимпиад Тематика математических олимпиад 22 Структура варианта 25 Организация проведения туров олимпиады 26 Организация проверки работ 27 Определение победителей и призеров 33 Особенности подготовки и проведения школьных олимпиад 33 Особенности подготовки и проведения районных олимпиад 34 Особенности подготовки и проведения региональных олимпиад 35 Особенности подготовки и проведения IV—V этапов олимпиады — 3
ГЛАВА 4. Олимпиада 2005/2006 учебного года II этап (районный) 37 III этап (региональный) 51 IV этап (федеральный окружной) 71 V этап (заключительный) 91 ГЛАВА 5. Олимпиада 2006/2007 учебного года II этап (районный) 110 III этап (региональный) 124 IV этап (федеральный окружной) 144 V этап (заключительный) 166 Список литературы 191
В последние годы наблюдается динамичное развитие олимпиадного движения как в России, так и во всем мире. Всероссийские олимпиады проводятся уже по двум десяткам предметов, а число стран, участвующих в Международной математической олимпиаде, приближается к сотне. Предметные олимпиады школьников доказали свою эффективность в решении задач поиска и отбора интеллектуально одаренных учащихся. Подтверждением этому является и законодательно закрепленное право победителей Всероссийских олимпиад школьников на внеконкурсное поступление в профильные вузы. Анализ выступлений школьников на математических олимпиадах высокого уровня показывает, что наибольшего успеха добиваются учащиеся из тех регионов России, где работа с одаренными детьми педагогов-энтузиастов активно поддерживается чиновниками системы образования. Плоды приносит гармоничное сочетание грамотной организации олимпиад, при которой снимаются искусственные организационные или финансовые ограничения, препятствующие участию в олимпиадах всех одаренных школьников, и привлечение к работе с детьми наиболее талантливых педагогов. Это могут быть также вузовские преподаватели, студенты и аспиранты, становившиеся в прошлом победителями и призерами олимпиад высокого уровня. В книге описаны структура Всероссийской олимпиады школьников по математике, особенности проведения различных этапов, в нее включены практические советы по организации олимпиад. В книге приведены комплекты заданий Всероссийской математической олимпиады школьников различных этапов (от начальных до заключительных).
ГЛАВА 1 математических олимпиадах Математика как самостоятельный предмет начинает изучаться в школе уже с первого класса. Во-первых, математика является универсальным языком всех наук, и этим обусловлено ее особое положение в школьной программе. Во-вторых, способности в изучении математики определяют и способности учащихся в точных науках. Об этом свидетельствует, в частности, включение экзаменов по математике в конкурсные испытания всех вузов естественно- математического профиля. Математические способности — это не просто усвоенный набор знаний, умение запоминать и воспроизводить конкретные факты, а способности к логическому осмыслению знания, к умению абстрагироваться от конкретного, к обобщению частного. Наиболее распространенной и отработанной формой отбора математически одаренных школьников являются математические олимпиады. В олимпиадах естественно-математического цикла, в первую очередь по физике, математике и информатике, главную роль играет не столько сумма конкретных знаний молодого человека, сколько его способность за ограниченное время олимпиады построить и исследовать достаточно сложную модель или логическую конструкцию, с которой он прежде никогда не сталкивался. В олимпиадах по этим предметам невозможны тестовые задания, проверяющие знания школьника, его начитанность. Напротив, обязательным требованием, предъявляемым к заданиям этих олимпиад, является их новизна для участников. Поэтому успешное выступление в олимпиаде предполагает: психологическую готовность школьника к выполнению нестандартных заданий, отказ от стереотипных подходов (тем более что задания следующего этапа 6 О математических олимпиадах
олимпиады заметно превосходят по сложности задания предыдущего этапа); математическую одаренность, т. е. способность к построению нестандартных логических конструкций; высокие «спортивные» качества участника — умение собраться, сконцентрироваться на выполнении нескольких заданий за непродолжительное время олимпиады; математическую грамотность участника — умение строго (с использованием математических понятий и терминов) записать в работе решения задач; успешное и полное овладение школьником изучаемых разделов математики. Стремление к достижению олимпиадных успехов является стимулом для учащихся, поддерживает серьезный интерес к учебе и дополнительным занятиям. Важную роль в проявлении интереса к занятиям математикой играет эстетическая красота олимпиадных задач. Наконец, успехи учеников на математических олимпиадах наряду с успешностью поступления в вузы (в том числе с результатами сдачи ЕГЭ) являются общественно признанными объективными критериями качества работы учителя. Поэтому факультативная работа со школьниками является инструментом профессиональной самореализации учителя; кроме того, она приносит учителю удовлетворение от творческого сотрудничества со своими учениками. Таким образом, олимпиадное движение является стимулом для ведения учителем внеклассной работы и для повышения им своей квалификации. Результаты на математических олимпиадах международного уровня говорят об общем уровне развития образования в стране и готовности этой страны создавать и воспроизводить новые технологии. Поэтому в государствах, стремящихся занять лидирующие экономические и политические позиции в мире, придается большое значение как развитию национальных математических соревнований школьников, являющихся инструментом поиска и отбора одаренных молодых людей, так и успехам своих команд на Международных математических олимпиадах. От решения этих вопросов зависит формирование будущей интеллектуальной элиты государства, укрепление экономического могущества страны. О математических олимпиадах 7
Советский Союз был в числе первых стран, начавших регулярно проводить математические соревнования школьников. В стране был накоплен большой опыт проведения олимпиад. Стабильность выступления команды России на Международных олимпиадах подтверждает сохранение в стране советских олимпиадных традиций. Более того, за последние десять лет значительно расширилась география регионов, в которых ведется серьезная работа со школьниками. В тех территориях, где есть энтузиасты, поддержанные местной администрацией, из года в год появляются сильные школьники. Если еще 12 лет назад, на Международной математической олимпиаде 1995 года, команда России состояла только из школьников Санкт-Петербурга и Москвы, то в последние годы в национальную команду включаются школьники из самых разных регионов России (Сибири, Урала, Центра и Юга страны). ф- Что такое математическая олимпиада В настоящее время математическая олимпиада — это соревнование между школьниками, где участник за фиксированное время должен решить предложенные задачи. Обычно решение оформляется в письменном виде (некоторые олимпиады в Санкт-Петербурге, согласно традиции, проводятся в форме устных олимпиад). Жюри за каждую задачу ставит определенное количество баллов в зависимости от степени продвижения участника в ее решении. Итоговый результат выступления определяется по сумме баллов, набранных участником. В прежние годы количество баллов по каждой задаче зависело от ее сложности и определялось либо априорно, либо уже во время самой олимпиады после первой проверки работ и обработки статистики успешности выполнения заданий. В настоящее время на всех этапах Всероссийской математической олимпиады школьников, как и на Международных математических олимпиадах, правильное решение каждой задачи оценивается в 7 баллов. Можно сказать, что математическая олимпиада — это творческое соревнование, являющееся гармоничным сочетанием спорта (точнее, интеллектуального состязания) и науки. Спортивная сторона олимпиады. Математические олимпиады используют некоторые человеческие особенно- 8 I О математических олимпиадах
сти, заложенные на генетическом уровне и наиболее ярко проявляющиеся в детском и подростковом возрасте. Это — желание соперничать. Почти во всех детских играх присутствует соревновательный элемент. Дети хотят соревноваться и соотносить свои возможности и достижения с достижениями других ребят. Для талантливых детей очень важны моральные стимулы, и они должны чувствовать интерес к себе, интерес к своим способностям. Свойственный подростковому возрасту дух состязательности является стимулом к систематическим углубленным занятиям математикой с целью максимальной реализации своих способностей во время олимпиады. Школьники, увлекающиеся олимпиадами, стремятся получить все более высокие результаты. Это требует большого напряжения и концентрации при подготовке к олимпиаде и на самой олимпиаде, что ведет к стремительному развитию и раскрытию способностей учащихся. Давно известно, что человек может подняться на следующий уровень достижений только при предельном напряжении сил. При этом, как и в спорте, в олимпиадах невозможно достижение серьезных результатов без регулярных самостоятельных или кружковых (факультативных) занятий. Соревновательный дух математической олимпиады не приводит к разобщению ее участников. Напротив, для участников олимпиада становится настоящим праздником, на котором они не только знакомятся с новыми интересными задачами, но и активно общаются друг с другом, участвуют в культурно-познавательной программе, подготовленной оргкомитетом. Многие контакты, установленные на олимпиадах еще в школьном возрасте, перерастают в дальнейшем в тесную дружбу и научное сотрудничество. Математические олимпиады сближают не только участников, но и всех людей, объединенных идеями как повышения качества математического образования в стране вообще, так и работы с одаренными школьниками в частности. На федеральных окружных и финальных турах Всероссийской олимпиады школьников по математике проходят встречи и семинары членов жюри и педагогов, работающих со школьниками, обмен опытом работы в регионах. Научная составляющая математических олимпиад. В математических олимпиадах многие задания начинаются со слов: «Докажите, что...» Уже сама формулировка заданий показывает, что школьнику предлагается самостоятельно вывести некоторое научное утверждение. Несомненно, О математических олимпиадах | 9
в силу ограниченности математического инструментария, которым владеет школьник, вывод таких утверждений еще нельзя назвать полноценной научной деятельностью. Но вырабатывающиеся в процессе решения олимпиадных задач навыки творческой деятельности в дальнейшем (после окончания вуза) облегчают переход к самостоятельным научным исследованиям. И хотя для успеха на олимпиаде необходимо иметь некоторые специфические «спортивные» качества — психологическую устойчивость, умение выкладываться в ограниченный промежуток времени (большая мощность мыслительной деятельности), бойцовские качества (умение собираться в нужный момент, «выкладываться» до конца и переносить поражения), остроту ума, — успехов в математике, как правило, добиваются именно бывшие « олимпийцы ». Почти все российские математики, получившие крупные международные премии (в том числе Филдсов- скую — самую престижную международную награду в области математики), были победителями Всероссийской (Всесоюзной) и Международной математической олимпиад. Новая, «прорывная» идея в математике порой может оказаться чисто олимпиадной, и решение математических проблем, над которыми многие годы бились математики всего мира, иногда удается найти с помощью нестандартных, «олимпиадных» подходов. Например, именно так Ю. В. Матиясевич (победитель VI Международной математической олимпиады) решил 10-ю проблему Гильберта, а А. А. Суслин (победитель IX Международной математической олимпиады) — проблему Серра. Научная важность олимпиад подчеркивается и тем, что подавляющее большинство выдающихся российских математиков занимались организацией олимпиад и подготовкой школьников к ним. Задания математических олимпиад являются, по сути, маленькими научными проблемами, поэтому при их составлении постоянно требуются новые идеи. И носителями этих идей часто становятся студенты, сами в недавнем прошлом успешно выступавшие на олимпиадах. От их участия зависит и качество работы жюри олимпиады. В математических олимпиадах не существует тестовых заданий, проверяемых по трафарету. Практически у любого задания возможны несколько вариантов решения, частичные продвижения в решении, поэтому проверка олимпиадных работ является таким же творчеством, как и их решение. 10 I О математических олимпиадах
По работе проверяющий должен восстановить логику рассуждений участника и оценить степень их достоверности и полноты. И наиболее успешно эту работу могут выполнить бывшие «олимпийцы». ▼ История математических олимпиад Любое эмпирическое знание становится научным фактом только после того, как оно облечено в числа и формулы и найдено логическое рассуждение, обосновывающее верность этих формул. Поэтому математика, фактически являясь языком науки, занимает центральное место в системе не только естественно-научных, но и гуманитарных знаний. И успешность в любой науке требует владения математическим аппаратом. В то же время в математике часто возникают (и имеют практический смысл) объекты, весьма далекие от реального мира. А потому круг логических конструкций, возникающих в математике, очень широк. Эти качества математики, видимо, стали причиной того, что математические соревнования появились и завоевали популярность раньше, чем другие научные соревнования. Математические олимпиады имеют давнюю историю. Первый очный математический конкурс для выпускников лицеев был проведен в Румынии в 1886 году, а первая математическая олимпиада в современном смысле состоялась в 1894 году в Венгрии по инициативе Венгерского физико-математического общества, возглавляемого будущим Нобелевским лауреатом по физике Л. Этвешом. С тех пор с перерывами, вызванными двумя мировыми войнами, эти олимпиады проводились ежегодно. Отметим, что первые олимпийские игры современности прошли в Афинах в 1896 году. Во многих странах олимпиадам предшествовали различные заочные конкурсы по решению задач. Так, например, в России они начали проводиться с 1886 года. Первая математическая олимпиада в Советском Союзе состоялась в Ленинграде в 1934 году, а ее инициаторами стали члены-корреспонденты АН СССР Л. Г. Шнирельман и Б. Н. Делоне. На следующий год будущие академики А. Н. Колмогоров и П. С. Александров провели первую олимпиаду в Москве. Изначально подчеркивалось, что олимпиады не спорт, а средство отбора и развития талантливых ребят. Не слу- 0 математических олимпиадах 11
чайно на первых олимпиадах действовало правило: победитель не допускается к участию в следующем году. Позже Московский и Ленинградский университеты стали проводить олимпиады по физике и химии. До войны олимпиады проводились ежегодно и быстро завоевали популярность. Сразу после войны они были возобновлены и проводились первоначально только в больших городах, где были сильные университеты. В конце 50-х — начале 60-х годов прошлого столетия математические олимпиады стали традиционными для многих городов Советского Союза, их проводили университеты и пединституты совместно с органами народного образования. В Советском Союзе идея олимпиады объединила научных работников, преподавателей вузов, аспирантов, студентов, которые стремились выявить одаренных молодых людей, помочь их становлению. Этот общественный феномен был замечен и поддержан государством, нуждавшимся в период «холодной войны» в первоклассных инженерах и специалистах в естественных науках. Первой математической олимпиадой, в которой приняли участие несколько областей РСФСР, стала проводившаяся в Москве олимпиада 1960 года. Ее иногда называют «нулевой» Всероссийской математической олимпиадой школьников. Официальная нумерация началась с 1961 года. На первую Всероссийскую математическую олимпиаду приехали команды почти всех областей РСФСР. Также были приглашены команды союзных республик. Фактически эти олимпиады стали всесоюзными, ведь в них принимали участие победители республиканских олимпиад. С 1967 года эта олимпиада получила официальное название — «Всесоюзная олимпиада школьников по математике». Всероссийская олимпиада школьников по математике организационно оформилась в 1974 году, когда по инициативе Министерства просвещения РСФСР, Министерства высшего образования РСФСР, общества «Знание» РСФСР и Центрального комитета ВЛКСМ был создан Центральный оргкомитет Всероссийской физико-математической и химической олимпиады школьников. Первыми руководителями математической части этой олимпиады стали профессор Московского государственного университета член- корреспондент АН СССР (ныне академик) В. И. Арнольд и доцент Московского физико-технического института А. П. Савин. 12 О математических олимпиадах
Центральным оргкомитетом и методическими комиссиями по физике, математике и химии были разработаны структура, задачи и цели олимпиады. Территория Российской Федерации была разделена на четыре зоны: Северо- Западную, Центральную, Юго-Западную и Сибири и Дальнего Востока (начиная с 2001 года, было введено новое деление — на семь федеральных округов: Южный, Центральный, Северо-Западный, Приволжский, Уральский, Сибирский и Дальневосточный). В отдельные зоны были выделены города Москва и Ленинград, в которых математические олимпиады начали проводиться еще в 30-е годы. Организаторами олимпиады было решено: в этих городах олимпиаду проводить по традиционно сложившейся схеме. Этот особый статус Москвы и Ленинграда (ныне Санкт-Петербург) сохранился и до сих пор. Согласно Положению об олимпиаде, Всероссийская олимпиада школьников по математике до 1992 года проводилась в четыре этапа: школьный, районный (городской), областной (краевой, республиканский) и зональный. До 1992 года заключительный этап республиканской математической олимпиады проводился во всех республиках Советского Союза, кроме РСФСР. Заключительный этап Всероссийской олимпиады заменяла Всесоюзная математическая олимпиада, на которой Российскую Федерацию представляли шесть команд — это команды городов Москвы и Ленинграда и четырех указанных выше зон (Северо- Западной, Центральной, Юго-Западной и Сибири и Дальнего Востока). В 1992 году в связи с распадом Советского Союза Всесоюзная олимпиада проводилась под названием Межреспубликанской. В том же году в последний раз бывший Советский Союз был представлен единой командой СНГ на Международной математической олимпиаде. Кроме того, в олимпиаде приняли участие и команды ставших независимыми государств, в том числе и России. А с 1992/93 учебного года стал проводиться пятый (заключительный) этап Всероссийской олимпиады школьников, и первым городом, принявшим у себя финал Всероссийской олимпиады, стала Анапа. В последующие годы заключительные этапы Всероссийской математической олимпиады проходили трижды в Майкопе, дважды в Твери и по одному разу в Казани, Калуге, Нижнем Новгороде, Орле, Пскове, Рязани, Саратове, Чебоксарах, Ярославле. О математических олимпиадах 13
ГЛАВА 2 труктура олимпиады Согласно Положению о Всероссийской олимпиаде школьников, олимпиада по математике проводится в пять этапов. Первый (школьный) этап проводится в октябре для учащихся 5—11 классов. Второй (районный, городской) этап проводится в ноябре — декабре для учащихся 6—11 классов. Третий (региональный) этап проводится в январе — феврале для учащихся 8—11 классов. Четвертый (федеральный окружной) этап проводится в семи округах (Южном, Уральском, Центральном, Приволжском, Сибирском, Северо-Западном, Дальневосточном) в марте. Олимпиада проводится для учащихся 8—11 классов. Городские олимпиады Москвы и Санкт-Петербурга по статусу приравнены к федеральным окружным. Пятый (заключительный) этап проводится в апреле для учащихся 9—11 классов. На каждом этапе участниками олимпиады могут быть и учащиеся младших классов, успешно прошедшие отбор на предыдущих этапах. 'Ф Школьные олимпиады Первый (школьный) этап олимпиады проводится общеобразовательными учреждениями. Его участником может быть каждый школьник. Вся организационная и методическая работа по его проведению обеспечивается педагогическими коллективами школ. Курируется первый этап городскими (муниципальными) органами управления образованием. Сроки и условия проведения олимпиады определяет образовательное учреждение самостоятельно. 14 Структура олимпиады
▼ Районные (городские) олимпиады Второй (муниципальный) этап олимпиады проводится городскими (районными) органами управления образованием по заданиям, разработанным муниципальными предметными комиссиями. В ряде областей второй этап олимпиады проводится по единым заданиям, подготовленным методической комиссией субъекта Российской Федерации. Второй этап олимпиады проходит в один день, как правило, в выходной. Для организации и проведения второго этапа олимпиады муниципальный орган управления образованием создает оргкомитет, предметные комиссии и жюри, в состав которых наряду с представителями образовательных и научных учреждений, органов управления образованием, могут входить члены Оргкомитета и Жюри третьего этапа. Место, сроки и условия проведения олимпиады определяются муниципальным органом управления образованием. Участниками второго этапа олимпиады являются победители и призеры первого этапа, а также победители и призеры второго этапа олимпиады предыдущего года. По решению муниципальных органов управления образованием второй этап олимпиады может носить открытый характер. ▼ Региональные (областные, республиканские) олимпиады Третий (региональный) этап олимпиады проводится государственными органами управления образованием субъектов Российской Федерации одновременно во всех субъектах Российской Федерации, в сроки, определенные Министерством образования и науки Российской Федерации. Третий этап олимпиады проходит, как правило, в два тура. Третий этап олимпиады проводится по методическим рекомендациям, разработанным Центральной предметной Методической комиссией по математике. Для организации и проведения третьего этапа олимпиады государственный орган управления образованием субъекта Российской Федерации создает Оргкомитет и Жюри. Жюри формируются из математиков — ведущих педагогов региона, а также преподавателей, аспирантов и студентов вузов. Структура олимпиады 15
Место и условия проведения третьего этапа определяются государственным органом управления образованием субъекта Российской Федерации самостоятельно. Для обучающихся закрытых административно-территориальных образований, отдаленных военных городков и гарнизонов, расположенных за пределами Российской Федерации, третий этап олимпиады проводится также по заданиям, разработанным Центральной предметной комиссией олимпиады. Место, сроки и условия проведения олимпиады определяются Федеральным агентством по образованию. Исторические Московская и Ленинградская (Санкт-Петербургская) городские математические олимпиады, сложившиеся ранее Всероссийских олимпиад, имеют свои традиции. Поэтому окружной этап олимпиады города Москвы и районный этап олимпиады города Санкт-Петербурга приравниваются по статусу к третьему этапу олимпиады; однако эти этапы проводятся по оригинальным заданиям и в свои, не совпадающие с региональным этапом, сроки. Участниками третьего этапа олимпиады являются победители и призеры второго этапа, а также победители и призеры третьего этапа олимпиады предыдущего года. По решению органов управления образованием субъектов Российской Федерации третий этап олимпиады может носить открытый характер. Как показывает опыт, участники из регионов, в которых второй и третий этапы носят открытый характер, успешней выступают на последующих этапах олимпиады. 0 Федеральные окружные олимпиады Четвертый (федеральный окружной) этап олимпиады проводится государственными органами управления образованием субъектов Российской Федерации одновременно во всех федеральных округах Российской Федерации, в сроки, определенные Министерством образования и науки Российской Федерации (Федеральным агентством по образованию) по предложению Методической комиссии по математике. Четвертый этап олимпиады проводится по заданиям, разработанным Центральной предметной комиссией олимпиады. Городские этапы олимпиад Москвы и Санкт-Петербурга приравниваются по статусу к четвертому этапу олимпиады. 16 Структура олимпиады
Состав участников четвертого этапа олимпиады определяется из числа победителей и призеров третьего этапа в соответствии с квотами, установленными Министерством образования и науки Российской Федерации. Участниками данного этапа также являются победители и призеры четвертого этапа олимпиады предыдущего года. ▼ Заключительный этап олимпиады Пятый (заключительный) этап олимпиады проводится государственными органами управления образованием субъектов Российской Федерации в апреле, в сроки, определенные Министерством образования и науки Российской Федерации (Федеральным агентством по образованию) по предложению Методической комиссии по математике. Пятый этап олимпиады проводится по заданиям, разработанным Центральной предметной комиссией олимпиады. Для организации и проведения пятого этапа олимпиады государственный орган управления образованием субъекта Российской Федерации, на территории которого проводится олимпиада по отдельному предмету, создает Оргкомитет и Жюри по согласованию с Министерством образования и науки России. В состав Оргкомитета и Жюри наряду с представителями образовательных и научных учреждений, общественных организаций, органов управления образованием входят представители Центрального оргкомитета, Центральной предметной Методической комиссии и Центрального жюри. Состав участников пятого этапа олимпиады определяется из числа победителей и призеров предыдущего этапа в соответствии с квотами, установленными Министерством образования и науки Российской Федерации. Участниками данного этапа также являются победители и призеры пятого этапа олимпиады предыдущего года. ▼ Методическая комиссия и Жюри олимпиады Остановимся на принципах формирования и работы Методической комиссии Всероссийской математической олимпиады школьников, на основе которой затем формируется Жюри заключительных этапов олимпиады. В Методическую комиссию входят преподаватели вузов и специализированных школ, сотрудники научных учреж- Структура олимпиады 17
дений Москвы, Санкт-Петербурга, Ярославля, Кирова, Калуги, Новосибирска, Иваново, а также члены редколлегии журнала «Квант». Также в работе Методической комиссии активно участвуют недавние «олимпийцы» — победители Всероссийских и Международных олимпиад последних лет — студенты и аспиранты ведущих вузов России (МГУ, МФТИ(ГУ), СПбГУ). Для проведения федерального окружного и заключительного этапов Всероссийской олимпиады по математике формируется Жюри, состоящее из членов Методической комиссии, а также математиков — ведущих специалистов по работе со школьниками региона, в котором проводится олимпиада. Составы Методической комиссии и Жюри ежегодно утверждаются приказами Министерства образования и науки РФ по представлению Центрального оргкомитета. Методическая комиссия ведет свою работу в течение всего года. При этом в широком составе она собирается на заседания для подготовки заданий III, IV и V этапов Всероссийской олимпиады школьников по математике. В конце октября начинается подготовка III (регионального) этапа олимпиады. На заседаниях комиссии собирается 20—25 человек. Они составляют банк заданий, в который, как правило, включается около 150 авторских задач, подготовленных членами Методической комиссии. К сожалению, как и в музыке, где далеко не все замечательные исполнители могут стать композиторами, в математических олимпиадах достаточно узок круг хороших заданных композиторов. Наиболее сильные традиции задачного творчества сохраняются на протяжении многих лет в Санкт-Петербурге. Вернемся к работе Методической комиссии. В первый день заседаний проходит обсуждение задач, поиск различных путей их решения, а также обобщение доказываемых утверждений. Во второй день Методическая комиссия разбивается на секции: геометрии, комбинаторики, алгебры и теории чисел, в которых происходит более детальное обсуждение задач и вырабатываются рекомендации по включению тех или иных задач на различные позиции в варианте. Например: «Задача № 63 геометрической секцией рекомендуется на позиции 10.3 или 10.7» (т. е. в качестве третьей по сложности в один из дней олимпиады 10 класса). Огромную роль в работе комиссии на этом этапе, когда нужно за короткое время просмотреть и оценить большое число задач, играют студенты — бывшие «олимпийцы». 18 Структура олимпиады
Вечером второго дня комиссия вновь собирается на общем заседании. Проходит черновое составление вариантов на основе рекомендаций секций. Нередко несколько задач претендуют на одну позицию. А некоторые позиции могут остаться незаполненными. Как правило, это первые (самые легкие*) задачи. Очень сложно придумать нетрудную, но в то же время новую и эстетически привлекательную задачу. После чернового составления вариантов часть членов Методической комиссии занимается обсуждением и отработкой формулировок и решений задач, включенных в вариант, а другая часть — составлением новых задач на вакантные позиции. Кроме того, Методическая комиссия готовит список критериев оценки решений по каждой задаче. Это необходимо для унификации оценки работ участников III этапа, поскольку в отличие от заключительных этапов олимпиады (IV и V) в состав Жюри не входят представители Методической комиссии. (На II этапе, где основную роль играет привлечение большого числа школьников к занятиям математикой, задания являются достаточно простыми и Жюри самостоятельно легко устанавливает критерии оценивания.) Эта работа продолжается более узким кругом Методической комиссии (в том числе кураторами классов) на протяжении двух-трех недель. Параллельно проходит компьютерный набор заданий и решений, подготовка и вычитка оригинал-макета. Окончательная версия заданий III этапа передается в Министерство образования и науки РФ. Отметим, что к каждому предлагаемому заданию дается подробное решение. Если задача имеет два принципиально различающихся метода решения, то приводятся оба, хотя нередко участникам олимпиады удается найти и неизвестный ранее, до олимпиады, способ решения. Иногда в случаях, когда решение школьника проще и элегантнее решения, придуманного Жюри, на олимпиаде он получает специальный приз «За оригинальное решение задачи». На Международной олимпиаде также вручается подобная награда. По такой же схеме в конце января или в феврале проходят заседания Методической комиссии при подготовке IV (федерального окружного) этапа, а также в марте или в начале апреля — при подготовке V (заключительного) этапа Всероссийской олимпиады. Учитывая возрастающую труд- * О сравнительной сложности задач см. параграф «Структура варианта». Структура олимпиады 19
ность этапов олимпиады, члены Методической комиссии предлагают наиболее сложные задачи на заседаниях по заключительному этапу. Кроме того, третий этап олимпиады носит достаточно массовый характер, многие его участники не обучаются в специализированных физико-математических школах. Поэтому нежелательным является включение в III этап задач, требующих от участников достаточно хорошо развитой математической техники или знания некоторых разделов математики, изучаемых только в специализированных школах. В то же время на заключительном этапе, являющемся одновременно и отборочным при формировании сборной команды России на Международную математическую олимпиаду, допустимо включение задач, в решениях которых используются некоторые классические теоремы математики, не входящие в стандартную школьную программу, но изучаемые на факультативных занятиях и в летних математических школах. К таким относятся, например, малая теорема Ферма, китайская теорема об остатках, неравенство Йенсена, теорема Эйлера об окружности девяти точек, теорема Холла о паросочетаниях и т. п. Не следует думать, что работа Методической комиссии и Жюри проходит в жесткой формальной обстановке. Находится место и шуткам. Одна из них была растиражирована популярным в СССР журналом. Так, в середине 70-х годов прошлого века в связи с проведением параллельно других мероприятий большое число призеров Московской городской олимпиады не явилось на награждение. Было решено организовать дополнительное награждение перед очередным занятием кружка, проводившегося в МГУ. На заседании Жюри (в котором участвовал и один из авторов этой книги) возникла и тут же получила горячую поддержку идея послать неформальное приглашение, в результате чего родилось следующее творение: Уважаемый ...! За успешное выступление на Московской городской математической олимпиаде Вы награждены наградой. Сообщаем, что награждение наградами награжденных, не награжденных наградами на награждении, проходившем ..., состоится .... Жюри Московской городской математической олимпиады 20 Структура олимпиады
Конечно, составители письма получили выговор от руководства Московской олимпиады, в который поступила масса недоуменных и гневных писем и звонков от родителей школьников. А через некоторое время в журнале «Крокодил» в разделе «Нарочно не придумаешь» появился текст этого приглашения с подписью: «прислал ... и еще большое количество читателей». Сейчас эта забавная фраза традиционно содержится в брошюрах Московской математической олимпиады. Иногда, помимо традиционных спецпризов и дипломов, появляются и шуточные награды. Например, школьника, решившего на олимпиаде только задачи с номерами 1, 3, 5 и 7, Жюри предложило наградить «ПоНечетной грамотой».
одготовка и проведение математических олимпиад Щ Тематика математических олимпиад Вопреки традиционному мнению в заданиях математических олимпиад очень мало задач на вычисления. В отличие от задач школьной математики, в которых проверяются вычислительные навыки учащихся, задачи олимпиад основаны на логике, на способности построения логической конструкции. Поэтому в основном в олимпиадных задачах требуется обоснование какого-либо математического утверждения. Часть условий задач так и начинаются со слов: «Докажите, что...» В некоторых задачах вначале требуется отыскать числовой ответ, а затем обосновать его. К такому классу относятся, например, задачи типа «оценка + пример», начинающиеся со слов: «Найдите наименьшее (наибольшее) число N, удовлетворяющее условию...» Первый этап решения — интуитивное получение ответа, второй — построение примера, реализующего ответ, третий — доказательство невозможности построения примера для меньшего (большего) значения п. Классическим примером такого класса является задача о наибольшем количестве ладей, которые можно расставить на шахматной доске так, чтобы они не били друг друга. Правильный ответ 8 легко угадать, и существует множество вариантов расстановки ладей требуемым способом. Но почему нельзя расставить большее число ладей? Очень просто: в каждую строку можно поставить с соблюдением условия задачи не более чем одну ладью. Значит, на всей доске их не может оказаться больше 8. Другим примером задач, в которых вначале требуется угадать ответ, являются задачи, вопрос в которых начинается со слов: «Существует ли...», «Верно ли...», «Можно ли...», «Кто выигрывает...». Составители олимпиадных заданий стремятся к тематическому разнообразию задач, учитывая при этом не только программу по математике соответствующего класса 22 Подготовка и проведение математических олимпиад
школы, но и возрастные особенности школьников. Учащиеся, перешедшие из младшего в среднее звено школы, имеют слабые навыки в построении четких логических конструкций (не умеют «строго доказывать»), поэтому большинство заданий для учащихся 5—7 классов — это задачи на интуицию, «на догадку», либо задачи, в которых обоснование является достаточно простым. Вот пример задачи, предлагавшейся на II туре для учащихся 6 класса: «В Ю мешках находятся золотые монеты, при этом в одном мешке все монеты фальшивые, в остальных мешках все монеты настоящие. Все настоящие монеты весят по 10 граммов, а все фальшивые монеты — по 9 граммов. Как с помощью ровно одного взвешивания на чашечных весах со стрелкой определить, в каком мешке находятся фальшивые монеты?» Решение. □ Нужно из первого мешка взять одну монету, из второго — две, из третьего — три и т. д. Тогда величина 550 - S, где S — суммарный вес взвешиваемых монет, и укажет номер мешка с фальшивыми монетами. ■ Фактически школьник, предложивший такую запись, уже решил задачу. Каждому после недолгих размышлений становится ясно, что предложенный метод является правильным. Строгим можно считать, например, такое обоснование предложенного метода. □ Если бы каждая монета была настоящей, то взвешиваемые монеты весили бы 10 + 20 + + 30 + ... + 100 = 550 грамм. Но каждая фальшивая монета ровно на 1 грамм легче, поэтому число 550 - S показывает, сколько фальшивых монет мы взвешиваем, а это и есть номер мешка. ■ Нередко в задания включаются числовые ребусы, задачи на разрезание, построение конструкций, простые логические задачи либо задачи на игры, в которых требуется угадать выигрышную стратегию одного из игроков. С психологической точки зрения желательным является включение в задания для учащихся 5—7 классов задач, формулировка которых носит повествовательный, игровой характер. Поэтому нередко в тексте присутствуют знакомые школьникам литературные или мультипликационные герои («Малыш и Карлсон по очереди съедают конфеты...»). Наиболее схожими по формулировке с задачами школьной математики являются задачи математических олимпиад для старшеклассников. Ведь решения задач, предлагаемых для старшеклассников, требуют владения всем курсом Подготовка и проведение математических олимпиад 23
школьной математики, наиболее сложные разделы которой как раз и изучаются в выпускных классах. Здесь хочется отметить заметное отличие тематики математических олимпиад для школьников в России и в странах, входивших в состав СССР, от тематики олимпиад в целом ряде других стран. В Советском Союзе в силу огромной протяженности страны олимпиады выступали основным инструментом поиска одаренных школьников, а также поддержки системы дополнительного (факультативного) математического образования. Поэтому олимпиады проводились для учащихся не только старшего, но и среднего звена и обязательно включали в себя задания, выявляющие в первую очередь творческие способности школьников, а не степень и качество их математической подготовки. В то же время во многих странах Европы, Америки и Азии продолжительность обучения в школе составляет 11—13 лет, и олимпиадные задачи построены на хорошем владении математической техникой. В таких задачах от школьника требуется не создание маленького «открытия» (нахождения новой — по крайней мере для него — идеи), а реализация своих знаний и навыков в относительно простой с творческой точки зрения ситуации. Как следствие, в России (Советском Союзе) сложилась и обрела богатые традиции «композиторская» математическая школа*. Во всем мире огромной популярностью пользуются сборники олимпиадных задач Всероссийских, а также городских Санкт-Петербургской и Московской олимпиад, ежегодно пополняющих копилку олимпиадных задач новыми, авторскими идеями. При этом особенно выделяются задачи по геометрии и комбинаторике. Это отличие наиболее заметно проявляется на Международных математических олимпиадах, где российские участники нередко превосходят соперников в решении комбинаторных задач, в которых основу составляет нахождение новой идеи, но заметно уступают своим основным соперникам — китайским школьникам в решении сложных технических задач. В Китае, например, как и в ряде других стран, олимпиадные задания едины для всех участников независимо от класса, в котором они обучаются, и тем самым основаны на владении всем изучаемым в школе материалом. * См. главу 2, пункт «Методическая комиссия и Жюри олимпиады». Подготовка и проведение математических олимпиад
т Структура варианта Вернемся к тематическим и структурным особенностям наших олимпиад. Методическая комиссия Всероссийской олимпиады школьников по математике формирует задания, исходя из следующих принципов: 1) Нарастание сложности заданий от первого к последнему. При этом их трудность должна быть такой, чтобы в каждый из двух дней олимпиады с первым заданием успешно справились примерно 70% участников, со вторым — около 50%, с третьим — около 20%, а с четвертым — лишь несколько участников. (Конечно, не всегда удается выдержать такие установки по трудности заданий, так как новые, авторские задачи не всегда точно соответствуют заданной сложности). 2) Тематическое разнообразие заданий. Каждый день в комплект должны входить задачи по геометрии, алгебре, комбинаторике, в старших классах желательно включение задач по теории чисел, тригонометрии, стереометрии, математическому анализу. (При этом допустимо и даже поощряется включение задач, объединяющее различные разделы школьной математики.) 3) Обязательная новизна задач. Недопустимой является ситуация, когда участник математической олимпиады заранее знаком с идеей решения задачи. (В этом заключается коренное отличие математических олимпиад от олимпиад по тем дисциплинам, в которых предлагаются тестовые задания, проверяющие как раз знание участником олимпиады тех или иных фактов, его начитанность, «эн- циклопедичность»). Олимпиады по математике в первую очередь проверяют способность ученика к творчеству, умение логически мыслить, а не объем его знаний. 4) Эстетическая красота заданий. В математике существует понятие «красивая задача». К таковым относят задачи, в которых сочетаются интересный с научной точки зрения факт, простота формулировки и элегантность решения. К ним относится, например, задача, получившая название «Полоски Климова» (автор задачи — победитель Международной математической олимпиады в составе команды СССР Аркадий Климов): «Прямоугольник разрезан на прямоугольники, у каждого из которых длина одной из сторон — целое число. Докажите, что и у исходного прямоугольника длина одной из сторон — целое число». Интересно, что у этой комбинаторной задачи, помимо Подготовка и проведение математических олимпиад 25
«симпатичного» комбинаторного решения, имеется и короткое неэлементарное решение, основанное на свойствах двойных интегралов. ▼ Организация проведения туров олимпиады Важной составляющей проведения олимпиады является организация условий для выполнения работ участниками. Помимо основных требований, предъявляемых к учебным аудиториям (освещение, проветривание и т. п.), необходимо учесть специфику проведения математических олимпиад. В кабинетах должны находиться чертежные принадлежности (карандаши, циркули, линейки). Участники должны быть обеспечены одинаковыми тетрадями в клеточку. На математических олимпиадах часто предлагаются задачи, в которых участвуют клетчатые фигуры. Также достаточно распространены задачи на раскраски и заполнение таблиц. Наличие тетради в клеточку избавляет участника от необходимости проведения дополнительной работы по разлиновыванию листов, на что требуется время. Однотипность тетрадей является необходимым требованием для шифровки работ. При проведении второго этапа дежурный по аудитории составляет список присутствующих. Начиная с третьего этапа олимпиады все участники заполняют анкеты. Рекомендуется подготовить инструкцию для дежурных по аудиториям (кабинетам). В эти инструкции, помимо прочих, традиционно включают следующие четыре основных пункта: 1- Обратите внимание на рассадку участников. Рассадите школьников так, чтобы учащиеся из одной школы (начиная с третьего этапа — из одного региона, города) не сидели рядом. Обратите внимание на организацию заполнения титульных листов и оформление работ. Напомните участникам, что фамилию, имя и отчество надо вписывать в титульный лист разборчиво, печатными буквами; в тексте работы не должно быть никаких указаний на ее авторство; в работе следует указывать, какая часть является чистовиком, а какая — черновиком. 26 Подготовка и проведение математических олимпиад
3. Обратите внимание участников на то, что мобильными средствами связи, а также калькуляторами на олимпиаде пользоваться нельзя. 4. Дежурный по аудитории отвечает только на организационные вопросы (оформление работ, выход из аудитории, время и т. п.). Ответы на вопросы по условиям осуществляет только дежурный член жюри. По окончании олимпиады представители Оргкомитета осуществляют шифровку работ. Обложки и протоколы шифровки хранятся в Оргкомитете до окончания проверки и определения победителей и призеров. Определение призеров и победителей олимпиады должно проводиться Жюри и Оргкомитетом до расшифровки работ. ▼ Организация проверки работ Задания математических олимпиад являются творческими, допускают несколько различных вариантов решений. Кроме того, необходимо оценивать частичные продвижения в задачах (например, разбор важного случая, доказательство леммы, нахождение примера и т. п.). Наконец, возможны логические и арифметические ошибки в решениях. Окончательные баллы по задаче должны учитывать все вышеперечисленное. Поэтому проверка работ на математических олимпиадах проводится в два этапа. На первом этапе Жюри производит проверку работ без выставления баллов по так называемой системе «плюс — минус». Знак выставляется в соответствии с приведенной таблицей. При этом предварительная оценка по системе «плюс — минус» может быть незначительно изменена после обсуждения критериев и классификации случаев. Знак + +. Правильность (ошибочность) решения Полное верное решение. Верное решение. Имеются небольшие недочеты, в целом не влияющие на решение. Подготовка и проведение математических олимпиад 27
Продолжение Знак ± +/2 + -. - 0 Правильность (ошибочность) решения Решение в целом верное. Однако решение содержит существенные ошибки либо пропущены случаи, не влияющие на логику рассуждений. Верно рассмотрен один из двух (более сложный) существенных случаев или в задаче типа «оценка + пример» верно получена оценка. Доказаны вспомогательные утверждения, помогающие в решении задачи. Рассмотрены отдельные важные случаи при отсутствии решения. Решение неверное, продвижение отсутствует. Решение отсутствует. Иногда выставляется оценка «+!», чтобы отметить правильное красивое решение. Как правило, подобные решения отмечаются спецпризами. По окончании первого этапа группа проверяющих по каждой задаче, анализируя и обобщая приведенные решения, выделяет различные способы решения, типичные частичные продвижения, основные ошибки. В соответствии со сравнительным анализом различных продвижений вырабатывается шкала критериев оценивания. На втором этапе выставляются окончательные баллы по каждой задаче. В соответствии с регламентом проведения математических олимпиад школьников каждая задача оценивается по 7 баллов. В таблице приведена шкала перевода знаков в баллы. Знак + +. ± Балл 7 6-7 5—6 28 Подготовка и проведение математических олимпиад
+/2 + -. - 0 4 2-3 0—1 0 0 Важно отметить, что любое правильное решение оценивается в 7 баллов. Недопустимо снимать баллы за то, что решение слишком длинное, или за то, что решение школьника отличается от приведенного в методических разработках или от других решений, известных Жюри. В то же время любой сколь угодно длинный текст решения, не содержащий полезных продвижений, должен быть оценен в 0 баллов. Традиционной ошибкой школьников при решении задач на доказательство является использование доказываемого утверждения в качестве начального условия. Например, в задаче требуется доказать, что треугольник является равносторонним, а доказательство начинается со слов: «Пусть треугольник ABC — равносторонний». Подобные «решения» оцениваются в 0 баллов в силу грубой логической ошибки. Еще раз остановимся на задачах на нахождение наибольшего (наименьшего) значения некоторой величины (задачи типа «оценка + пример»). Решение таких задач включает в себя два шага. 1. Обычно более сложный шаг (оценка) — доказательство того, что некоторая величина не больше (не меньше) некоторого значения. 2. Построение примера, показывающего достижимость указанного значения. Как правило, только первый шаг оценивается в 4 балла, второй шаг — в 1—2 балла. Решения школьников, особенно на последних этапах олимпиады, часто бывают очень длинными и запутанными; никакой, даже сколь угодно квалифицированный член Жюри, не может гарантировать, что он поймет все логические шаги, содержащиеся в работе. Поэтому, начиная с третьего этапа, каждая работа оценивается и проверяется (перепроверяется) не менее чем двумя членами Жюри. Подготовка и проведение математических олимпиад 29
На втором этапе рекомендуется повторная проверка работ победителей и призеров до расшифровки работ. Приведем конкретные примеры оценивания работ третьего этапа XXXII Всероссийской олимпиады школьников по классам. Конкретные критерии оценивания даны для задач, условия которых приведены на с. 51—56 данной книги. Приводятся оценки за типичные продвижения в решении. Ф 8 класс 8.1. Замечено, что каждый лжец (рыцарь) знает одно и то же число лжецов и рыцарей, — 1 балл. Замечено, что количество лжецов, которых знает рыцарь, и количество лжецов, которых знает лжец, имеют разную четность — 3 балла. 8.2. Только ответ без примера и оценки — 0 баллов. Ответ и пример без оценки — 3 балла. Ответ и оценка без примера (или с неправильным примером) — 3 балла. 8.3. Только ответ без примера и оценки — 0 баллов. Ответ и пример без оценки — 2 балла. Ответ и оценка без примера (или с неправильным примером) — 4 балла. 8.5. Указана правильная выигрышная стратегия, но не проверено, что выписываются различные числа, — 5 баллов. 8.6. Только ответ без примера и оценки — 0 баллов. Ответ и пример без оценки — 2 балла. Ответ и оценка без примера (или с неправильным примером) — 3 балла. 8.7. Угаданный ответ — 0 баллов. Получена явная формула для нахождения числа пирожных, съеденных Карлсоном (НЕ рекуррентное соотношение), — 2 балла. 30 Подготовка и проведение математических олимпиад
г 9 класс 9.1. Сформулировано утверждение, что последний шаг сделать не удастся. Однако данное утверждение не обосновано — 2 балла. 9.2. Только ответ — 0 баллов. Сформулировано, но не доказано неравенство, помогающее в решении, — 2 балла. Сформулировано, но не доказано утверждение о том, что на конце факториала больше нулей, — 2 балла. Если сформулированы, но не доказаны оба предыдущих утверждения — 2 балла. 9.4. Разобран один из двух случаев «Президент — рыцарь» и «Президент — лжец» — 4 балла. Доказана нужная лемма — 1 балл. 9.5. Угаданный ответ — 0 баллов. Получена явная формула для нахождения числа пирожных, съеденных Карлсоном (НЕ рекуррентное соотношение), — 2 балла. 9.8. Угаданный ответ — 0 баллов. ▼■ 10 класс 10.1. Сформулировано утверждение, что последний шаг сделать не удастся. Однако данное утверждение не обосновано — 2 балла. 10.3. Доказано, что а3 и Ь3 делятся на ab> — 2 балла. 10.4. Доказано одно из утверждений «Из равенства углов следует вписанность» или «Из вписанности следует равенство углов» — 4 балла. Не доказано, что из равенства отношений соответствующих синусов следует равенство углов, — не более 6 баллов. Подготовка и проведение математических олимпиад 31
10.5. Приведен ответ — 1 балл. Найдены возможные значения свободного члена - 3 балла. Не проведен отбор решений — снять по 1 баллу s каждый лишний ответ (но не более 3 снятых баллов). 10.6. Только ответ без примера и оценки — 1 балл. Ответ и пример без оценки — 3 балла. Ответ и оценка без примера (или с неправильным npi мером) — 4 балла. 10.8. Только ответ без примера и оценки — 1 балл. Ответ и пример без оценки — 2 балла. Ответ и оценка без примера (или с неправильным пр] мером) — 3 балла. 11 класс Арифметическая ошибка, не влияющая на ход реш ния, — снять 1 балл. 11.3. Не снимать баллы за использование без доказательст] неравенств о средних. Доказано, что х + у ^ 2, — 5 баллов. 11.5. Доказано, что число 1 содержится в прогрессии, 4 балла. 11.6. Доказано равенство каких-то двух ребер — 2 балла. 11.7. Только ответ без примера и оценки — 1 балл. Ответ и пример без оценки — 3 балла. Ответ и оценка без примера (или с неправильным пр мером) — 3 балла. 11.8. Ответ без объяснений — 0 баллов. Разобран только случай четного N — 1 балл. Разобран только случай нечетного N — 4 балла. 32 Подготовка и проведение математических олимпиад
г Определение победителей и призеров Победителями олимпиады соответствующего этапа считаются участники, награжденные дипломами первой степени. Призерами олимпиады соответствующего этапа считаются участники, награжденные дипломами второй и третьей степени. Другие участники могут награждаться дипломами участника, грамотами, специальными призами. Важно отметить, что победителем (призером) олимпиады в параллели не обязательно должен быть ровно один ученик. На математических олимпиадах, как на Международных, так и на Всероссийских (в обязательном порядке — на 4 и 5 этапах), принята практика награждения дипломами до 45% всех участников олимпиады. Кроме того, расхождение результатов двух школьников в 1— 2 балла может отражать только умение одного из них более четко записывать решения. Поэтому рекомендуемая схема распределения дипломов такова. Школьник, набравший наибольшее количество баллов, а также все школьники, решившие одинаковое с ним число задач (засчитыва- ются задачи, по которым поставлено не менее 5 баллов), награждаются дипломами первой степени. Школьники, решившие на 1—2 задачи меньше (набравшие на 5—15 баллов меньше), награждаются дипломами второй степени. Следующая группа школьников (решивших еще на 1— 2 задачи меньше) награждается дипломами третьей степени. Отметим, что в ряде случаев в варианте оказывается слишком много сложных задач. Поэтому отсутствие в параллели ученика, решившего все задачи, не должно означать отказ от присуждения диплома первой степени лучшему из участников. Аналогичные замечания касаются второго и третьего дипломов. На олимпиадах начальных (первого, второго) этапов с учетом возрастных особенностей рекомендуется награждение дипломами и грамотами значительного (до 60%) числа учащихся младших классов (5—6 класс) в целях развития у них интереса к дополнительным занятиям математикой. ▼'■ Особенности подготовки и проведения школьных олимпиад Школьная олимпиада проводится в один день для учащихся 5—11 классов. Рекомендуемое время проведения: для 5—6 классов — Подготовка и проведение математических олимпиад 33
2 урока, для 7—8 классов — 3 урока, для 9—11 классов — 4 урока. Вариант должен содержать 4—6 задач разной сложности. Желательно, чтобы задания охватывали все разделы школьной математики, изученные к моменту проведения олимпиады. Первые две (самые легкие) задачи варианта должны быть доступны подавляющему большинству участников. В качестве сложных задач рекомендуется включать в вариант задачи, для решения которых используются материалы, изучаемые на факультативных занятиях. Например, по темам: «Четность», «Делимость», «Принцип Дирихле», «Логика», «Игры», «Комбинаторика». Подготовка заданий и их проверка осуществляются учителями математики, руководителями кружков и факультативов (школьными методическими объединениями учителей математики). <▼> Особенности подготовки и проведения районных олимпиад Районная олимпиада проводится в один день для учащихся 6—11 классов. Следует допускать для участия в олимпиаде за 6 класс учащихся 4—5 классов по рекомендации ведущих учителей. Во многих регионах России в олимпиаде могут принять участие все желающие. В некоторых регионах в число участников дополнительно включаются школьники, рекомендованные руководителями школьных и городских математических кружков и факультативов. Рекомендуемое время проведения: для 6—7 классов — 3 часа, для 8—11 классов — 4 часа. Вариант должен содержать 5—6 задач разной сложности. Обязательным является требование включения в вариант заданий по темам, изученным к моменту проведения олимпиады в соответствии с программами всех базовых учебников по математике. Первые две (самые легкие) задачи варианта должны быть доступны подавляющему большинству участников. Рекомендуется включать в вариант задачи, использующие материалы, изучаемые на факультативных занятиях. Для проверки заданий формируется Жюри, в состав которого включаются учителя математики из разных школ, руководители кружков и факультативов. Рекомен- 34 Подготовка и проведение математических олимпиад
дуется включение в состав Жюри студентов, аспирантов и преподавателей математических факультетов университетов, технических и педагогических вузов региона. ▼ Особенности подготовки и проведения региональных олимпиад Региональная олимпиада проводится, как правило, в два тура. Олимпиада проводится для учащихся 8—11 классов. По согласованию с Оргкомитетом и по рекомендации Жюри второго этапа, допускается участие в олимпиаде учащихся 6—7 классов. Время проведения каждого тура — 4 часа. При проведении олимпиады в два дня — каждый день школьникам предлагается решить 4 задачи. При проведении олимпиады в один день — задание включает 5 задач. В некоторых случаях в силу больших размеров территории региона допускается проведение третьего тура Всероссийской олимпиады одновременно в нескольких городах региона. В этом случае работы доставляются в единый центр для последующей шифровки и проверки работ. ▼ Особенности подготовки и проведения IV—V этапов олимпиады Четвертый и пятый этапы олимпиады проводятся в два тура. Четвертый (федеральной окружной) этап олимпиады проводится для учащихся 8—11 классов. Время проведения каждого тура — 4,5 часа. Еще раз отметим, что в окружном этапе должны принимать участие, в соответствии с квотой, победители третьего этапа — даже в том случае, если они учатся в более младших классах. Например, учащийся 7 класса, победивший в областной олимпиаде по 8 классам, должен быть включен в число участников окружной олимпиады по 8 классам. Пятый (заключительный) этап олимпиады проводится для учащихся 9—11 классов (разумеется, в олимпиаде также принимают участие учащиеся и более младших классов, ставшие победителями и призерами федеральной окружной олимпиады по указанным классам). В заключительном этапе принимают участие около 60 человек в каж- Подготовка и проведение математических олимпиад 35
дой параллели. Время проведения каждого тура — 5 часов. В связи со сложностью предлагаемых заданий и трудностью изложения участниками четкого решения за ограниченное время олимпиады, у Жюри олимпиады при проверке возникают сложности по однозначности трактовки записанного решения. Поэтому по окончании проверки работ до окончательного определения победителей и призеров на третьем, четвертом и пятом этапах олимпиады Жюри проводит разбор задач и показ работ участникам олимпиады. На разборе Жюри рассказывает типичные ошибки и критерии оценивания работ, выработанные в процессе проверки в соответствии с рекомендациями Методической комиссии. На показе работ Жюри разбирается вместе с участниками во всех спорных ситуациях. На показе работ после обсуждения по конкретной задаче оценка может быть исправлена (только по согласованию с председателем Жюри или его заместителем). Важно отметить, что если на показе работ участник приводит правильное решение, основанное на своих записях, но предложенное решение не содержится в полном виде в работе, то оценивается доля решения, присутствующая в работе. В исключительных случаях, если проверяющий и участник не могут прийти к единому мнению по оценке работы, участник имеет право подать в течение двух часов с момента окончания показа работ письменную апелляцию. Апелляция рассматривается комиссией, возглавляемой председателем Жюри, в течение одного дня с момента ее подачи. В силу того что победители заключительных этапов имеют законодательно утвержденные льготы при поступлении в учебные учреждения высшего профессионального образования, дипломы победителей четвертого и пятого этапов олимпиады являются документами строгой отчетности. Ежегодно по итогам пятого этапа олимпиады Министерство образования и науки РФ издает приказ, утверждающий список победителей.
ГЛАВА 4 а лимпиада 2005/2006 учебного года II ЭТАП (районный) УСЛОВИЯ ЗАДАЧ* ▼ 6 класс 6.1. Поставьте вместо звездочек в выражение * + ** + *** + **** = 3330 десять различных цифр так, чтобы получилось верное равенство. 6.2. Коммерсант Вася занялся торговлей. Каждое утро он покупает товар на некоторую часть имеющихся у него денег (возможно, на все имеющиеся у него деньги). После обеда он продает купленный товар в два раза дороже, чем купил. Как нужно торговать Васе, чтобы через 5 дней у него было ровно 25 000 рублей, если сначала у него было 1000 рублей. 6.3. Проезжая по лесной дороге, Иван-царевич встретил медведя, волка и лису. Медведь всегда говорит правду, лиса всегда лжет, а волк чередует правду и ложь, всегда начиная с правды. Звери сказали Ивану-царевичу по 2 предложения. 1-й: «Ты коня спасешь». «Но сам погибнешь». 2-й: «Ты целым-невредимым останешься». «И коня спасешь». 3-й: «Ты цел останешься». «А вот коня потеряешь». Определите, какому зверю принадлежит каждый ответ и что ждет Ивана-царевича впереди. 6-4. Разрежьте квадрат 3 х 3 на две части и квадрат 4x4 на две части так, чтобы из получившихся четырех кусков можно было сложить квадрат. Авторы задач и составители: Н. X. Агаханов, О. К. Подлипский. II ЭТАП. 2005/2006 учебный год | 37
6.5. В таблицу 2x5 записали все натуральные числа от 1 до 10. После этого подсчитали каждую из сумм чисел по строке и по столбцу (всего получилось 7 сумм). Какое наибольшее количество этих сумм может оказаться простыми числами? ■у* 7 класс 7.1. Коммерсант Вася занялся торговлей. Каждое утро он покупает товар на некоторую часть имеющихся у него денег (возможно, на все имеющиеся у него деньги). После обеда он продает купленный товар в два раза дороже, чем купил. Как нужно торговать Васе, чтобы через 5 дней у него было ровно 25 000 рублей, если сначала у него было 1000 рублей. 7.2. На смотре войска Острова Лжецов и Рыцарей (лжецы всегда лгут, рыцари всегда говорят правду) вождь построил всех воинов в шеренгу. Каждый из воинов, стоящих в шеренге, сказал: «Мои соседи по шеренге — лжецы». (Воины, стоящие в концах шеренги, сказали: «Мой сосед по шеренге — лжец».) Какое наибольшее число рыцарей могло оказаться в шеренге, если на смотр вышли 2005 воинов? 7.3. У продавца есть стрелочные весы для взвешивания сахара с двумя чашками. Весы могут показывать вес от 0 до 5 кг. При этом сахар можно класть только на левую чашку, а гири можно ставить на любую из двух чашек. Какое наименьшее количество гирь достаточно иметь продавцу, чтобы взвесить любое количество сахара от 0 до 25 кг? Ответ объясните. 7.4. Для натурального числа N вычислили суммы всех пар соседних цифр (например, для N = 35 207 суммы составляют {8, 7, 2, 7}). Найдите наименьшее N, для которого среди этих сумм есть все числа от 1 до 9. 7.5. Клетки таблицы 8x8 покрашены в три цвета. Оказалось, что в таблице нет трехклеточного уголка, все клетки которого одного цвета (трехклеточный уголок — это фигура, получаемая из квадрата 2x2 удалением одной клетки). Также оказалось, что в таблице нет трехклеточного уголка, все клетки которого трех разных цветов. Докажите, что количество клеток каждого цвета четно. 38 XXXII Всероссийская математическая олимпиада школьников
г 8 класс 8.1. Вася возвел натуральное число А в квадрат, записал результат на доску и стер последние 2005 цифр. Могла ли последняя цифра оставшегося на доске числа равняться единице? 8.2. Набор, состоящий из целых чисел а, Ь, с, заменили на набор а-1, Ь + 1, с2. В результате получившийся набор совпал с исходным. Найдите числа ау Ъу су если известно, что их сумма равна 2005. 8.3. На смотре войска Острова Лжецов и Рыцарей (лжецы всегда лгут, рыцари всегда говорят правду) вождь построил всех воинов в шеренгу. Каждый из воинов, стоящих в шеренге, сказал: «Мои соседи по шеренге — лжецы». (Воины, стоящие в концах шеренги, сказали: «Мой сосед по шеренге — лжец».) Какое наибольшее число лжецов могло оказаться в шеренге, если на смотр вышли 2005 воинов? 8.4. Найдите углы прямоугольного треугольника, если известно, что точка, симметричная вершине прямого угла относительно гипотенузы, лежит на прямой, проходящей через середины двух сторон треугольника. 8.5. Хромая ладья (это ладья, которая может ходить только по горизонтали или только по вертикали ровно на 1 клетку) обошла доску 10 х 10 клеток, побывав на каждой клетке ровно по одному разу. В первой клетке, где побывала ладья, запишем число 1, во второй — число 2, в третьей — 3 и т. д. до 100. Могло ли оказаться так, что сумма чисел, записанных в двух соседних по стороне клетках, делится на 4? т 9 класс 9.1. Вася взял 11 подряд идущих натуральных чисел и перемножил их. Коля взял эти же 11 чисел и сложил их. Могли ли две последние цифры результата Васи совпасть с последними двумя цифрами результата Коли? 9-2. Набор, состоящий из чисел а, Ь, с, заменили на набор а4 - 262, б4 — 2с2, с4 - 2а2. В результате получившийся набор совпал с исходным. Найдите числа ауЪу су если их сумма равна —3. II ЭТАП. 2005/2006 учебный год | 39
9.3. На основании АС треугольника ABC взята точка D. Докажите, что окружности, вписанные в треугольники ABD и CBDy точками касания не могут делить отрезок BD на три равные части. 9.4. Каждая из точек плоскости покрашена в один из трех цветов, причем все три цвета используются. Верно ли, что при любой такой покраске можно выбрать окружность, на которой есть точки всех трех цветов? 9.5. В клетках таблицы 8x8 расставлены целые числа. Оказалось, что если выбрать любые три столбца и любые три строки таблицы, то сумма девяти чисел, стоящих на их пересечении, будет равна нулю. Докажите, что все числа в таблице равны нулю. * 10 класс 10.1. Синус и косинус некоторого угла оказались различными корнями квадратного трехчлена ах2 + Ъх + с. Докажите, что Ъ2 = а2 + 2ас. 10.2. Каждая из точек плоскости покрашена в один из трех цветов, причем все три цвета используются. Верно ли, что при любой такой покраске можно выбрать окружность, на которой есть точки всех трех цветов? 10.3. Решите в натуральных числах уравнение НОК (а; Ъ) + НОД (а; Ъ) = аЪ. (НОД — наибольший общий делитель, НОК — наименьшее общее кратное). 10.4. Окружность, вписанная в треугольник АВСУ касается сторон АВ и ВС в точках Е и F соответственно. Точки М и N — основания перпендикуляров, опущенных из точек А и С на прямую EF. Докажите, что если стороны треугольника ABC образуют арифметическую прогрессию и АС — средняя сторона, то ME + FN = EF. 10.5. Вася назвал натуральное число N. После чего Петя нашел сумму цифр числа N> потом сумму цифр числа N + IN у потом сумму цифр числа N + 2 • 7Ny потом сумму цифр числа N + 3 • IN и т. д. Мог ли он каждый следующий раз получать результат, больший предыдущего? 40 XXXII Всероссийская математическая олимпиада школьников
г 11 класс -11.1. Найдите какую-нибудь функцию f (х), отличную от константы, такую, что она не принимает отрицательных значений и для любого а выполняется неравенство / (sin а) + / (cos а) ^ 4/ (sin а cos а). 11.2. Для каждого из 8 сечений куба с ребром а, являющихся треугольниками с вершинами в серединах ребер куба, рассматривается точка пересечения высот сечения. Найдите объем многогранника с вершинами в этих 8 точках. 11.3. Пусть у = kxx + Ъ19 y = k2x + b2, у = кгх + Ъг — уравнения трех касательных к параболе у = х2. Докажите, что если k3 = kx + ft2, то b3 ^ 2 (bx + b2). 11.4. Вася назвал натуральное число N. После чего Петя нашел сумму цифр числа iV, потом сумму цифр числа N + 13ЛГ, потом сумму цифр числа N + 2 - 13iV, потом сумму цифр числа N + 3 ■ 13iV и т. д. Мог ли он каждый следующий раз получать результат, больший предыдущего? 11.5. Можно ли нарисовать на плоскости 2005 ненулевых векторов так, что из любых десяти из них можно выбрать три с нулевой суммой? РЕШЕНИЯ ЗАДАЧ Т 6 класс 6.1. Например, 5 + 40 + 367 + 2918 = 3330. 6.2. Один из вариантов следующий. Первые четыре дня Вася должен покупать товар на все имеющиеся у него деньги. Тогда через четыре дня у него будет 16 000 рублей (1000 -+ 2000 -+ 4000 -+ 8000 -+ 16 000). На пятый день он должен купить товар на 9000 рублей. У него останется 7000 рублей. После обеда он продаст товар за 18 000 рублей, и у него станет ровно 25 000 рублей. 6-3. Ответ. 1-й — лисе, 2-й — волку, 3-й — медведю. Иван- царевич цел останется, но коня потеряет. II ЭТАП. 2005/2006 учебный год 41
Среди ответов есть два полностью противоположных: первый и третий. Значит, второй ответ принадлежит волку, а тогда правильным является третий ответ. Замечание. За правильный ответ без объяснения — 2 балла. 6.4. Ответ. Два из возможных примеров разрезания приведены на рисунках 1 и 2. Рис. 1 Рис. 2 6.5. Ответ. 6. Если бы все 7 сумм были бы простыми числами, то простыми в частности были бы две суммы по 5 чисел. Каждая из этих сумм больше 5. Если бы обе эти суммы были простыми числами больше 5, то каждая из этих сумм была бы нечетной (поскольку только 2 является простым четным числом). Но если мы сложим эти суммы, получим четное число. Однако в эти две суммы входят все числа от 1 до 10, и их сумма равна 55 — числу нечетному. Поэтому среди полученных сумм не больше 6 будут простыми числами. На рисунке 3 показано, как расставить числа в таблице, чтобы получить 6 простых сумм (в нашем примере все суммы по 2 числа равны 11, и 1 + 2 + 3 + 7 + 6 = 19). Замечание. За пример без оценки — 3 балла. Рис. 3 1 10 2 9 3 8 7 4 6 5 42 XXXII Всероссийская математическая олимпиада школьников
г 7 класс 7.1. См. решение задачи 6.2. 7.2. Ответ. 1003. Заметим, что два воина, стоящие рядом, не могли оказаться рыцарями. Действительно, если бы они оба были рыцарями, то они оба сказали неправду. Выберем воина, стоящего слева, и разобьем ряд из оставшихся 2004 воинов на 1002 группы по два рядом стоящих воина. В каждой такой группе не более одного рыцаря. То есть среди рассматриваемых 2004 воинов не более 1002 рыцарей. То есть всего в шеренге не более 1002 + 1 = 1003 рыцарей. Рассмотрим шеренгу: РЛРЛР...РЛРЛР. В такой шеренге стоит ровно 1003 рыцаря. Замечание. Если дан только ответ, ставить 0 баллов, если приведен только пример, — 2 балла. 7.3. Ответ. Две гири. Одной гири продавцу не хватит, поскольку для взвешивания 25 кг сахара требуется гиря весом не менее 20 кг. Имея только такую гирю, продавец не сможет взвесить, например, 10 кг сахара. Покажем, что продавцу хватит двух гирь: одной весом 5 кг и одной весом 15 кг. Сахар весом от 0 до 5 кг можно взвесить без гирь. Чтобы взвесить от 5 до 10 кг сахара, нужно поставить гирю в 5 кг на правую чашку. Чтобы взвесить от 10 до 15 кг сахара, нужно поставить гирю в 5 кг на левую чашку, а гирю в 15 кг на правую чашку. Чтобы взвесить от 15 до 20 кг сахара, нужно поставить гирю в 15 кг на правую чашку. Чтобы взвесить от 20 до 25 кг сахара, нужно поставить гири в 5 кг и 15 кг на правую чашку. 7.4. Ответ. N = 1 021 324 354. Число N по крайней мере десятизначное, так как различных сумм 9. Поэтому наименьшее число десятизначное, при этом каждая из сумм 1, ..., 9 должна встречаться ровно один раз. Из двух десятизначных чисел, начинающихся с одинаковых цифр, то меньше, у которого первая различающаяся цифра меньше. Поэтому первая цифра числа N равна 1, вторая — 0. Сумма 1 уже встретилась, поэтому наименьшая третья цифра — 2 и т. д. 7-5. Рассмотрим произвольный квадратик 2 х 2. В нем не может быть клеток всех трех цветов, так как тогда II ЭТАП. 2005/2006 учебный год ( 43
можно было бы найти трехклеточный уголок, все клетки которого трех разных цветов. Также в этом квадратике 2x2 все клетки не могут быть одного цвета, так как тогда можно было бы найти трехклеточный уголок, все клетки которого одного цвета. Значит, в этом квадратике клетки только двух цветов. Заметим, что в этом квадратике клеток одного цвета не может быть 3, так как тогда можно было бы найти трехклеточный уголок, все клетки которого одного цвета. То есть в этом квадратике по 2 клетки двух разных цветов. Разобьем теперь таблицу 8x8 на 16 квадратиков 2 х 2. В каждом из них либо нет клеток первого цвета, либо две клетки первого цвета. То есть всего клеток первого цвета четное количество. Аналогично клеток второго и третьего цветов четное количество. ф 8 класс 8.1. Ответ. Могла. Рассмотрим, например, число А = 32 000...000 (на конце 1001 ноль). Тогда А2 = 1 024 000...000 (на конце 2002 ноля). Если стереть последние 2005 цифр, то останется число 1. 8.2. Ответ. 1003, 1002, 0. Из того, что наборы совпадают, следует равенство a + b + c = a-l + b+l + c2. Получаем с = с2. То есть с = 0 или с = 1. Так как с = с2, то а - 1 = Ь, Ь + 1 = а. Это означает, что возможны два случая: набор 6 + 1, 6, 0 и 6 + 1, 6, 1. Из того, что сумма чисел набора равна 2005, в первом случае получаем 26 + 1 = 2005, 6 = 1002 и набор 1003, 1002, 0, во втором случае получаем 26 + 2 = 2005, 6 = 1001, 5 — не целое число, т. е. второй случай невозможен. Замечание. Если дан только ответ, то ставить 0 баллов. 8.3. Ответ. 1336. Заметим, что оба воина, стоящих на одном краю шеренги, не могли оказаться лжецами. Действительно, если бы они оба были лжецами, то воин, стоящий с краю шеренги, сказал бы правду. Поэтому из двух самых левых и двух самых правых воинов суммарно не более двух лжецов. Рассмотрим ряд из оставшегося 2001 воина. 44 XXXII Всероссийская математическая олимпиада школьников
Заметим, что три подряд стоящих воина не могли оказаться лжецами, поскольку тогда средний из них сказал бы правду. Разобьем ряд из 2001 воина на 667 групп по три рядом стоящих воина. В каждой такой группе не более двух лжецов. То есть среди рассматриваемого 2001 воина не более 1334 лжецов. То есть всего в шеренге из 2005 воинов не более 1334 + 2 = 1336 лжецов. Рассмотрим шеренгу: ЛР (ЛЛР) (ЛЛР)...(ЛЛР) ЛР. В такой шеренге стоит ровно 1336 лжецов. 8.4. Ответ. 60°, 30°, 90°. В данной задаче приведено подробное решение. В дальнейшем подробные обоснования фактов, которые читатель может восстановить самостоятельно, опускаются. Прямая, проходящая через середины катетов, делит высоту СН пополам, поэтому искомая точка Р е MN9 где М и N — середины катета и гипотенузы (рис. 4), т. е. MN — средняя линия ДABC. В Н М Рис. 4 Тогда MN || ВС => ZP = ZBCH (как внутренние накрест лежащие углы при параллельных прямых) => => АВСН = ANPH (ZCHB = ZPHN = 90°, СН = РН — по стороне и острому углу) => ВН = NH => CN = СВ = а (в равнобедренном треугольнике высота является биссектрисой). Но CN — медиана прямоугольного треугольника ABC у поэтому CN = BN (ясно, если описать около треугольника ABC окружность) => ABCN — равносторонний, следовательно, ZB = 60°. II ЭТАП. 2005/2006 учебный год 45
8,5- Ответ. Не могло. Рассмотрим шахматную раскраску доски 10 х 10. Заметим, что из белой клетки своим ходом хромая ладья попадает в черную, а из черной клетки — в белую. Пусть ладья начала обход с белой клетки. Тогда 1 будет стоять в белой клетке, 2 — в черной, 3 — в белой, ..., 100 — в черной. То есть в белых клетках будут стоять нечетные числа, а в черных — четные. Но из двух соседних по стороне клеток одна — черная, а другая — белая. То есть сумма чисел, записанная в этих клетках, всегда будет нечетной, и не будет делиться на 4. Замечание. За «решения», в которых рассматривается только пример какого-то обхода, ставить 0 баллов. ▼• 9 класс 9.1. Ответ. Могли. Заметим, что среди 11 подряд идущих натуральных чисел есть два, делящиеся на 5, и есть два четных числа, поэтому их произведение оканчивается на два нуля. Заметим теперь, что а + (а + 1) + (а + 2) + ... + + (а + 10) = (а + 5) ■ 11. Если взять, например, а = 95 (т. е. Вася выбрал числа 95, 96, ..., 105), то сумма также будет заканчиваться на два нуля. 9.2. Ответ, а = Ъ = с = -1. Из того, что наборы совпадают, следует совпадение их сумм. Значит, а4 - 2Ь2 + Ь4 - 2с2 + с4 — 2а2 = = а + Ь + с = -3, (а2 - I)2 + (Ь2 - I)2 + (с2 - I)2 = 0. Откуда а2- 1 = Ь2- 1 = с2- 1 = 0, т.е. а = ±1, Ь = ±1, с = ±1. Условию а + Ъ + с = -3 удовлетворяют только а = Ъ = с = -1. Осталось проверить, что найденная тройка удовлетворяет условиям задачи. 9.3. Пусть £, .F, К, L, М, N — точки касания (рис. 5). Предположим, что DE = EF = FB = х. Тогда АК = = AL = a, BL = BE = 2x, BM = BF = х, CM = CN = c9 DK = DE = jc, DN = DF = 2x => AB + ВС = a + Зх + с = = AC, что противоречит неравенству треугольника. Замечание. Так же доказывается невозможность равенства BF = DE. Вообще, если для вписанной в треугольник ABD окружности Е — точка касания и BF = DE> то F — точка, в которой вневписанная окружность AABD касается BD. 46 XXXII Всероссийская математическая олимпиада школьников
м Рис. 5 А К D N С 9.4. Ответ. Верно. Предположим, что нельзя выбрать окружность, на которой есть точки всех трех цветов. Выберем точку А первого цвета и точку В второго цвета и проведем через них прямую /. Если вне прямой / найдется точка С третьего цвета, то на окружности, описанной около треугольника ABC, найдутся точки всех трех цветов (например, Л, Б и С). Значит, вне прямой / нет точек третьего цвета. Но раз хотя бы одна точка плоскости покрашена в третий цвет, то эта точка (назовем ее D) лежит на прямой I. Если теперь рассмотреть точки А и D, то аналогично можно показать, что вне прямой / нет точек второго цвета. Рассмотрев точки В и D, можно показать, что вне прямой / нет точек первого цвета. То есть вне прямой / нет покрашенных точек. Получили противоречие с условием. Значит, можно выбрать окружность, на которой есть точки всех трех цветов. 9.5. Выберем строки с номерами а19 а2, а3 и столбцы с номерами bl9 Ъ2у х. Сумма девяти чисел, стоящих на их пересечении, будет равна нулю. Выберем теперь строки с номерами а19 а2, а3 и столбцы с номерами Ьи Ь2> У- Сумма девяти чисел, стоящих на их пересечении, будет равна нулю. Отсюда следует, что сумма трех чисел, стоящих в столбце х в строках с номерами а19 а2, а3, равна сумме трех чисел, стоящих в столбце у в строках с номерами а19 а2, а3 (обозначим эту сумму S). Аналогично сумма трех чисел, стоящих в другом произвольном столбце z в строках с номерами а19 а2, а3, также равна S. Выберем строки с номерами а19 а2, а3 и столбцы с номерами х, у> г. Сумма девяти чисел, стоящих на их пересечении, будет II ЭТАП. 2005/2006 учебный год 47
равна нулю. Но она также равна 3S. То есть S = 0. В силу произвольности выбора чисел а19 а2, а3, ху у, г мы показали, что сумма любых трех чисел, стоящих в одном столбце, равна нулю. Рассмотрим теперь четыре произвольных числа ft, Z, m, ny стоящих в одном столбце. Для них k + l + m = k + l + n = 0 и k + l + m = k + m + n = 0. Из первого равенства получаем т = пу а из второго равенства получаем I = п, т. е. I = т = п. Теперь из I + т + п = 0 получаем / = т = п = 0. В силу произвольности выбора чисел мы доказали, что все числа в таблице равны нулю. Ф 10 класс 10,1- Из теоремы Виета получаем sin a cos а = - и sin а + cos а = —. Нам требуется доказать равенство а Ь2 = а2 + 2ас. Так как а Ф 0, то разделим это равенст- ( V во на а2. Нам нужно доказать равенство - = 1 + 2 -. \а) а ( V Получаем - = (sin a + cos а)2 = sin2 а + cos2 а + \а) + 2 sin а cos а = 1 + 2 sin а cos а = 1 + 2 -, что и тре- а бовалось. 10-2- См. решение задачи 9.3. 10-3. Ответ- а = Ъ = 2. Пусть НОД (a; b) = d. Тогда а = axd, Ъ = bxdy где НОД (аг; bi) = 1. Тогда НОК (а; Ъ) = aj)xd. Отсюда a^)xd + d — axdbxdi или alb1 + 1 = axbxd. Откуда a1b1 (d - 1) = 1. To есть a1 = b1 = l и d = 2, значит, a = b = 2- Замечание. Другое решение можно было получить, воспользовавшись равенством НОК (а; Ь) - НОД (а; Ъ) = аЪ. Замечание. Если дан только ответ, то ставить 0 баллов. 10-4- Пусть ВР — высота равнобедренного треугольника FBE (рис. 6). Тогда из подобия ААМЕ ~ АВРЕ следует, что ^§ = jj§- Аналогично Щ = Ц. Далее, = ^f9 AE = AK9 CF = СК, где К — точка ка- сания окружности со стороной АС. С учетом равенст- 48 XXXII Всероссийская математическая олимпиада школьников
N Рис. 6 nri nr, AB + ВС - (AE + CF) 2AC -(AK + CK) ва BE =BF = - = - AC „ r оол, 2ME ■ 2NF 2AK.2CK = — получаем + = + = = 2. Утверждение доказано. 10.5. Ответ. Не мог. Рассмотрим число N + 142 857 - 7N = N + 999 999ЛГ = = 1 000 OOOiV. Его сумма цифр равна сумме цифр числа N. Значит, Петя не мог каждый раз получать результат больше предыдущего. ▼ 11 класс 11.1. Например, подойдет функция / (х) = х2. Действительно, / (sin а) + / (cos а) = sin2 а + cos2 а = 1, 4/ (sin а cos а) = = 4 sin2 а cos2 а = (2 sin а cos а)2 = sin2 (2а), а неравенство 1 ^ sin2 (2a) выполняется для любого а. 11.2. Ответ. |а3. Каждый из таких треугольников равносторонний, так как его стороны равны половинам диагоналей граней куба. Тогда вершины рассматриваемого многогранника — центры этих треугольников (рис. 7 на с. 50). При повороте куба вокруг диагонали АХС, переводящем куб в себя (например, А —► Вх —► Dx —► А, Сх —► D -* В —► Сх), вершины сечения также меняются местами (К —> L —> М —► К), поэтому центр Т треугольника KLM лежит на диагонали АХС, а плоскость KLM перпендикулярна АХС. Тогда из подобия II ЭТАП. 2005/2006 учебный год 49
L / / 1 К А Рис- 7 К А Рис. 8 т. е. 2V3 (рис. 8) находим •ОГ = |ОА1. ТАХ _ АХА V3' Значит, Ах переходит в Т при гомотетии с центром О о и коэффициентом k = -. Аналогично получаем и ос- о тальные вершины многогранника. Значит, он является кубом, гомотетичным исходному, а его объем — 11.3. Прямая у = kx + b касается параболы у = х29 если уравнение jc2 = ftjc + b имеет единственное решение, т. е. D = к2 + 4Ь = 0, откуда 4bt = -kt2, i = 1, 2, 3. Тогда 3 § f | f | = 8&! + 8b2, так как k\ + k2^ 2кхк2. 11.4. Ответ. Не мог. Рассмотрим число N + 76 923 - 13iV = N + 999 999iV = = 1 000 OOOiV. Его сумма цифр равна сумме цифр числа N. Значит, Петя не мог каждый раз получать результат больше предыдущего. 11.5. Ответ. Нельзя. Предположим, что мы смогли нарисовать на плоскости 2005 ненулевых векторов так, что из любых 10 из них можно выбрать 3 с нулевой суммой. Выберем прямую, которая не перпендикулярна ни одному из нарисованных 2005 векторов. Рассмотрим проекции 50 XXXII Всероссийская математическая олимпиада школьников
нарисованных векторов на выбранную прямую. Ни одна из этих проекций не равна нулевому вектору. При этом по крайней мере 1003 проекции направлены в одну сторону. Выберем 10 векторов, проекции которых направлены в одну сторону. Тогда проекции любых трех из них направлены в одну сторону, и поэтому их сумма не равна нулю. (Так как если сумма векторов равна нулю, то и сумма проекций этих векторов на прямую также равна нулю.) Ill ЭТАП (региональный) УСЛОВИЯ ЗАДАЧ ▼ 8 класс 8.1. В государстве каждый житель либо рыцарь, либо лжец. Рыцари всегда говорят правду, а лжецы всегда лгут. Все жители знакомы друг с другом. Президент однажды сделал два утверждения — «Я знаком с четным числом рыцарей» и «Я знаком с нечетным числом лжецов». Докажите, что любой другой житель сделает такие же утверждения. (Президент входит в число жителей.) (И. Богданов) 8.2. В произведении ДО - РЕ - МИ - СИ одинаковые буквы обозначают одинаковые цифры, а разные буквы — разные цифры. Каким наибольшим количеством нулей может заканчиваться это произведение? (И. Рубанов) 8.3. Назовем диагональ пятиугольника хорошей, если какие-то другие диагонали делят ее на 3 равные части. Какое наибольшее число хороших диагоналей может быть у выпуклого пятиугольника? (Н.Рубанов) 8.4. В приборе имеется п ^ 4 контактов и т ^ 4 проводов, причем каждый провод соединяет ровно два контакта. Известно, что для любых четырех проводов найдутся такие два контакта, что любой из этих проводов подсоединен хотя бы к одному из них. Докажите, что найдутся такие три контакта, что любой провод в приборе подсоединен хотя бы к одному из них. (В. Дольников) III ЭТАП. 2005/2006 учебный год 51
AC 8.5. В написанном на доске выражении ——— Петя и Коля В + С заменяют буквы тремя различными натуральными числами: вначале Петя заменяет букву А, затем Коля — букву Б, затем опять Петя — букву С. Докажите, что Петя может писать числа так, чтобы значение записанного на доске выражения оказалось целым. (И.Рубанов, Б. Сендеров) 8.6. Клетки прямоугольника 7x8 покрашены в три цвета, причем в любом квадратике 2x2 есть клетки всех трех цветов. Какое наибольшее количество клеток может быть покрашено в первый цвет? (О.Под- липский) 8.7. Каждый день Малыш и Карлсон едят пирожные. В первый день они съели по одному пирожному. Затем Малыш каждый день съедает ровно одно пирожное, а Карлсон — ровно столько, сколько они съели вместе за все предыдущие дни. Могло ли число пирожных, съеденных однажды Карлсоном, оканчиваться на 101? (Н. Агаханов) 8.8. На разных сторонах угла с вершиной S выбраны точки Р и Q (SP ^ SQ). Через середину М отрезка PQ проведена прямая, перпендикулярная биссектрисе угла. Эта прямая пересекается с прямой SP в точке Т. Докажите, что перпендикуляр к SP, восставленный в точке Г, и перпендикуляр к PQ, восставленный в точке М, пересекаются на биссектрисе угла. (Л. Емельянов) ф 9 класс 9.1. Клетки доски 8x8 раскрашены в шахматном порядке. Одним ходом разрешается перекрасить любую клетку в цвет одной из соседних с ней клеток. Можно ли с помощью таких перекрашиваний изменить цвет всех клеток на противоположный? (Соседними считаются клетки, имеющие общую сторону.) (Н. Агаханов) 9.2. Найдите все натуральные п9 удовлетворяющие равенству /г! = 1...71 (в правой части последовательно выписаны друг за другом десятичные записи всех натуральных чисел от 1 до л; в левой части п\ = 1 • 2 - ...- п). (В. Сендеров) 52 XXXII Всероссийская математическая олимпиада школьников
g.3. Из точки А, расположенной вне окружности со, проведены две касательные АВ и АС к этой окружности. Точка D лежит на со и диаметрально противоположна С. Перпендикуляр, опущенный из точки В на прямую CD, пересекает ее в точке Н. Докажите, что прямая AD делит отрезок ВН пополам. (В. Астахов) 9.4. На острове живут 2006 человек, каждый — либо рыцарь, либо лжец. Рыцари всегда говорят правду, а лжецы всегда лгут. Некоторые из жителей знакомы (если А знаком с В, то и В знаком с А). Каждый житель острова, кроме президента, сделал два утверждения — «Я знаю четное число рыцарей» и «Я знаю нечетное число лжецов». Докажите, что президент должен сделать такие же утверждения. (Президент входит в число жителей острова.) (И.Богданов) 9.5. Каждый день Малыш и Карлсон едят пирожные. В первый день они съели по одному пирожному. Затем Малыш каждый день съедает ровно одно пирожное, а Карлсон — ровно столько, сколько они съели вместе за все предыдущие дни. Могло ли число пирожных, съеденных однажды Карлсоном, оканчиваться на 101? (Н.Агаханов) 9.6. На доске написаны многочлены х + 1ихЧ1. Разрешается дописывать на доску многочлен /, равный сумме, разности или произведению любых двух различных из написанных многочленов, если многочлен / не был выписан на доске ранее. Можно ли выписать на доску многочлен л:2006 + 1? (Н.Агаханов) 9.7. В выпуклом четырехугольнике ABCD описанная окружность треугольника ABC пересекает стороны CD и DA в точках Р и Q, а описанная окружность треугольника CDA пересекает стороны АВ и ВС в точках R и S соответственно. Прямые ВР и BQ пересекают отрезок RS в точках М и N. Докажите, что точки М, N9 P, Q лежат на одной окружности. (С. Берлов) 9.8. Есть 15 монет, среди которых четное (не известное нам) число фальшивых. Все настоящие монеты весят одинаково, все фальшивые тоже весят одинаково, но они легче настоящих. Можно ли за 3 взвешивания на чашечных весах найти хотя бы одну настоящую монету? (С. Токарев) III ЭТАП. 2005/2006 учебный год 53
0 10 класс 10.1. Клетки доски 8x8 раскрашены в шахматном порядке. Одним ходом разрешается перекрасить любую клетку в цвет одной из соседних с ней клеток. Можно ли с помощью таких перекрашиваний изменить цвет всех клеток на противоположный? (Соседними считаются клетки, имеющие общую сторону.) (Н.Агаха- нов) 10.2. Числа от 1 до 100 выписали в строку в некотором порядке. Докажите, что найдутся два рядом стоящих числа, сумма которых больше 50, но меньше 150. (С. Берлов) 10.3. Пусть а и Ъ — различные натуральные числа, большие 1 000 000, и такие, что (а + Ь)3 делится на ab. Докажите, что | а - Ъ \ > 10 000. (И. Богданов, В. Сен- деров) 10.4. Продолжения сторон АВ и CD выпуклого четырехугольника ABCD пересекаются в точке Р, а продолжения сторон ВС и AD — в точке Q. Обозначим через М и N середины диагоналей АС и BD. Докажите, что если /.АРМ = /CPN, то ZBQN = ZDQM. (Л.Емельянов) 10.5. Уравнение 2х3 + ах2 + Ъх + с = 0 с целыми коэффициентами имеет три различных корня. Оказалось, что первый корень является синусом, второй — косинусом, а третий — тангенсом одного угла. Найдите все такие уравнения. (Н. Агаханов, И. Богданов) 10.6. На плоскости провели 8 прямых, никакие две из которых не параллельны. Какое наибольшее число равнобедренных треугольников со сторонами, лежащими на этих прямых, могло образоваться? (И.Рубанов) 10.7. Дана трапеция ABCD с основаниями AD и ВС. Окружность сох касается основания ВС в точке М и продолжений сторон АВ и CD за точки В к С; окружность со2 касается основания AD в точке N и продолжений сторон АВ и CD за точки А и D. Докажите, что отрезок MN проходит через точку пересечения диагоналей трапеции. (Л. Емельянов) 10.8. В стране есть несколько городов, соединенных дорогами. Каждая дорога соединяет только два города, и на ней введено одностороннее движение; при этом пара городов соединена не более чем одной дорогой. 54 XXXII Всероссийская математическая олимпиада школьников
Выехав из любого города, нельзя в него вернуться. Известно, что из города А в город В можно проехать ровно 2006 способами. Найдите минимальное возможное число городов в стране. (И.Богданов) т 11 класс 11.1. График линейной функции касается графика квадратичной функции у = f (x), а график квадрата этой линейной функции получается из графика функции у — f (x) сдвигом вниз на величину р. Докажите, что для всех таких / (х) число р одинаково. (Н. Агаха- нов) 11.2. Числа от 1 до 100 выписали в строку в некотором порядке. Докажите, что найдутся два рядом стоящих числа, сумма которых больше 50, но меньше 150. (С. Вер лов) 11.3. Положительные числа хну таковы, что для некоторого натурального п справедливо равенство х + х2 + ... + хп + у + ... + уп = 2п. Докажите, что тогда выполняется неравенство 1 + ^2. х у (В. Астахов, А. Гаврилюк) 11.4. Описанная окружность четырехугольника ABCD отражается симметрично относительно сторон АВ и AD. Построенные окружности вторично пересекаются в точке А', отличной от А. Аналогично строятся точки В', С и D'. Докажите, что четырехугольники ABCD и A'B'C'D' равны. (Л.Емельянов) 11-5. Числа sin х, cos x, tg x являются членами некоторой бесконечной в обе стороны геометрической прогрессии (..., Ь_2, Ь_19 b0, bl9 b2, ...)• Докажите, что ctg x также входит в эту прогрессию. (Н.Агаханов) 11.6. Основания трех высот треугольной пирамиды являются точками пересечения медиан граней, к которым они проведены. Докажите, что все ребра пирамиды равны. (Н. Агаханов) 11-7. На плоскости проведено 12 прямых, никакие две из которых не параллельны. Какое наибольшее число равнобедренных треугольников со сторонами, лежащими на этих прямых, могло образоваться? (И.Рубанов) III ЭТАП. 2005/2006 учебный год 55
11-8- Имеется куча из N (N > 1) камней. Двое играют в игру. За один ход можно либо разделить любую имеющуюся кучу на две произвольным образом (если в куче более одного камня), либо забрать один камень из любой кучки. Побеждает тот, кто заберет последний камень. Кто из соперников сможет победить независимо от игры соперника? (Д.Храмцов) РЕШЕНИЯ ЗАДАЧ Ф 8 класс 8-1- Заметим, что все рыцари знакомы с одним и тем же числом рыцарей и одним и тем же числом лжецов; аналогично для лжецов. Значит, достаточно доказать, что если рыцарь сделал такие два утверждения, то и лжец сделает такие же (и наоборот). Но количество лжецов, с которыми знакомы рыцарь и лжец, различаются на 1, поэтому утверждение «Я знаком с нечетным числом лжецов» могут сделать либо оба, либо ни один. Аналогично с другим утверждением. 8.2. Ответ. 5 нулей. Двузначные числа в произведении П = ДО • РЕ • МИ • СИ оканчиваются на три разных цифры. По крайней мере одна из них не 0 и не 5, поэтому хотя бы один из сомножителей не делится на 5. Каждое из трех остальных чисел делится не более чем на вторую степень пятерки, причем на 52 оно может делиться только в том случае, если оно 25, 50 или 75. Поэтому П делится не более чем на шестую степень пятерки, причем шестая степень может оказаться только в том случае, если наши три числа — это 25, 50 и 75. Но они все содержат в своей записи цифру 5, а у нас каждая цифра встречается не более двух раз. Поэтому произведение может делиться максимум на пятую степень пятерки и, как следствие, не более чем на пятую степень десятки (соответственно заканчиваться не более чем пятью нулями). В то же время, например, произведение П = 30 - 64 - 25 - 75 = 3 600 000 оканчивается на 5 нулей. 8.3. Ответ. Две хороших диагонали. Если у пятиугольника не менее 3 хороших диагона- 56 XXXII Всероссийская математическая олимпиада школьников
А Рис. 9 Е Рис. 10 лей, то какие-то две из них пересекаются. Пусть это диагонали АС и BE. Тогда в четырехугольнике ABMN (рис. 9) диагонали делятся точкой пересечения О пополам. Значит, ABMN — параллелограмм. Но тогда ВМ || AN, т. е. BD || AD — противоречие. На рисунке 10 приведен пример пятиугольника с двумя хорошими диагоналями. Он может быть получен из произвольного треугольника АСЕ делением боковых сторон на три равные части (точки М, N, К, L) и построением вершин B = EM0NK и D = AL0 NK. 8.4. Можно считать, что любая пара контактов соединена не более чем одним проводом, и проводов не меньше четырех. Если найдутся два провода АВ и XY, не имеющие общих концов, то любой третий провод должен иметь общий конец либо с АВ, либо с XY. Значит, в любом случае нашлись два провода, имеющие общий конец (скажем, АВ и АС). Если любой провод имеет концом один из контактов А, В, С, то А, В, С — требуемая тройка контактов. Иначе имеется провод DE, где контакты D и Е отличны от А, В, С. Добавим к АВ, AC, DE четвертый провод XY. Для них найдутся такие два контакта, что любой из этих четырех проводов подсоединен хотя бы к одному из них. Один из контактов — D или Е, значит, другой — А. Таким образом, любой провод подсоединен к одному из контактов A, D, Е, что и требовалось. 8-5. Вначале Пете достаточно взять произвольное нечетное А > 2, а затем, какое бы В ни написал Коля, положить С = АВ - В. Очевидно, С > 0. Число на доске АС АВ(А-1) окажется целым: В + С АВ = А - 1. Осталось III ЭТАП. 2005/2006 учебный год 57
доказать, что С Ф А, С Ф В. Поскольку А Ф 2, то С Ф В. Далее, С = (А - 1) В четно, следовательно, СфА. Замечание. Неравенство С Ф А легко доказать и при произвольном А, большем 2. 8.6. Ответ. 32 клетки. Пример показан на рисунке 11. Докажем теперь, что более 32 клеток не может быть покрашено в первый цвет. 1 2 1 2 1 3 1 3 1 2 1 2 1 3 1 3 1 2 1 2 1 3 1 3 1 2 1 2 1 3 1 3 ... Рис. 11 Рис. 12 Заметим, что в любом квадратике 2 х 2 не более двух клеток первого цвета. Разобьем прямоугольник 7x8 на 12 квадратиков 2 х 2 и один прямоугольник 1x8 (рис. 12). В каждом квадрате не более двух клеток первого цвета, и в прямоугольнике не более 8 клеток первого цвета. Итого клеток первого цвета не более 12-2 + 8 = 32. 8.7. Ответ. Не могло. Если в /1-й (п ^ 2) день Карлсон съел ап пирожных, то в (п + 1)-й день он съел ап + г = ап + 1 + ап = 2ап + 1 пирожное. Тогда ах = 1, а2 = 2, а3 = 5, а4 = 11, т. е. на четвертый день он съел 11 пирожных. Но 11 = 4й - 1, и на следующий день Карлсон съест 2 (4k -1) + 1 = 8й-1 = 4- 2й-1 пирожное. Следовательно, начиная с четвертого дня, количество пирожных, съеденных Карлсоном, дает остаток 3 при делении на 4. А число, оканчивающееся на 101, дает остаток 1 при делении на 4. 8.8. Отложим на луче SP отрезок SQ' = SQ (рис. 13). В равнобедренном треугольнике QSQ' биссектриса I угла PSQ является высотой и серединным перпендикуляром к отрезку QQ'. Отсюда следует, что прямые QQ' и МТ параллельны, так как они перпендику- 58 XXXII Всероссийская математическая олимпиада школьников
Рис. 13 лярны Z. Так как МТ проходит через середину PQ, то МТ — средняя линия треугольника QPQ', и Г — середина PQ'. Пусть К — точка пересечения перпендикуляра к PQ, восставленного в М (т. е. серединного перпендикуляра к PQ), и перпендикуляра к SP, восставленного в Т (т. е. серединного перпендикуляра к PQ'). Точка К равноудалена от Р и Q, а также от Р и Q'. Значит, она равноудалена от Q и Q'. Следовательно, К лежит на серединном перпендикуляре I к отрезку QQ'. ▼ 9 класс 9.1. Ответ. Нельзя. Пусть мы сможем поменять цвет всех клеток. Рассмотрим клетку А, которую мы перекрашиваем последней. К моменту перекрашивания клетки А все остальные клетки должны были изменить свой цвет на противоположный. Значит, в этот момент все соседние с А клетки одного с ней цвета, и А перекрасить нельзя. 9.2. Ответ, п = 1. Первое решение. Докажем, что при п > 1 Т^п>п\. (1) Для этого вначале докажем, что при любых а, Ъ е N ab> а - Ь. (2) Имеем аЪ> а - 10л, если Ъ - /г-значное число. Отсюда а - 10л > а • Ь. Неравенство (2) доказано, а неравенство (1) сразу получается из неравенства (2) по индукции. Второе решение. Заметим, что число l.../г имеет на конце столько же нулей, как и число /г, а число /г! при п > 5 — большее количество нулей, так как при п > 5 в произведении /г! = 1 • 2 - ... - п встречаются, кроме /г, еще 2 и 5. Случаи п < 5 легко разбираются по отдельности. ЭТАП. 2005/2006 учебный год 59
Рис. 14 9.3. Так как CD — диаметр, проведенный в точку касания окружности с прямой АС у то CD J_ AC и, следовательно, ВН || АС (рис. 14). Продлим DB до пересечения с прямой АС в точке Е. Углы CBD и С BE прямые, поскольку CD — диаметр. Из равенства касательных АС и АВ следует, что точка А лежит на серединном перпендикуляре к катету ВС прямоугольного треугольника ЕВС. Значит, она лежит на средней линии этого треугольника, поэтому точка А — середина стороны ЕС. Следовательно, прямая AD делит отрезок СЕ пополам. Тогда она делит пополам и параллельный ему отрезок ВН. 9.4. Для решения задачи докажем лемму. Лемма. Пусть в некоторой компании каждый человек из этой компании записал, сколько у него в компании знакомых. Тогда количество записанных нечетных чисел четно. Доказательство. Рассмотрим количество (упорядоченных) пар (А, Б) знакомых в этой компании. Оно четно, так как вместе с парой (А, В) присутствует и пара (В, А). С другой стороны, оно равно сумме всех записанных чисел, так как количество пар вида (А, х) равно числу, записанному А. ■ Рассмотрим любого человека, отличного от президента. Если он лжец, то у него четное число знакомых лжецов и нечетное число знакомых рыцарей. Если он рыцарь, то наоборот — нечетное число знакомых лжецов и четное число знакомых рыцарей. В любом случае у него нечетное число знакомых. Тогда по лемме и у президента также нечетное число знакомых. Пусть президент — рыцарь. Применим к компании из всех рыцарей лемму. Так как у каждого рыцаря, 60 XXXII Всероссийская математическая олимпиада школьников
отличного от президента, было четное число знакомых рыцарей, получаем, что у президента тоже четное число знакомых рыцарей, а значит, нечетное число знакомых лжецов, и утверждение задачи доказано. Если же президент лжец, достаточно аналогичным образом применить лемму ко множеству всех лжецов. 9.5. См. решение задачи 8.7. 9.6. Ответ. Можно. Покажем по индукции, как получить любой многочлен вида хп + 1. База индукции: хп + 1 для п = 1 и п = 2 заданы изначально. Пусть на доске есть многочлены fn_2 = xn~2+l и /n_1 = xn"1 + l. Тогда выпишем gn = fn-l ~ fn-2 = X"-1 - ХП~2, hn = (x+l).gn = X»-2X х (х - 1) (х + 1) = хп - хп -2, hn + fn _ 2 = хп - хп - 2 + + хп~2 + 1 = хп + 1. (Если в какой-то момент мы хотим выписать многочлен, уже написанный на доске, то пропустим этот шаг.) Утверждение доказано. 9.7. Используя то, что четырехугольники ABCQ, BCPQ и ARSC вписанные (рис. 15), получаем равенства: ZNQP = ZBQC + ZCQP = ZBAC + Z.CBP = (180° - ZBSC) + + ZCBP = ZBSR + ZCBP = ZBSM + ZSBM = ZSMP = = 180° - ZNMP. Итак, ZNQP + ZNMP = 180°, откуда следует требуемое. В Рис. 15 9-8. Ответ. Можно. Укажем способ отыскания настоящей монеты. Для первого взвешивания положим на чашки весов по 4 монеты. Возможны два случая. 1- Одна из чашек перевесила. Обозначим через А, Б, С и D массы монет на этой чашке; ясно, что хотя бы III ЭТАП. 2005/2006 учебный год 61
одна из этих монет настоящая. Вторым взвешиванием сравниваем величины А + В и С + D. Если А + В > С + D или А + В = С + D, то монеты с массами А и Б не могут обе быть фальшивыми, и тогда третьим взвешиванием сравниваем А и В. Более тяжелая из монет обязательно настоящая, а при А = В настоящие обе. (Если же А + В < С + D, то третьим взвешиванием сравним С и D.) 2. При первом взвешивании зафиксировано равенство масс. Это значит, что на чашках по одинаковому числу фальшивых монет, а общее число взвешенных фальшивых монет четно. Следовательно, среди остальных 7 монет число фальшивых также четно. Для второго взвешивания положим на чашки по 2 монеты из ранее не взвешенных. Если какая-то пара тяжелее, то третьим взвешиванием сравним монеты этой пары; монета, которая тяжелее или равна другой, настоящая. Если же массы пар монет во втором взвешивании равны, то в этих парах по одинаковому числу фальшивых монет, общее число фальшивых среди взвешенных четно, четно оно и среди 3 оставшихся (ни разу не взвешенных). Тогда возьмем любые 2 из этих 3 монет и сравним их массы; если какая-то чашка перевесит, то монета на ней настоящая, в случае же равенства настоящей обязательно будет третья монета. ▼ 10 класс 10.1. Ответ. Нельзя. См. решение задачи 9.1. 10.2. Пусть нашлась такая перестановка аи а2, ..., а100 чисел 1, 2, ..., 100, что каждая из сумм ах + а2, а2 + а3, ..., а99 + а100 либо не больше 50 (в этом случае назовем сумму маленькой), либо не меньше 150 (в этом случае назовем сумму большой). Тогда среди этих сумм найдется как маленькая (например, содержащая число 1), так и большая (например, содержащая число 100). Значит, найдутся две соседние суммы ai_1 + ai9 ai + ai + i9 одна из которых маленькая, а другая — большая. Модуль разности между этими суммами должен быть не меньше 150 - 50 = 100. С другой стороны, \(ai_1 + ai)-(ai + ai + 1)\ = \ai_1-al + 1\< < 100 - 1 = 99. Противоречие. 62 XXXII Всероссийская математическая олимпиада школьников
Ю-3. Из условия следует, что а3 + Ь3 = (а + Ь)3 — ЗаЬ (а + Ь) делится на ab. Докажем, что каждое из чисел а3 и Ь3 делится на аЪ. Пусть р — простой делитель числа ab. Без ограничения общности будем считать, что число а делится на ра и не делится на ра + *, число Ъ делится на /?р и не делится на /?р + *, где а ^ р ^ 0. Тогда а& делится на /?а + Р и не делится на/?a + p + 1. Число а3 делится на р3а, а значит, и на/?а + р; отсюда &3 = (а3 + &3) - а3 также делится на /?а + р. Значит, и а3, и &3 делятся на /?а + р. В силу произвольности выбора простого р, получаем, что а3 и &3 делятся на аЪ. Тогда (а — Ь)3 — а3 — Ъ3 — ЗаЬ (а - Ь) делится на ab. Значит, | а - Ъ |3 > ab > 106 • 106 = 1012. Получаем \a-b\> 104, что и требовалось. Замечание. Пример чисел, удовлетворяющих условию задачи: а = 101 ■ ЮЗ2, Ь = 1012 • 103. 10.4. Первое решение. Так как РМ— медиана ЛАРС, то = S MPC ±АР- РМ sin ZAPM = J МР ■ СР sin ZCPM (рис. 16). Следовательно, Аналогично дует, что sin sin из того, sin sin ZCPM что PN ZBPN ZDPN CP AP' — медиана DP BP' ABPD, еле- Рис. 16 D A = D III ЭТАП. 2005/2006 учебный год 63
Так как ZBPM = ZCPN, то ZCPM = ZBPN, значитЭ5 СР = ВР АР DP9 т. е. PC PD = PB • PA. Значит, четырехугольник ABCD вписанный. Тогда QB • QC = QA- QD, откуда ^ = ^-. Анало- гично, рассматривая медианы QM и QiV в треугольниках ACQ и BDQ, приходим к выводу, что sin ZBQN _ sin ZAQM sin ZNQD ~ sin ZMQB ' кроме того, ZBQN + ZNQD = ZAQM + ZMQS = ZAQB (обозначим этот угол (р). Тогда ZCQN = ZAQM, так ... sin (ф — jc) sin ф cos jc - sin jc cos ф как функция f (x) = : = : = ^J sin x sin x = sin ф ctg jc - cos ф строго монотонна на (0, ф). Второе решение. Отразив треугольник BPD симметрично относительно биссектрисы I угла BPD, получим треугольник B'PD' (см. рис. 16). Медиана PN' треугольника B'PD' симметрична PN относительно 19 поэтому N' лежит на луче РМ. Рассмотрим гомотетию с центром Р, переводящую N' в М. Точки В' и D' перейдут при этой гомотетии соответственно в точки Б" и D"j лежащие на лучах PC и РА. Если В" не совпадает с С, a D" не совпадает с А, то в четырехугольнике AD"CB" диагонали АС и B"D" делятся точкой М пополам, откуда следует, что AD"CB" — параллелограмм, и АВ || CD — противоречие. Следовательно, В" = С и D" = А, значит, АСРА равен AB"PD" и подобен треугольнику BPD. Из подобия следует, что ZBAC = ZBDC9 поэтому четырехугольник ABCD вписанный. Тогда ZQBD = ZQAC, значит, AQBD ~ AQAC. Углы BQN и AQM равны как соответственные углы (между медианой и стороной) в подобных треугольниках. 10.5. Ответ. 2х3 + 2х2 - х - 1 = 0. Пусть sin a, cos а, tg а — корни уравнения. Тогда 2х3 + ах2 + Ьх + с = 2 (х - sin а) (х - cos а) (х - tg а) = = 2х3 - 2 (sin а + cos а + tg а) х2 + 2 (sin а cos а + + sin а tg а + cos а tg а) х - 2 sin а cos а tg а. Отсюда 64 XXXII Всероссийская математическая олимпиада школьников
sin a • cos a • tg a = - £, т. e. sin2 a = - £, откуда, учи- Z Z тывая, что с — целое число, sin2 a = 0, - или 1. Но Z если sin a = 0, то и tg a = О, и два корня совпадают, а по условию уравнение имеет три различных корня. А если sin2 a =1, то tg a не существует. Значит, sin a = i-7=» тогда cos a = + —f= (так как корни различ- v2 v2 ны) и tg a = -1. Также -%- sin a + cos a + tg a = -1, \ = sin a • cos a + sin a • tg a + cos a • tg a = -. z z 10.6. Ответ. 24. Выделим в каждом равнобедренном треугольнике прямую, на которой лежит его основание. Если он равносторонний, произвольным образом объявим его основанием одну из его сторон. Пусть I и т — некоторые две из проведенных прямых. Тогда существует не более одного равнобедренного треугольника, у которого на I лежит основание, а на т — боковая сторона. Действительно, направление третьей стороны тогда находится однозначно, и существует не более одной проведенной прямой этого направления. Тогда прямая I содержит основания не более чем трех треугольников; иначе у треугольников с основаниями, лежащими на Z, было бы как минимум (если этих треугольников четыре) 8 боковых сторон, и две лежали бы на одной прямой, что невозможно, так как по условию никакие две прямые не параллельны. Итого, имеется не более 3 • 8 = 24 оснований, т. е. треугольников не больше 24. Равнобедренных треугольников будет 24, если провести 8 прямых, параллельных восьми последовательным сторонам правильного 16-угольника, так, чтобы никакие три прямые не пересеклись в одной точке. Легко видеть, что тогда на каждой прямой будет лежать ровно по три основания. 10-7. Пусть диагонали трапеции пересекаются в точке S, а отрезки BD и MN — в точке S'. Из подобия тре- угольников ASD и CSB следует —— = ——. Из подо- бия треугольников NS'D и MS'В следует —— = -гттг. Ill ЭТАП. 2005/2006 учебный год 65
Рис. 17 Чтобы точки S и S' совпадали, достаточно доказать равенство —=- = -—- или —— = ——. Сделаем это. Пусть прямые АВ и CD пересекаются в точке О и пусть О лежит для определенности на продолжениях отрезков АВ и CD за точки В и С соответственно. Обозначим через К, L, Р, Q точки касания окружностей сох, со2 с прямыми АВ и CD (рис. 17). Так как ОК + СМ = OL + CL = ОС и ВМ = ВК9 то Так как ОР = О А + AN, OQ = OD + DN и OP = OQ, ^ OA+AD + OD то OQ = и Треугольники О ВС и OAD подобны, поэтому ОВ ВС ОС u /Qx ОА " ~AD = OD ~ k- (3) ^ /о. ATI) OA+AD-OD Отсюда из равенства (2) получим —— = —— . Домножим и поделим на k числитель и знаменатель. Получим k - О А + k - AD - k-OD _ OB + ВС -ОС _ ВМ 2k • AD (из равенства (3)) 2ВС (из равенства (1))ВС что и требовалось. 10.8. Ответ. 13. Для каждого возможного способа проезда запишем множество городов (отличных от А и В), через которые путь проходит. Тогда для разных способов получаются разные множества. 66 XXXII Всероссийская математическая олимпиада школьников
Действительно, пусть для двух способов эти множества совпали. Поскольку каждые два города соединены не более чем одной дорогой, то порядок, в котором эти города встречаются на пути, разный. Следовательно, найдутся такие два города X и У, что при первом способе X встречается раньше, чем У, а при втором способе наоборот. Но тогда из X можно добраться до У и затем вернуться в X, что противоречит условию. Пусть в стране п городов. Тогда способов не больше, чем подмножеств множества из (п — 2) городов, отличных от А и Б, т. е. 2006 ^ 2Л~2, откуда п ^ 13. Приведем пример страны с 13 городами, в которой условие выполнено. Перенумеруем города от 0 до 12 (А — нулевой, В — двенадцатый) и соединим пока любые два города в направлении от меньшего к большему. Тогда для каждого подмножества промежуточных городов существует путь по ним, т. е. всего способов 211 = 2048. Теперь закроем дороги, ведущие из второго, четвертого и шестого городов в Б. Тогда из общего количества способов вычлось количество способов добраться из А во второй, из А в четвертый и из А в шестой города, т. е. количество способов стало равно 2048 - 21 - 23 - 25 = 2006, что и требовалось. ▼ 11 класс 11-1. Пусть у = I (х) — данная линейная функция. Тогда из условия / (х) = I2 (х) + р. Условие касания означает, что уравнение I2 (х) + р = I (х) имеет единственное решение х = х0. Но линейная функция, отличная от постоянной, каждое свое значение принимает ровно один раз. Значит, уравнение I2 + р = I должно иметь единственное решение, I = 10, т. е. D = 1 - 4р = 0, откуда р = ^. 11-2. См. решение задачи 10.2. 11-3. Докажем сначала, что х + у ^ 2. Предположим противное, т. е. х + у > 2. Поскольку для любого натурального k (i, ЭТАП. 2005/2006 учебный год 67
то xk + yk > 2 и, следовательно, х + х2 + ... + хп + у + ... + уп = = (х + у) + (х2 + у2) + ... + (хп + j/n) > 2п, что противоречит условию. __ а + Ь 2 Подставив в неравенство —-— ^ -—- вместо а и Ь a b 11 х у 2 11 числа — и -, получаем —-— ^ ^ 1, т. е. — + - ^ 2. Замечание. Неравенство (1) можно доказать так: пусть х + у = 2, тогда х=1-а, i/=l+a=>:x;* + i/* = = (1 - а)* + (1 + а)* = (1 - ka + k {k~ l) a2 - ... j + + [l + fea + fe(fe2~1) a2 +... J=2 + C1a2 + C2a4 + ..., где Cl9 C2, ..., Ck > 0. Значит, если х + у = 2, то jc* + i/* ^ 2. Если же л: + у > 2, то можно записать х = jcxd, j/ = yxdj где jcx + уг = 2, d > 1. Поэтому xk + yk = dk (x% + yf) ^ ^ 2dfe > 2. 11.4. Докажем, что AB'C'D — параллелограмм. Пусть О — центр описанной окружности четырехугольника ABCD, О1 и О2 — центры окружностей, симметричных описанной относительно сторон АВ и ВС (рис. 18). Отрезки ОХО2 и ОХО являются серединными перпендикулярами к отрезкам ВВ' и В А. Пусть М и К — середины этих отрезков. Отрезок МК является средней линией AOOxOg, а также средней линией ААВВ'. Рис. 18 68 XXXII Всероссийская математическая олимпиада школьников
Отсюда следует, что отрезки ОО2 и АВ' параллельны и равны. Аналогично получаем, что CD || ОО2 и CD = ОО2. Следовательно, AB'C'D — параллелограмм. Значит, В'С = AD. Аналогично CD' = ВА9 D'A' = СВ9 А'В' = DC у следовательно, четырехугольники A'B'CD' и ABCD равны. 11.5. Пусть q^O — знаменатель прогрессии. Из условия sin х следует, что tg х = = qn для некоторого цело- cos х tg х го п. Тогда ctg х = —— = tg x • q~2n также ее член. 11.6. Пусть ВВХ и ССг — две высоты пирамиды DABC (рис. 19). По условию Бх е СМ9 Сх е ВМ9 где М — середина ребра AD. Из определения высоты пирамиды следует, что ВВ1 J_ AD, СС1 1 АВ9 значит, AD 1 ВМС (по признаку перпендикулярности прямой и плоскости). Тогда ВМ — медиана и высота AABD9 т. е. АВ = BD. Аналогично из AACD АС = CD. Рассматривая другие пары высот, получаем AD = AC9 BD = ВС и АВ = AD9 ВС = CD. Отсюда следует утверждение задачи. D Рис. 19 11-7. Ответ. 60. Выделим в каждом равнобедренном треугольнике прямую, на которой лежит его основание. Если же он равносторонний, то произвольным образом объявим основанием одну из его сторон. Пусть I и т — некоторые две из проведенных прямых. Тогда существует не более одного равнобедренного треугольника, у которого на I лежит основание, а на т — боковая сторона. Действительно, направле- III ЭТАП. 2005/2006 учебный год 69
ние третьей стороны тогда задается однозначно и существует не более одной проведенной прямой этого направления. Тогда прямая I содержит основания не более чем пяти треугольников; иначе у треугольников с основаниями, лежащими на Z, было бы 12 боковых сторон и две лежали бы на одной прямой. Итого, всего есть не более 5 • 12 = 60 оснований, т. е. треугольников не больше 60. Равнобедренных треугольников будет 60, если провести 11 прямых, параллельных сторонам правильного 11-угольника, и двенадцатую прямую, перпендикулярную одной из его сторон (при этом никакие три прямые не должны пересекаться в одной точке). Легко видеть, что тогда на каждой прямой будет лежать ровно по пять оснований. Замечание. Прямые, параллельные двенадцати последовательным сторонам правильного 24-угольника, не дают оптимального примера. Действительно, если в конфигурации есть равносторонний треугольник, то для двух прямых (не содержащих его основания) будет существовать не более чем по 4 треугольника с основаниями на них. 11.8. Ответ. Если N четно, то побеждает первый; если же N нечетно, то побеждает второй. Пусть N четно. Докажем, что первый может каждым своим ходом добиваться того, что оставшиеся кучи делятся на пары равных (такую ситуацию назовем симметричной). Первым своим ходом он разделит кучу на две равные. Далее, если второй очередным своим ходом сделает некоторое действие с какой-то кучей, то первый может сделать такое же действие с парной кучей, и симметрия сохранится. Тогда, очевидно, второй не сможет взять последний камень. Пусть N нечетно. Докажем, что второй каждым своим ходом может добиться одной из двух ситуаций: либо все кучи делятся на пары равных (симметричная ситуация), либо же все кучи, кроме одной, делятся на пары равных, а в оставшейся — нечетное число камней, большее 1 (такую ситуацию назовем почти симметричной). Тогда, очевидно, первый не сможет забрать последний камень. Заметим, что перед первым ходом первого именно такая, почти симметричная ситуация. Рассмотрим, что 70 XXXII Всероссийская математическая олимпиада школьников
сделал первый своим очередным ходом. Если он совершил какое-то действие с кучей, входящей в одну из пар, то второй делает аналогичное действие с парной кучей. Если первый взял камень из непарной нечетной кучи, то она стала четной (и в ней есть еще камни!), и второй может разделить ее на две равных части, приводя к симметричной ситуации. Пусть первый разделил нечетную кучу на две части, состоящие из а и Ь камней, а < Ь. Если а < Ь — 1, то второй кучу из Ъ камней разделит на кучи из Ъ и Ь - а > 1 камней, приводя ситуацию к почти симметричной. Если же Ъ — а— 1, то второй возьмет один камень из кучи в Ъ камней, приводя ситуацию к симметричной. Итак, второй всегда может сделать ход с соблюдением этого условия. Поэтому он выиграет. IV ЭТАП (федеральный окружной) УСЛОВИЯ ЗАДАЧ ▼■ 8 класс 8.1. Найдите какое-нибудь девятизначное число iV, состоящее из различных цифр, такое, что среди всех чисел, получающихся из N вычеркиванием семи цифр, было бы не более одного простого. Докажите, что найденное число подходит. (Если полученное вычеркиванием цифр число начинается на ноль, то ноль тоже вычеркивается.) (О. Подлипский) 8.2. Числа 1, 2, 3, 4 расположены по кругу. Двое играют в такую игру. Каждым своим ходом первый прибавляет к двум соседним числам по 1, а второй меняет любые два соседних числа местами. Первый выигрывает, если все числа станут равными. Может ли второй ему помешать? (П.Мартынов) 8-3. В круговых автогонках участвовали четыре гонщика. Их машины стартовали одновременно из одной точки и двигались с постоянными скоростями. Известно, что после начала гонок для любых трех машин нашелся момент, когда они встретились. Докажите, что IV ЭТАП. 2005/2006 учебный год 71
после начала гонок найдется момент, когда встретятся все 4 машины. (Гонки считаем бесконечно долгими по времени.) (И.Богданов, П.Кожевников, О. Под- липский, Г. Челноков) 8.4. Каждая деталь конструктора «Юный паяльщик» —- это скобка в виде буквы «П», состоящая из трех единичных отрезков. Можно ли из деталей этого конструктора спаять полный проволочный каркас куба 2x2x2, разбитого на кубики 1x1x1? (Каркас состоит из 27 точек, соединенных единичными отрезками; любые две соседние точки должны быть соединены ровно одним проволочным отрезком.) (Л. Емельянов) 8.5. На доске записано произведение аг • а2 • ... • а100, где аи ..., а100 — натуральные числа. Рассмотрим 99 выражений, каждое из которых получается заменой одного из знаков умножения на знак сложения. Известно, что значения ровно 32 из этих выражений четные. Какое наибольшее количество четных чисел среди а19 а2, ..., а100 могло быть? (Р.Женодаров) 8.6. В клетчатом квадрате 101 х 101 каждая клетка внутреннего квадрата 99 х 99 покрашена в один из десяти цветов, а все клетки, примыкающие к границе квадрата 101 х 101, не покрашены. Может ли оказаться, что в каждом квадрате 3 х 3 в цвет центральной клетки покрашена еще ровно одна клетка? (Н. Агаханов) 8.7. Медиану АА0 треугольника ABC отложили от точки Ао перпендикулярно стороне ВС во внешнюю сторону треугольника. Обозначим второй конец построенного отрезка через Аг. Аналогично строятся точки Вх и С1в Найдите углы треугольника A^Bfi^ если углы треугольника ABC равны 30°, 30° и 120°. (Л. Емельянов) 8.8. При изготовлении партии из N (N ^ 5) монет работник по ошибке изготовил две монеты из другого материала (все монеты выглядят одинаково). Начальник знает, что таких монет ровно две, что они весят одинаково, но отличаются по весу от остальных. Работник знает, какие это монеты и что они легче остальных. Ему нужно, проведя два взвешивания на чашечных весах без гирь, убедить начальника в том, что фальшивые монеты легче настоящих, и в том, какие именно монеты фальшивые. Может ли он это сделать? (К.Кноп, Л.Емельянов) 72 XXXII Всероссийская математическая олимпиада школьников
г 9 класс 9.1. Найдите какое-нибудь девятизначное число N, состоящее из различных цифр, такое, что среди всех чисел, получающихся из N вычеркиванием семи цифр, было бы не более одного простого. Докажите, что найденное число подходит. (Если полученное вычеркиванием цифр число начинается на ноль, то ноль тоже вычеркивается.) (О. Подлипский) 9.2. В каждую клетку бесконечной клетчатой плоскости записано одно из чисел 1, 2, 3, 4 так, что каждое число встречается хотя бы один раз. Назовем клетку правильной, если количество различных чисел, записанных в четыре соседние (по стороне) с ней клетки, равно числу, записанному в эту клетку. Могут ли все клетки плоскости оказаться правильными! (Н. Ага- ханов) 9.3. Известно, что х\ + х\ + ... + х\ — 6 и хх + х2 + ... + + х6 = О. Докажите, что хххг,,,х^ ^ -. (А. Храброе) 9.4. Биссектрисы углов А и С треугольника ABC пересекают описанную окружность этого треугольника в точках Ао и Со соответственно. Прямая, проходящая через центр вписанной окружности треугольника ABC параллельно стороне АС, пересекается с прямой А0С0 в точке Р. Докажите, что прямая РВ касается описанной окружности треугольника ABC. (Л. Емельянов) 9.5. На доске записано произведение аг - а2 • ... ■ а100, где а19 ..., а100 — натуральные числа. Рассмотрим 99 выражений, каждое из которых получается заменой одного из знаков умножения на знак сложения. Известно, что значения ровно 32 из этих выражений четные. Какое наибольшее количество четных чисел среди а19 а2, ..., а100 могло быть? (Р.Женодаров) 9.6. В остроугольном треугольнике ABC проведена биссектриса AD и высота BE. Докажите, что угол CED больше 45°. (М.Мурашкин) 9.7. При изготовлении партии из N (N ^ 5) монет работник по ошибке изготовил две монеты из другого материала (все монеты выглядят одинаково). Начальник знает, что таких монет ровно две, что они весят одинаково, но отличаются по весу от остальных. IV ЭТАП. 2005/2006 учебный год 73
Работник знает, какие это монеты и что они легче остальных. Ему нужно, проведя два взвешивания на чашечных весах без гирь, убедить начальника в том, что фальшивые монеты легче настоящих, и в том, какие именно монеты фальшивые. Может ли он это сделать? (К.Кноп, Л. Емельянов) 9.8. Число N, не делящееся на 81, представимо в виде суммы квадратов трех целых чисел, делящихся на 3. Докажите, что оно также представимо в виде суммы квадратов трех целых чисел, не делящихся на 3. (П. Козлов) (5) 10 класс 10.1. Натуральные числа от 1 до 200 разбили на 50 множеств. Докажите, что в одном из них найдутся три числа, являющиеся длинами сторон некоторого треугольника. (М.Мурашкин) 10.2. Назовем раскраску доски 8 х 8 в три цвета хорошей, если в любом уголке из пяти клеток присутствуют клетки всех трех цветов. (Уголок из пяти клеток — это фигура, получающаяся из квадрата 3x3 вырезанием квадрата 2x2.) Докажите, что количество хороших раскрасок не меньше чем б8. (О.Подлипский) 10.3. Биссектрисы углов А и С треугольника ABC пересекают описанную окружность этого треугольника в точках Ао и Со соответственно. Прямая, проходящая через центр вписанной окружности треугольника ABC параллельно стороне АС, пересекается с прямой А0С0 в точке Р. Докажите, что прямая РВ касается описанной окружности треугольника ABC. (Л. Емельянов) 10.4. Даны п (п > 1) приведенных квадратных трехчленов х2 — агх + Ьи ..., х2 — апх + bn, причем все 2/г чисел а19 ..., anJ bl9 ..., Ьп различны. Может ли случиться, что каждое из чисел а19 ..., anJ bl9 ..., Ьп является корнем одного из этих трехчленов? (А.Бадзян) 10.5. Докажите, что для каждого х9 такого, что sin x * 0, найдется такое натуральное п, что |sin nx\ > ^-. (И. Богданов, А. Храброе) 10.6. Через точку Н пересечения высот остроугольного треугольника ABC проходят три окружности, каждая из которых касается одной из сторон треугольника 74 XXXII Всероссийская математическая олимпиада школьников
в основании высоты. Докажите, что вторые точки пересечения окружностей (отличные от Н) являются вершинами треугольника, подобного исходному. (Л. Емельянов) 10.7. При каких натуральных п найдутся такие положительные рациональные, но не целые числа а и Ь, что оба числа а + Ъ и ап + Ьп целые? (В. Сендеров) 10.8. У выпуклого многогранника 2/г граней (п ^ 3), и все грани являются треугольниками. Какое наибольшее число вершин, в которых сходится ровно 3 ребра, может быть у такого многогранника? (А. Гарбер) т 11 класс 11.1. Натуральные числа от 1 до 200 разбили на 50 множеств. Докажите, что в одном из них найдутся три числа, являющиеся длинами сторон некоторого треугольника. (М.Мурашкин) 11.2. Произведение квадратных трехчленов х2 + ахх + bl9 х2 + а2п + b2y ..., х2 + апх + bn равно многочлену Р (х) = х2п + схх2п-1 + с2х2п-2 + ... + с2п_ хх + с2п, где коэффициенты с19 с2, ..., с2п положительны. Докажите, что для некоторого k (I ^ k ^ п) коэффициенты ak и bk положительны. (В. Сендеров) 11.3. В гоночном турнире 12 этапов и п участников. После каждого этапа все участники в зависимости от занятого места k получают баллы ak (числа ak натуральны и ах > а2 > ... > ап). При каком наименьшем п устроитель турнира может выбрать числа аи ..., ап так, что после предпоследнего этапа при любом возможном распределении мест хотя бы двое участников имели шансы занять первое место. (М.Мурашкин) 11.4. Биссектрисы углов А и С треугольника ABC пересекают его стороны в точках Ах и С19 а описанную окружность этого треугольника — в точках Ао и Со соответственно. Прямые АХСХ и А0С0 пересекаются в точке Р. Докажите, что отрезок, соединяющий Р с центром вписанной окружности треугольника ABC, параллелен АС. (Л. Емельянов) 11-5. Докажите, что для каждого х, такого, что sin x Ф 0, i . i ^ V3 найдется такое натуральное /г, что |sinn:x;|^—. (И.Богданов, А.Храброе) IV ЭТАП. 2005/2006 учебный год 75
11.6. В тетраэдре ABCD из вершины А опустили перпендикуляры АВ', АС у AD' на плоскости, делящие двугранные углы при ребрах CD, BDy ВС пополам. Докажите, что плоскость (B'C'D') параллельна плоскости (BCD). (А.Бадзян) 11.7. Докажите, что если натуральное число N представляется в виде суммы трех квадратов целых чисел, делящихся на 3, то оно также представляется в виде суммы трех квадратов целых чисел, не делящихся на 3. (П. Козлов) 11.8. Какое минимальное количество клеток можно закрасить черным в белом квадрате 300 х 300, чтобы никакие три черные клетки не образовывали уголок (квадрат 2x2 без одной клетки), а после закрашивания любой белой клетки это условие нарушалось? (И.Богданов, О.Подлипский) РЕШЕНИЯ ЗАДАЧ 0 8 класс 8.1. Подойдет, например, число 391 524 680. Действительно, если не вычеркнуть одну из последних шести цифр, то полученное число будет составным (оно будет делиться либо на 5, либо на 2). Если же вычеркнуть шесть последних цифр, то останется вычеркнуть одну цифру из числа 391. А числа 39 и 91 составные. 8.2. Ответ. Может. Докажем, что второй может добиться того, чтобы перед каждым ходом первого четные и нечетные числа чередовались. Тогда этим ходом первому не удастся сделать все числа равными, и он не выиграет. Заметим, что изначально четные и нечетные числа чередуются. После любого хода первого игрока получим подряд два четных и два нечетных числа. Поменяем четное и соседнее с ним нечетное число местами. Таким образом, после хода второго четные и нечетные числа чередуются, поэтому первый игрок никогда не получит четыре равных числа (рис. 20). Ч — Н Н — Н Н — Ч Рис.20 Н —Ч Ч —Ч Ч —Н 76 XXXII Всероссийская математическая олимпиада школьников
8 3. Достаточно решить задачу для случая, когда самый медленный гонщик стоит на месте в точке старта, а остальные (назовем их Петя, Вася и Коля) едут со скоростями, уменьшенными на скорость самого медленного; при таком изменении условия моменты встреч гонщиков останутся неизменными. Пусть до первой встречи с Васей в точке старта Петя проехал а кругов, а с Колей — Ъ кругов. Тогда Петя будет встречаться на старте с Васей через каждые а кругов, а с Колей через каждые Ъ кругов, и через аЪ Петиных кругов в точке старта окажутся все трое. 8.4. Ответ. Нельзя. Так как каркас состоит из 54 единичных отрезков, для его изготовления необходимо 18 деталей (54 = 18 ■ 3). Рассмотрим одну из восьми вершин куба. Для того чтобы припаять к ней три отрезка, нужно использовать не менее двух деталей. То есть для формирования вершин куба нужно не менее 16 деталей. Рассмотрим центр куба. Из него должно выходить 6 отрезков, и для этого необходимо не менее трех деталей. А так как одна и та же деталь не может использоваться в вершине куба и в его центре, то необходимо не менее 19 деталей. Противоречие с начальным условием. 8.5. Ответ. 33. Рассмотрим самое левое четное число аь и самое правое четное число ak. Заметим, что четными являются все суммы с номерами от i до k — 1, и только они (в суммах с меньшими номерами первое слагаемое нечетно, а второе четно; в суммах с большими номерами наоборот). Таким образом, k - i = 32. Между at и ak могут стоять как четные, так и нечетные числа — от этого четность сумм не изменится. Количество четных чисел будет наибольшим, если все числа между at и ak также четны. В этом случае количество четных чисел будет равно 33. 8-6. Ответ. Не может. Пусть условие задачи выполнено. Тогда каждой покрашенной клетке А поставим в соответствие клетку В (отличную от А) того же цвета, находящуюся в квадрате 3 х 3 с центральной клеткой А. По условию для данной клетки А клетка В определяется единственным образом. Заметим, что клетка А нахо- IV ЭТАП. 2005/2006 учебный год 77
дится в квадрате 3 х 3, у которого центральной является клетка В, значит, клетка А поставлена в соответствие клетке В. Таким образом, рассмотренное соответствие разбивает все покрашенные клетки на пары. Однако количество покрашенных клеток равно 992 — нечетному числу. Противоречие. 8.7. Ответ. АА1В1С1 — равносторонний, все углы равны 60°. Так как ААВС равнобедренный, то ВВ0 — серединный перпендикуляр к основанию АС (рис. 21). Значит, Вх лежит на этом перпендикуляре и СВ0 ± ВВ1и Рис. 21 Вг В АВСВХ точка Бо — основание высоты и медианы на стороне ВВи откуда ВС = ВгС, а так как ZBxBC = \zABC, то угол ВХВС равен 60°. Значит, АВХВС равносторонний, а точка Ао, будучи серединой стороны ВС, является основанием высоты в треугольнике ВХВС. Получается, что точки Аг и Вг лежат на серединном перпендикуляре к отрезку ВС. Аналогично точки Сх и Вг лежат на серединном перпендикуляре к отрезку АВ. Значит, в четырехугольнике BxCqBAq два угла прямые, поэтому ZC1BlA1 = 180° - ZABC = 60°, + С0Сг = ВгС1У т. е. АА1В1С1 — равнобедренный с углом при вершине 60°. 8.8. Ответ. Может. Пусть т1 и т2 — фальшивые монеты, т3, т4 и т5 — какие-то три из настоящих монет. Работник делает два таких взвешивания: т1 v m3, 78 XXXII Всероссийская математическая олимпиада школьников
В результате начальник убеждается, что ml< тг и тА + тпъ > m2 + m3. Из первого взвешивания он делает вывод, что т1 — более легкая монета, чем т3, а из второго — что среди монет т4 и тъ тяжелых монет больше, чем среди т2 и т3. Таким образом, он делает вывод, что т4 и тъ — тяжелые монеты, а т2 — легкая. В итоге он видит три тяжелых монеты и две легких, а поскольку он знает, что фальшивых монет ровно две, то он у беж дается в том, что фальшивыми являются именно легкие монеты т1 и т2. т 9 класс 9.1. См. решение задачи 8.1. 9.2. Ответ. Не могут. Предположим, что при некоторой расстановке чисел все клетки оказались правильными. Пусть в клетку А записано число 4, тогда в одной из соседних с А клеток, например в клетку Б, записано число 4, а в одну из клеток С, £>, Е — число 1 (рис. 22). Если число 1 записано в клетку С, то в клетку F также записано 4 — противоречие: у клетки В две соседние клетки с числом 4. Аналогично число 1 не может быть записано в клетку Е. Значит, число 1 — в клетке D. Аналогично число 1 — в клетке G, а тогда в клетках U и V—опять четверки. Итак, числа 4 порождают «цепочки» ...-1 — 4-4-1 — 4 — 4-.... Из того, что в клетку К записано число 4, следует, что в клетку М записано 4, в клетку N — число 1, и тогда однозначно восстанавливаются числа в выделенных строках и столбцах (см. рис. 22). Далее, в один из незаполненных еще квадратов 2x2 записа- Рис. 22 4А Е IV ЭТАП. 2005/2006 учебный год 79
но число 3. Пусть для определенности число 3 записано в клетку С. В клетки F и Н нельзя записать числа 1 и 4, значит, там записаны числа 2 и 3. Без ограничения общности пусть число 2 записано в клетку F, а число 3 — в клетку Н. Рассматривая клетку Н9 получаем, что в клетку Р записано число 2. Но в этом случае у клетки F с числом 2 в соседних клетках три различных числа. Противоречие. 9.3. Если среди чисел х19 х2У ..., х6 нечетное число отрицательных или есть ноль, то неравенство очевидно. Пусть отрицательных чисел два или четыре. Меняя, если нужно, знаки у всех чисел, можно добиться того, что ровно два числа будут отрицательными. Пусть для определенности это числа хги х2. Положим yk = \xk\. Тогда у\ + у\ + ... + у\ = 6 и у1 + у2 = у3 + у4 + + уъ + у6. Обозначим последнюю сумму через s. По неравенству о средних для двух чисел имеем Аналогично УгУАУьУб ^ I 4 J ~ 4*"" s6 Значит, У!У2-..Уб<^5- С другой стороны, 2 (у! + у1) > (г/1 + у2)2 = s2n 4 (у* + у\ + у! + yl) > (уг + г/2 + г/з + г/4)2 = *2, что следует из неравенства между средним арифметическим и средним квадратическим (или доказывается с помощью простого раскрытия скобок). Таким образом, Стало быть, 9.4. Обозначим центр вписанной окружности через /. Пусть Во — точка пересечения биссектрисы угла В с описанной окружностью (рис. 23). Тогда ZIBC0 = | (ЛСО + АВ0) = | (ВСО + СВ0) = ZBIC0, 80 XXXII Всероссийская математическая олимпиада школьников
Рис. 23 т. е. треугольник В1С0 равнобедренный, ВС0 = 1С0. Аналогично ВА0 = 1А0, значит, треугольники Ао/Со и AqBCq равны (см. рис. 23). Следовательно, точки В и J симметричны относительно С0А0, т. е. А0Р — серединный перпендикуляр к отрезку BI. Пусть для определенности точка Р лежит на луче С0А0. Пользуясь симметрией относительно AqP, имеем ZAqBP = ZAqIP = ZA0AC = ZA0AB. Получилось, что угол между хордой А0Б окружности и прямой ВР равен вписанному углу этой окружности, опирающемуся на дугу AqB. По теореме, обратной теореме об угле между касательной и хордой (если прямая, проходящая через конец хорды окружности, образует с этой хордой угол, равный вписанному углу, опирающемуся на эту хорду, то эта прямая — касательная), заключаем, что ВР — касательная к описанной окружности треугольника ABC. 9.5. Ответ. 33. См. решение задачи 8.5. 9.6. Обозначим через К, L, М основания перпендикуляров, опущенных из точки D соответственно на АВ, AC, BE (рис. 24). (Точки К, L, М лежат на отрезках АВУ АСУ BE, а не на их продолжениях, так как ААВС К Рис. 24 D IV ЭТАП. 2005/2006 учебный год 81
остроугольный.) Пусть DK пересекает BE в точке Р. Заметим, что DLEM — прямоугольник, а точка D равноудалена от сторон АВ и АС, так как лежит на биссектрисе AD. Отсюда ЕМ = DL = DK > DP > DM. Поскольку в треугольнике EDM против большей стороны лежит больший угол, ZEDM > ZDEM = = 90° - ZEDM, поэтому ZEDM > 45°. Но ZEDM = = ZCED, откуда получаем требуемое. 9.7, Ответ, Может. См. решение задачи 8.8. 9.8, Пусть N = 9а2 + 9Ь2 + 9с2. Если числа а, Ь, с делятся на 3, то Л^ делится на 81, что неверно. Пусть для определенности а не делится на 3, тогда одна из сумм а + b + с или (-а) + Ъ + с не делится на 3. Заменив, если нужно, а на (-а), можно считать, что а + Ъ + с не делится на 3. Тогда N = 9а2 + 9Ь2 + 9с2 = = (2а + 2Ь - с)2 + (2а + 2с - Ъ)2 + (2Ь + 2с - а)2, где число 2а + 2Ь - с = 2 (а + b + с) - Зс не делится на 3; аналогично и с остальными числами. Требуемое представление получено. •▼* 10 класс 10.1. Рассмотрим только числа 100, 101, ..., 200. Так как их всего 101, а множеств 50, то какие-то три из них попадут в одно множество. Сумма любых двух из этих трех чисел больше 200 и, следовательно, больше третьего числа. Значит, существует треугольник с соответствующими длинами сторон, что и требовалось доказать. 10.2, Заметим, что любой уголок содержит горизонтальный прямоугольник 1x3. Также заметим, что любую горизонталь можно покрасить в три цвета так, чтобы любой прямоугольник 1x3 содержал клетки трех различных цветов А, Б и С, ровно 3! = 6 способами (получится, например, раскраска горизонтали ABC ABC АВ, а цвета А, Б, С можно выбрать 6-ю способами). Это означает, что раскраска, при которой каждая горизонталь покрашена указанным способом, будет хорошей. Поскольку каждую горизонталь можно покрасить независимо от других, число хороших раскрасок такого вида будет в точности б8. о XXXII Всероссийская математическая олимпиада школьников
Замечание. Покажем, что есть еще хорошие раскраски, которые мы не посчитали, например такая, при которой каждая из горизонталей одноцветна, а цвета горизонталей чередуются указанным выше образом (в этом случае любой вертикальный прямоугольник 1x3 будет содержать клетки трех различных цветов). Поэтому общее количество хороших раскрасок больше б8. 10.3. См. решение задачи 9.4. 10.4. Ответ. Не может. Предположим противное. Поскольку все 2п коэффициентов различны, то они и составляют все множество корней наших трехчленов, причем у каждого из них два корня. Пусть at, (3^ — корни трехчлена х2 - atx + bt. Тогда по теореме Виета а4 = ос£ + Р£, b; = (X;pj. Поскольку множество корней многочленов совпадает с множеством {аи ..., ал, bl9 ..., Ьл}, то i = 1 i = 1 i = 1 i = l / = 1 л Следовательно, ^ bt = 0. i = 1 Далее, a2 + p? = (a, + P,)2 - 2a,p£ = a? - 2b,. Тогда Значит, ^ft? = 0, т.е. все коэффициенты ft, равны i = 1 нулю. Это противоречит тому, что они различны. 10.5. Поскольку функция «синус» нечетная и имеет период 2л, можно считать, что 0 < х < л. Если J ^ х ^ -^, то подойдет п = 1. О о Если 0 < jc < ^, то, последовательно откладывая углы jc, 2jc, ..., пх, ..., мы когда-нибудь выйдем из промежутка 0; ^ , а поскольку «шаг» меньше ^, то мы V у при этом попадем в уже рассмотренный промежуток "я. 2я] 3' 3 J" IV ЭТАП. 2005/2006 учебный год 83
Если же хе —; к , то, учитывая равенство |sin nx\ = V d ) = | sin гс (я - х) |, этот случай сводится к случаю х е О; ^ L 10,6. Обозначим через if ортоцентр (т. е. точку пересечения высот) ААВС, через Hl9 Н2, Н3 — основания высот на сторонах ВС, С А, АВ соответственно, а через А19 В1У Сг — вторые точки пересечения окружностей (рис. 25). В Рис. 25 Так как прямая ВС — касательная к окружности, проходящей через точки Н и Нг и ННг ± ВС, то НН1 — диаметр одной из окружностей, данных в условии. Аналогично для двух других окружностей. На диаметр в любой окружности опирается прямой угол, поэтому ZHCXH2 + ZHC1H1 = 90° + 90° = 180°, т. е. точка Сх лежит на отрезке 11^2. Аналогично Ах е Н2Н3. Точки Б, Н2, Н3, С лежат на окружности с диаметром ВС, следовательно, ZHH2A1 — ZBH2HS = = \ ВН3 = ZBCH3 = 90° - ZABC. Аналогично ZHH2CY = Li = 90° - ZABC. Получается, что прямоугольные треугольники НН2АХ и НН2С1 равны, а точки Ах и Сх симметричны относительно НН2. Значит, АхСг _L HH2, откуда АгСг \\ АС. Аналогично ВХСХ || ВС и AXBX || AB, что и доказывает подобие треугольников. 10.7. Ответ. При всех нечетных п. Если п нечетно, то положим а = -, Ь = Тогда 84 XXXII Всероссийская математическая олимпиада школьников
a + b — целое, n an + bn = (a + b)(anl - ап~2Ь+ ... + bnl) = -2n~x (anl - an~2b + ... + bn ~x); знаменатель дроби в скобках равен 2п~г, поэтому число ап + Ьп также целое. Пусть п четное, п = 2k. Предположим, что требуемые числа а, Ъ нашлись. Так как их сумма целая, то знаменатели в их несократимой записи равны, т. е. их несократимая запись такова: а = —, b = -; при а а этом р + q кратно d. Тогда рп + qn = (p2k - q2k) + 2q2k = = (P2 ~ q2)(p2k~2 + p2k~4q2 + ... + q2k~2) + 2q2k = (p -f q)K + + 2q2k, где К = (p - q)(pn-2 +pn~4q2 + ... + qn~2) — це- pn + qn лое число. Заметим, что ап + bn = целое, d т. е. рп + qn делится на dn; в частности, оно делится на d. А так как р + q делится на d, то и 2qn должно делиться на d. Поскольку дробь - несократима, то 2 делится на d, т. е. d = 2. Но тогда рп и qn — квадраты нечетных чисел, следовательно, дают остаток 1 при делении на 4. Поэтому рП + дп не делится на 4, а должно делиться на 22\ которое кратно 4. Противоречие. [п 3 J Назовем вершину, в которой сходится ровно 3 ребра, хорошей. Докажем, что никакие две хорошие вершины не лежат в одной грани. Предположим противное — пусть хорошие вершины А л В лежат в одной грани ABC. Ребро АВ принадлежит еще одной грани ABD. Поскольку вершина А — хорошая, то, кроме АВ, АС, AD, нет других ребер, выходящих из А. В вершине А сходятся ровно 3 грани — ABC, ABD и грань, содержащая ребра АС и AD, т. е. грань ACD. Аналогично получаем, что BCD является гранью многогранника. Получается, что многогранник является тетраэдром ABCD, что противоречит условию п> 3. Из доказанного следует, что каждой хорошей вершине можно сопоставить три грани, сходящиеся в ней, причем различным хорошим вершинам сопоставлены IV ЭТАП. 2005/2006 учебный год 85
разные грани. Отсюда следует, что количество хоро- 2п ших вершин не превосходит —. о Опишем построение выпуклого 2гс-гранника Тп9 для которого количество хороших вершин Н(Тп '■>=[¥]• Определим вначале процедуру наращивания грани многогранника Т с треугольными гранями. Пусть KLM — одна из граней, KLP, LMQ, MKR — грани, отличные от KLM, содержащие соответственно ребра KL9 LMy МК (точки Р, Q, R не обязательно различны) (рис. 26). Пусть X — некоторая внутренняя точка треугольника KLM. На перпендикуляре к плоскости KLM, восставленном в точке X, вне многогранника Т выберем такую точку N, что точки К и N лежат по одну сторону от плоскости LMQ, точки L и N — по одну сторону от MKR, точки М и N — по одну сторону от KLP (этого можно добиться, выбирая длину XN достаточно малой). Рассмотрим многогранник Т", К Рис. 26 / в котором по сравнению с Т добавлена вершина N, а грань KLM заменена на три грани KLNy LMN, MKN. По построению многогранник Т' — выпуклый. Будем говорить, что Т" получен из Т наращиванием грани KLM с помощью вершины N. Склеив две равные правильные треугольные пирамиды по основаниям, получим пример многогранника Гд, для которого Н (Т3) = 2. Пусть уже имеется выпуклый бй-гранник T3k и Н (T3k) = 2k. Рассмотрим хорошую вершину А, в которой сходятся ребра АВ9 АСУ AD (и грани ABC, ABD, ACD). По доказанному вершины Б, С, D не являются хорошими. Построим многогранник T3k + lJ наращивая грань ABC с помощью вершины Е (рис. 27). При этом Н (Tsk + 1) 86 XXXII Всероссийская математическая олимпиада школьников
D Рис. 27 осталось равным 2k = 2 (Sk + 1) (добавилась хорошая вершина Е, а вершина А перестала быть хорошей). Многогранник T3k + 2 получим, наращивая в многограннике T3k + 1 грань ABD с помощью вершины F. При этом добавилась хорошая вершина F и ни одна хорошая вершина не перестала быть хорошей, [2 (Sk + 2)1 . Многогран- 3 J ник Т3 ik +1} получим, наращивая в многограннике T3k + 2 грань ACZ) с помощью вершины G. При этом добавилась хорошая вершина G и снова ни одна хорошая вершина не перестала быть хорошей, поэтому [ С помощью описанного построения по цепочке получаем нужные примеры многогранников Г3, Г4, Е5, Т6, .... т 11 класс 11-1. См. решение задачи 10.1. 11-2. Заметим, что Р (х) > 0 при х > 0, поэтому Р (я;) не имеет неотрицательных корней. Если при некотором k выполнено Ък ^ 0, то трехчлен х2 + akx + ЬЛ имеет неотрицательный корень. (Это следует из теоремы Виета.) Таким образом, Ьк > 0 для всех ft. Далее, сг = ах + а2 + ... + ал > 0, поэтому ал > 0 хотя бы при одном fe. 113. Ответ. 13. Предположим, что участников не больше 12. Пусть при этом один из участников (назовем его А) выиграл 87 IV ЭТАП. 200Е/2006 учебный год
все 11 этапов, а каждый из оставшихся хотя бы раз занял последнее место. Тогда участник А после 12 этапов наберет не меньше 11а1 + ап очков, а каждый из оставшихся — не больше ап + 10а2 + аг очков (поскольку каждый из оставшихся занимал последнее место на одном из этапов и мог быть первым только на 12 этапе), что меньше, чем llaj + an. Следовательно, только А может занять первое место. Поэтому для выполнения условия количество участников должно быть не менее 13. Пусть участников 13, тогда устроителю достаточно выбрать числа ах = 1011, а2 = 1010, ..., а12 = 1000, а13=1. Рассмотрим участника, набравшего наибольшее количество очков после 11 этапов (назовем его А). Тогда среди 12 оставшихся найдется по крайней мере один участник, который не занимал последнего места ни на одном из этапов (назовем его В). Заметим, что А набрал не более 1011 • 11 = 11 121 очков, а Б — не менее 1000 • 11 = 11 000 очков. То есть после 11 этапов А опередит В не более чем на 121 очко. Очевидно, что после 11 этапа А имеет шансы занять первое место. Однако если на 12 этапе А займет последнее место, то В наберет на 12 этапе хотя бы на 999 очков больше, чем А и обгонит его. То есть А будет не первым. Это и означает, что еще какой-то участник после 11 этапа имеет шансы занять первое место. 11.4. Обозначим центр вписанной окружности через I (рис. 28). Заметим, что ZIAC0 = \ (вС0 + ВА0) = — - \АС0 +СА0) = ZAICQy т. е. треугольник А1С0 т 88 XXXII Всероссийская математическая олимпиада школьников
равнобедренный, С0А = С01. Далее, ZC0AC1 = — С0В = = - C0A = ZC0CA, поэтому треугольники СОАСХ и С0СА С А С С подобны по двум углам. Отсюда получаем ° = ° , т* е* С01 _С0С 01 ^О1 Проведем через точку I прямые !и/п, параллельные АС и АгСг соответственно. Пусть Р' — точка пересечения прямых I и АХС^ a Q — точка пересечения прямых m и АС. Из доказанного соотношения следует, что при гомотетии с центром Со, переводящей С1 в J, точка I переходит в С, прямые I и АХСХ — в АС и m соответственно; поэтому точка Р' переходит в точку Q. Следовательно, точка Со лежит на прямой P'Q. Аналогично Ао лежит на прямой P'Q, поэтому P'Q и А0С0 совпадают, Р' лежит на А0С0 и совпадает с Р, что и требовалось. 11.5. См. решение задачи 10.5. 11.6. Продолжим отрезок AD' до пересечения с плоскостью BCD в точке D" (рис. 29). Так как плоскости (BCD) и (ABC) симметричны относительно биссектор- ной плоскости, то AD' = D'D". Аналогично по точкам С и В' строим точки Си Б". При гомотетии с центром А и коэффициентом - плоскость А (B"C"Dn) переходит в плоскость (B'C'D')y поэтому (B'C'D') || (B"C"D") = (BCD), что и требовалось доказать, 11.7. Пусть первое из чисел имеет вид Зп • а, где а не делится на 3, а два других числа делятся на Зл. Из условия следует, что число N можно представить в виде где п е Ny a, b, с е Z. Лемма. Всякое число вида (*) можно представить в виде 9п ~х (х2 + у2 + 22), где х, у, z e Z, jc, i/, x не делятся на 3. IV ЭТАП. 2005/2006 учебный год 89
Доказательство. Без ограничения общности будем считать, что а + Ь + с не делится на 3 (иначе число а можно заменить на -а). Имеем 9 (а2 + Ь2 + с2) = = (4а2 + 4Ь2 + с2) + (4Ь2 + 4с2 + а2) + (4с2 + 4а2 + Ь2) = = (2а + 2Ь - с)2 + (2Ь + 2с - а)2 + (2с + 2а - Ъ)2. Каждое из чисел 2а + 2Ь - с, 2Ь 4- 2с — а, 2с + 2а — Ь сравнимо по модулю 3 с не делящимся на 3 числом 2 (а + Ь 4- с). Значит, эти числа тоже не делятся на 3. Лемма доказана. Для завершения решения осталось применить эту лемму п раз. 11.8. Ответ. 30 000. Закрасив в квадрате все клетки 2-й, 5-й, 8-й, ..., 299-й горизонталей в черный цвет, получим требуемый пример. Осталось показать, что меньшим количеством закрашенных клеток не обойтись. Пусть раскраска, в которой есть Ъ черных и w белых клеток, удовлетворяет условию задачи. Запишем в каждую черную клетку ноль. Затем для каждой белой клетки выполним такую операцию. Если после ее закрашивания она становится центральной клеткой черного уголка, то прибавим к обеим остальным клеткам этого уголка по 1. В противном случае прибавим 2 к центральной клетке полученного уголка. В обоих случаях если получилось несколько уголков, то мы выполняем указанную операцию лишь с одним из них. Тогда сумма всех чисел в черных клетках равна 2w. Покажем, что в произвольной черной клетке А стоит число, не большее 4. Если у нее нет черных соседей, то после перекрашивания любого белого соседа А не может стать центральной клеткой уголка, поэтому каждый сосед добавил в нее не более 1. Если у А не Рис. 30 более двух белых соседей, то каждый из них добавил не более 2. Поэтому больше 4 в ней может добавиться только в том случае, если у нее 1 черный и 3 белых соседа (рис. 30). Клетка С не могла добавить в А двойку, так как тогда одна из клеток X или Z была бы черной. Если С добавила в А единицу, то одна из клеток У и Г черная — пусть это У. Тогда после закрашивания X 90 XXXII Всероссийская математическая олимпиада школьников
она (клетка X) становится центральной клеткой уголка, и поэтому также добавляет не более 1, a Z — не более 2. Если же С ничего в А не добавляет, то в А опять же не больше 4, полученных из клеток X и Z. Итого, в каждой черной клетке записано не более 4, поэтому сумма всех чисел не больше 4fc, т. е. 2w ^ 4fc, и черных клеток не меньше чем треть, что и требовалось. V ЭТАП (заключительный) УСЛОВИЯ ЗАДАЧ ▼> 9 класс 9.1. Дана доска 15 х 15, состоящая из клеток 1x1. Некоторые пары центров соседних по стороне клеток соединили отрезками так, что получилась замкнутая несамопересекающаяся ломаная, симметричная относительно одной из диагоналей доски. Докажите, что длина ломаной не больше 200. (С.Берлов, И.Богданов) 9.2. Докажите, что найдутся 4 таких целых числа a, fc, с, d, по модулю большие 1 000 000, что abed abed (С.Берлов) 9.3. Петя раскрашивает 2006 точек, расположенных на окружности, в 17 цветов. Затем Коля проводит хорды с концами в отмеченных точках так, чтобы концы любой хорды были одноцветны и хорды не имели общих точек (в том числе и общих концов). При этом Коля хочет провести как можно больше хорд, а Петя старается ему помешать. Какое наибольшее количество хорд заведомо сможет провести Коля? (С.Берлов) 9.4. Дан треугольник ABC. Окружность со касается описанной окружности треугольника ABC в точке А, пересекает сторону АВ в точке К, а также пересекает сторону ВС. Касательная CL к окружности со такова, что отрезок KL пересекает сторону ВС в точке Т. Докажите, что отрезок ВТ равен по длине касательной из точки Б к со. (Д. Скробот) V ЭТАП. 2005/2006 учебный год 91
9,5- Пусть al9 а2, ..., а10 — натуральные числа, а1 < а2 < ... < а10. Пусть bk — наиболыпий делитель ak, такой, что bk < ak. Оказалось, что Ьх > Ь2 > ... > Ь10. Докажите, что а10 > 500. (М.Мурашкин) 9.6. На сторонах АВ, ВС, С А треугольника ABC выбраны точки Р, Q, R соответственно таким образом, что АР = CQ и четырехугольник RPBQ вписанный. Касательные к описанной окружности треугольника ABC в точках А и С пересекают прямые RP и RQ в точках X и У соответственно. Докажите, что RX = RY. (С.Берлов) 9.7. Клетчатый квадрат 100 х 100 разрезан на «доминошки», т. е. прямоугольники 1x2. Двое играют в игру. Каждым ходом игрок склеивает две соседние по стороне клетки, между которыми был проведен разрез. Игрок проигрывает, если после его хода фигура получилась связной, т. е. весь квадрат можно поднять со стола, держа его за одну клетку. Кто выиграет при правильной игре — начинающий или его соперник? (И.Богданов) 9.8. Дан квадратный трехчлен / (х) = х2 + ах + Ь. Уравнение / (/ (х)) = 0 имеет четыре различных действительных корня, сумма двух из которых равна -1. Докажите, что Ъ < --. (С.Берлов) 4 ф 10 класс 10.1. Дана доска 15 х 15, состоящая из клеток 1x1. Некоторые пары центров соседних по стороне клеток соединили отрезками так, что получилась замкнутая несамопересекающаяся ломаная, симметричная относительно одной из диагоналей доски. Докажите, что длина ломаной не больше 200. (С.Берлов, И.Богданов) 10.2. Сумма кубов трех последовательных натуральных чисел оказалась кубом натурального числа. Докажите, что среднее из этих трех чисел делится на 4. (В. Сен- деров) 10.3. Петя раскрашивает 2006 точек, расположенных на окружности, в 17 цветов. Затем Коля проводит хорды с концами в отмеченных точках так, чтобы концы любой хорды были одноцветны и хорды не имели общих точек (в том числе и общих концов). При этом 92 XXXII Всероссийская математическая олимпиада школьников
Коля хочет провести как можно больше хорд, а Петя старается ему помешать. Какое наибольшее количество хорд заведомо сможет провести Коля? (С. Берлов) 10.4. Окружность со касается равных сторон АВ и АС равнобедренного треугольника ABC и пересекает сторону ВС в точках К и L. Отрезок АК пересекает со второй раз в точке М. Точки Р и Q симметричны точке К относительно точек В и С соответственно. Докажите, что описанная окружность треугольника PMQ касается окружности со. (В.Филимонов) 10.5. Пусть а19 а2, ..., а10 — натуральные числа, al<a2< < ... < а10. Пусть bk — наибольший делитель аА, такой, что bk < ak. Оказалось, что b1 > b2 > ... > Ью- Докажите, что а10 > 500. (М.Мурашкин) 10.6. На дугах АВ и ВС окружности, описанной около треугольника ABC, выбраны соответственно точки К и L так, что прямые KL и АС параллельны. Докажите, что центры вписанных окружностей треугольников АВК и CBL равноудалены от середины дуги ABC. (С. Берлов) 10.7. Дан квадратный трехчлен / (х) = х2 + ах + Ь. Уравнение / (/ (х)) = 0 имеет четыре различных действительных корня, сумма двух из которых равна -1. Докажите, что Ъ < --. (С. Берлов) 10.8. Квадрат 3000 х 3000 произвольным образом разбит на «доминошки» (т. е. прямоугольники 1x2 клетки). Докажите, что «доминошки» можно раскрасить в три цвета так, чтобы «доминошек» каждого цвета было поровну и у каждой «доминошки» было не более двух соседей ее цвета («доминошки» считаются соседними, если они содержат клетки, соседние по стороне). (А. Пастор) ▼ 11 класс 11.1. Докажите, что sin Vx < д/sin х при 0 < х < ^. (В. Сен- деров) 11.2. Сумма и произведение двух чисто периодических десятичных дробей — чисто периодические дроби с периодом Т. Докажите, что исходные дроби имеют периоды не больше Т. (А.Голованов) V ЭТАП. 2005/2006 учебный год 93
11.3- В клетчатом прямоугольнике 49 х 69 отмечены все 50 • 70 вершин клеток. Двое играют в следующую игру: каждым своим ходом каждый игрок соединяет две точки отрезком, при этом одна точка не может являться концом двух проведенных отрезков. Отрезки могут содержать общие точки. Отрезки проводятся до тех пор, пока точки не кончатся. Если после этого первый может выбрать на всех проведенных отрезках направления так, что сумма всех полученных векторов равна нулевому вектору, то он выигрывает, иначе выигрывает второй. Кто выигрывает при правильной игре? (О.Подлипский) 11.4. Биссектрисы ВВ1 и ССг треугольника ABC пересекаются в точке /. Прямая В1С1 пересекает описанную окружность треугольника ABC в точках М и N. Докажите, что радиус описанной окружности треугольника MIN вдвое больше радиуса описанной окружности треугольника ABC. (Л.Емельянов) 11-5- Последовательности положительных чисел (хп) и (уп) удовлетворяют условиям хп + 2 = хп + х\ +19 уп + 2 = = Уп + Уп + i ПРИ всех натуральных п. Докажите, что если все числа х19 х2, у и У г больше 1, то хп > уп при каком-нибудь натуральном п. (А.Голованов) 11.6- Окружность с центром J, вписанная в грань ABC треугольной пирамиды SABC, касается отрезков АВ9 ВС, С А в точках D9 E9 F соответственно. На отрезках SA, SB, SC отмечены соответственно точки А\ В\ С так, что АА' = AD, ВВ' = BE, СС = CF; S' — точка на описанной сфере пирамиды, диаметрально противоположная точке S. Известно, что SI является высотой пирамиды. Докажите, что точка S' равноудалена от точек А', Б', С". (Ф.Бахарев) 11-7. Известно, что многочлен (х + 1)п — 1 делится на некоторый многочлен Р (х) = xk + ck_xxk~1 + ck_2xk"2 + ... + + cxx + c0 четной степени ft, у которого все коэффициенты с0, си ..., ck_x — целые нечетные числа. Докажите, что п делится на ft + 1. (А.Гарбер) 11.8- В лагерь приехало несколько ребят, каждый из них имеет от 50 до 100 знакомых среди остальных. Докажите, что ребятам можно выдать бейсболки, покрашенные в 1331 цвет так, чтобы у знакомых каждого ребенка были бейсболки хотя бы 20 различных цветов. (Д.Карпов) 94 XXXII Всероссийская математическая олимпиада школьников
РЕШЕНИЯ ЗАДАЧ ▼« 9 класс 9.1. Ясно, что ломаная пересекает диагональ. Пусть А — одна из вершин ломаной, лежащая на диагонали. Будем двигаться по ломаной, пока не вернемся в вершину В, лежащую на диагонали. Из симметрии если двигаться по ломаной из вершины А в другую сторону, то вершина В также окажется первой вершиной на диагонали, в которую мы попадем. При этом ломаная уже замкнется, поэтому через остальные 13 центров клеток на диагонали ломаная не проходит. Раскрасим доску в шахматном порядке так, чтобы диагональ была черной. Заметим, что на нашей ломаной белые и черные клетки чередуются, поэтому их количества равны. В исходном же квадрате черных клеток на одну больше. Поскольку клетки диагонали черные и ломаная не проходит через 13 из них, то она не проходит и через 12 белых клеток. Итого, длина ломаной не более 152 — 13 — 12 = 200. 9.2. Рассмотрим какое-нибудь натуральное число п > 1 000 000. Покажем, что условию будет удовлетворять четверка чисел -n, n + 1, п (п + 1) + 1, п (п + 1) (п (п + 1) + 1) + 1. Действительно, применив трижды соотношение = — —, получаем а а + 1 а (а + 1) ~^п + п + 1 + п (п + 1) + 1 + п (п + 1) (п (л + 1) + 1) + 1 1.1. 1 п (п + 1) п (п + 1) + 1 п (п + 1) (п (п + 1) + 1) + 1 =+= п (п + 1) (п (п + 1) + 1) п (п + 1) (п (п + 1) + 1) + 1 = 1 п (п + 1) (п (л + 1) + 1) (п (п + 1) (п (п + 1) + 1) + 1)' что и требовалось. 9.3. Ответ. 117. Заметим, что 2006 = 17 • 118; поэтому найдутся 2 цвета, в которые покрашены в сумме не менее 2 • 118 = = 236 точек. Докажем индукцией по ft, что через 2ft - 1 точку двух цветов всегда можно провести ft — 1 непересекающую- V ЭТАП. 2005/2006 учебный год 95
ся хорду с одноцветными концами. База очевидна. Пусть k > 2. Тогда среди точек возьмем две одноцветных, стоящих подряд. Соединим их хордой, выбросим и применим предположение индукции к оставшимся точкам. Выбрав 235 точек двух цветов и применив данное утверждение, получаем, что 117 хорд Коля сможет провести всегда. Осталось привести пример, когда больше хорд провести нельзя. Пусть на окружности стоит 17ft точек. Пусть Петя покрасит каждую точку в цвет, соответствующий остатку от деления на 17 ее номера. Докажем индукцией по ft, что через эти точки можно провести не более ft — 1 хорды с выполнением условия. База очевидна, докажем переход. Пусть проведено некоторое количество хорд. Рассмотрев две соединенные точки А и Б на минимальном расстоянии друг от друга, получим такую хорду АБ, что на одной из дуг, на которые она делит окружность, нет концов других проведенных хорд. Теперь сотрем хорду АВ и уберем с окружности все точки этой дуги, включая один из концов хорды. Мы получили исходную раскраску 17/ точек при I < ft. Они соединены не более чем 1—1 хордой, поэтому изначально хорд было не больше I — 1 + l^ft — 1, что и требовалось. 9.4. Пусть М — вторая (отличная от А) точка пересечения со со стороной АС. Докажем, что четырехугольник ATLC — вписанный. Действительно, заметим, что при гомотетии с центром А, переводящей окружность со в описанную окружность треугольника ABC, прямая МК переходит в прямую СВ, а следовательно, они параллельны (рис. 31). Тогда получаем, что ZAMK = ZACB = ZACTy но из вписанности четырех- Рис. 31 96 XXXII Всероссийская математическая олимпиада школьников
угольника AMLK имеем ZAMK = ZALK = ZALT. Отсюда ZACT = ZALT, т. е. четырехугольник ATLC — вписанный. Следовательно, ZCTA = ZCLA, но по свойству касательной ZCLA = ZLKA, т. e. ZCTA = ZTKA, и, значит, ZBTA = ZBKT. Тогда треугольники ВТ А и ВКТ подобны по двум углам, откуда ВТ2 = ВК • ВА. С другой стороны, произведение В К • В А равно квадрату касательной к со из точки В. 9.5. Заметим, что bk = —, где ck — наименьший простой ck делитель ak. Так как Ъ9 > fc10, то Ъ9 > 1 и bQ ^ с9. Отсюда а10 > а9 > с\. Но из неравенств at< ai + u bt> bi + 1 следует, что ct < ci + l9 т. e. сг < c2< ... < c10. Значит, c9 ^ 23, так как 23 — девятое по счету простое число. Поэтому а10 > с% > 529 > 500. 9.6, Сначала заметим, что поскольку ZARP < < ZARP + ZQRC = ZABC, то точка X лежит на луче RP (иначе, если точки X и Р лежат по разные стороны от точки i?, то ZARP - п - ZARX > ZRAX = = ZABC) (рис. 32). Тогда ZACB = ZXAB и ZAPX = = ZRPB = ZRQC, и треугольники АРХ и CQR равны по стороне и двум углам. Следовательно, РХ = QR. Рис. 32 Аналогично PR = QY, откуда и следует утверждение задачи. 9.7. Ответ. Выигрывает игрок, делающий второй ход. Приведем выигрышную стратегию для второго. Первыми несколькими ходами он склеивает каждую V ЭТАП. 2005/2006 учебный год 97
клетку, примыкающую к границе квадрата со всеми ее соседями. На это потребуется не более 8 • 99 ходов, т. е. после этого будет склеено всего 16 • 99 пар сторон и, как следствие, не более 2 - 16 • 99 < —-— «до- миношек» окажутся склеенными с другими «доминошками». Следовательно, после этого еще останутся отдельные «доминошки», и фигура не будет связной. Далее второй будет действовать произвольным образом, следя только за тем, чтобы не проиграть своим очередным ходом. Пусть в некоторый момент ему не удастся сделать этого. Тогда все «доминошки» распадаются на две связные фигуры, причем все несклеенные отрезки — это граница между этими фигурами, так как любой другой отрезок можно склеить. При этом одна из этих фигур содержит все граничные клетки квадрата. В границе внутренней фигуры четное число отрезков (если мы обойдем эту ломаную из отрезков, то отрезков, по которым мы шли вверх и вниз, будет поровну; то же с отрезками вправо и влево). Подсчитаем изначальное число разрезанных сторон отрезков. Оно равно суммарному периметру всех «доминошек», уменьшенному на периметр квадрата и деленному на 2 (так как каждый из остальных отрезков считал- 6 • 5000 - 400 ся по два раза), т. е. - четному числу. Значит, к данному моменту склеено также четное число сторон, и ходить должен первый. Противоречие. 9.8. Обозначим через сх и с2 корни уравнения / (х) = 0, а через хги х2 — корни уравнения / (/ (х)) = 0, сумма которых равна —1. Множество корней последнего уравнения совпадает с объединением множеств корней уравнений / (х) = сги f (x) = с2. Если х1 и х2 являются корнями одного из последних двух уравнений, то их сумма, равная —1, будет по теореме Виета равна и —а, откуда а = 1. Можно считать, что cl^c2. Но поскольку по теореме Виета сх + с2 = -1, то с2^--. Из условия следует, что дискриминант уравнения / (х) = с2 неотрицателен, поэтому 1 - 4Ь + 4с2 > 0, откуда Ь ^ --. 98 XXXII Всероссийская математическая олимпиада школьников
В противном случае, не умаляя общности, можно записать, что х\ + ахг + Ь = сх и х\ + ах2 + b = с2. Складывая последние два равенства, получим х\ + х\ + а (хх + х2) + 2Ъ = сх + с2. (*) Поскольку с1 + с2 = —а по теореме Виета, а х1 + х2 = = -1 по условию, то равенство (*) после сокращения перепишется так: х\ + х\ + 2Ь = О. Но тогда ▼ 10 класс 10.1- См. решение задачи 9.1. 10.2. В решении латинскими буквами везде обозначены натуральные числа. По условию (х - I)3 + х3 + (х + I)3 = z/3, или Зх (х2 + 2) = г/3. Тогда у делится на 3, у = 3z и х (х2 + 2) = 9z3. Очевидно, НОД (х9 х2 + 2) ^ 2. Докажем, что случай НОД (х, х2 + 2) = 1 невозможен. Действительно, в этом случае либо 1) х = 9иг и х2 + 2 = и3, либо 2) х = и3, х2 + 2 = 9v3 при некоторых натуральных и, v. В случае 1) получаем 81и6 + 2 = и3, что невозможно, так как куб целого числа при делении на 9 дает остаток 0 или ± 1. Аналогично из равенства 2) следует, что и6 + 2 = 9и3, что невозможно по тем же причинам. Итак, НОД (х9 х2 + 2) = 2, х (х2 + 2) = 9v3. Тогда х (и, следовательно, z) четно, поэтому х (х2 + 2) делится на 8. Поскольку х2 + 2 не делится на 4, получаем, что х делится на 4, что и требовалось доказать. 10.3. Ответ. 117. См. решение задачи 9.3. 10.4. Обозначим через D и Е точки касания со со сторонами АВ и AC, DE || ВС из симметрии относительно биссектрисы угла ВАС (рис. 33 на с. 100). Пусть при гомотетии с центром А и коэффициентом ——: окруж- лм ность со переходит в окружность со'. Окружность со' проходит через точку К, а следовательно, и через L (из симметрии относительно биссектрисы угла ВАС), а также со' касается лучей АВ и АС в некоторых точках D' и Е'. Из гомотетии следует, что MD || KD'. Далее, по теоре- V ЭТАП. 2005/2006 учебный год 99
Рис. 33 ме о произведении отрезков касательных BD2 = = ВК • BL = BD'2, откуда BD = BD'. По построению ВК = ВР, поэтому DKD'P — параллелограмм, и значит, PD || KD'. Отсюда вытекает, что точки М, D, Р лежат на одной прямой. Аналогично М, Е и Q лежат на одной прямой. Треугольники MDE и MPQ гомотетичны с центром М, следовательно, их описанные окружности также гомотетичны, т. е. касаются в точке М. 10.5. См. решение задачи 9.5. 10.6. Если АВ = ВС у то утверждение задачи очевидно. Пусть для определенности АВ < ВС (рис. 34). Обозначим через 119 12 центры вписанных окружностей треугольников АКБ и CLB соответственно, через Р, Q — вторые (отличные от Б) точки пересечения прямых В1и В12 с описанной окружностью треугольни- Рис. 34 100 XXXII Всероссийская математическая олимпиада школьников
ка ABC, а через R — середину дуги ABC этой окружности. Тогда РА = QC как хорды, стягивающие половины равных дуг. Так как ZPAI1 = ZPAK + ZKAI1 = = ZPBK + ZBAIX = ZABIX + ZBAIX = ZAI^P, то треугольник AIXP равнобедренный и РА = Р1г. Аналогично QC = QI2j следовательно, PIX = QI2. Далее, PR = QR как хорды, стягивающие равные дуги, a ZI2QR = ZIXPR как углы, опирающиеся на одну дугу. Тогда треугольники RIXP и RI2Q равны по двум сторонам и углу между ними, откуда и следует утверждение задачи. 10.7. См. решение задачи 9.8. 10.8, Раскрасим клетки квадрата в черный и белый цвет в шахматном порядке- Каждая «доминошка» покрывает ровно одну черную клетку. Разобьем квадрат 3000 х 3000 на квадраты 6 х 6 и в каждом квадрате перекрасим черные клетки в 3 цвета (они обозначены разными цифрами на рисунках 35 или 36), Окрасим «доминошки» в первый, второй и третий цвета так, чтобы «доминошка» накрывала клетку своего цвета. В квадрате 6x6 поровну клеток каждого цвета, значит, и в квадрате 3000 х 3000 тоже. Следовательно «доминошек» каждого цвета поровну. Пусть две клетки одного цвета покрыты соседними «доминошками». Легко видеть, что для наших раскрасок это возможно, только если эти две клетки имеют общую вершину. Для данной клетки на рисунке 36 имеется Рис. 35 1 2 3 1 2 3 1 2 3 1 2 3 2 3 1 2 3 1 2 3 1 2 3 1 3 1 2 3 1 2 3 1 2 3 1 2 1 2 3 1 2 3 1 2 3 1 2 3 2 3 1 2 3 1 2 3 1 2 3 1 3 1 2 3 1 2 3 1 2 3 1 2 V ЭТАП. 2005/2006 учебный год 101
1 3 2 1 3 2 1 3 2 1 3 2 2 1 2 2 1 3 2 1 2 2 1 3 3 2 1 3 2 1 3 2 1 3 2 1 1 3 2 1 3 2 1 3 2 1 3 2 2 1 3 2 1 3 2 1 3 2 1 3 3 2 1 3 2 1 3 2 1 3 2 1 Рис. 36 не более двух клеток того же цвета, имеющих с ней общую вершину, а на рисунке 35 — не более одной такой клетки. Отсюда следует, что если использовать раскраску рисунка 36, то для любой «доминошки» имеется не более двух соседних «доминошек» того же цвета, а если использовать раскраску рисунка 35, то даже не более одной соседней «доминошки» того же цвета. ▼ 11 класс 11.1. При х ^ 1 имеем 1 ^ 4х ^ х < ^. Отсюда sin4x ^ sinx. Далее, поскольку О < sin x < 1, имеем sin x < д/sin х. Пусть О < х < 1. Перепишем неравенство: sin2 t < sin (t2) при О < t < 1. Так как sin2 О = sin (О2), то достаточно доказать (sin2 t)' < (sin (t2))', или 2 sin t cos t < < 2t cos (t2). Поскольку ^ > t > t2 > О, то cos t < cos (t2). Перемножив это неравенство и sin t < t, получим sin t cos t < t cos (t2). 11.2. Заметим, что чисто периодическая дробь с периодом Т после домножения на 10т - 1 становится целым числом. Домножим две данные дроби а и Ъ на число 10т - 1 = 99... 9. Получатся два новых ра- т циональных числа А = (10т - 1) а и В = (10т - 1) Ь. 102 XXXII Всероссийская математическая олимпиада школьников
Числа А + В = (1ОТ - 1) (а + Ъ) и АВ = (1ОТ - I)2 аЪ целые, так как а + Ь и аЪ — дроби с периодом Т, становящиеся целыми при домножении на 10т - 1 и тем более на (10т - I)2. Но два рациональных числа, сумма и произведение которых целые, являются корнями приведенного квадратного уравнения с целыми коэффициентами, т. е. сами являются целыми числами. Значит, а и Ъ могут быть записаны в виде обыкновенных дробей со знаменателем 10т - 1, откуда и следует утверждение задачи. 11.3. Ответ. Выигрывает первый. Будем считать, что большая сторона прямоугольника параллельна оси Ох, а меньшая — оси Оу, при этом левый нижний угол прямоугольника совпадает с началом координат. Лемма 1. Пусть игроки провели все отрезки. Тогда независимо от расстановки стрелок проекция вектора суммы на каждую из осей будет иметь четную длину. Доказательство. Рассмотрим проекцию вектора суммы на ось Ох. Для каждой точки в зависимости от направления вектора ее координата по оси Ох берется либо со знаком плюс, либо со знаком минус. Сумма координат (с соответствующими знаками) по оси Ох всех точек прямоугольника и даст проекцию вектора суммы на ось Ох. Однако пятьдесят из этих точек имеют координату О, пятьдесят имеют координату 1, ..., пятьдесят имеют координату 69. То есть среди этих чисел четное количество нечетных. Это и означает, что соответствующая сумма будет четной. Аналогично проекция вектора суммы на ось Оу будет иметь четную длину. Лемма доказана. ■ Лемма 2. Пусть имеется набор отрезков с концами в целочисленных точках прямоугольника 49 х 69. Тогда можно так выбрать направления на этих отрезках, что проекция вектора суммы на ось Ох будет меньше 140, а на ось Оу — меньше 100. Доказательство. Разобьем отрезки набора на четыре группы: параллельных оси Ох (группа А), параллельных оси Оу (группа Б), тех, у которых правый конец выше левого (группа С), и тех, у которых правый конец ниже левого (группа D). Заметим, что если взять два отрезка из одной группы, то на них так можно выбрать направления, что по модулю координаты V ЭТАП. 2005/2006 учебный год 103
вектора их суммы будут не больше 69 по оси Ох и не более 49 по оси Оу (назовем такой вектор коротким). Также заметим, что если у набора векторов все направления поменять на противоположные, то вектор суммы изменит лишь знак. Будем теперь проводить следующую процедуру. На каждом шаге будем выбирать пару отрезков из одной группы и заменять их соответствующим отрезком, который соответствует короткому вектору (новый отрезок может попасть в другую группу). Если в процессе получится отрезок нулевой длины, выкинем его. Заметим тогда, что если в полученном наборе мы можем расставить направления требуемым образом, то и в старом это было возможно. Через некоторое число шагов мы придем к такой ситуации, что в каждой группе будет не более одного отрезка. Выбрав направления на отрезках из групп С и £>, мы получим вектор, проекция которого на ось Ох будет меньше 140, а на ось Оу — меньше 100. Пусть обе его координаты неотрицательны. Если теперь на отрезках из групп А л В выбрать отрицательные направления, то у итогового вектора суммы проекция на ось Ох будет меньше 140, а на ось Оу — меньше 100. Лемма доказана. ■ Перейдем к решению задачи. Опишем стратегию первого игрока. Ему необходимо будет провести 140 горизонтальных отрезков длины 1 и 100 вертикальных отрезков длины 1 (назовем их красными). Тогда, после того как будут проведены все отрезки, на всех не красных отрезках по лемме 2 он так может выбрать направления, что проекция вектора суммы на ось Ох будет меньше 140, а на Оу — меньше 100. Теперь ему останется выбрать направления на красных отрезках так, чтобы проекция вектора суммы на каждую из осей была равна нулю. Поскольку по лемме 1 проекция вектора суммы на каждую из осей будет иметь четную длину, он сможет это сделать и выиграет. Покажем, как первому игроку провести требуемое количество красных отрезков. Выделим 25 - 35 = 875 квадратиков, каждый из которых содержит четыре отрезка длины 1 (рис. 37). Каждым сво- Рис. 37 XXXII Всероссийская математическая олимпиада школьников
им ходом первый будет отмечать вертикальный или горизонтальный отрезок в одном из квадратиков. Для этого ему потребуется 240 ходов, каждым из которых первый «тратит» один квадратик. Заметим, что второй каждым своим ходом «портит» не более двух квадратиков. Таким образом, после каждого хода первого и второго используется не более трех квадратиков. Поскольку 240 - 3 = 720 < 875, первый сможет провести 240 красных отрезков нужных направлений, что и требовалось. 11.4. Пусть биссектрисы А19 BJ, CI пересекают описанную окружность в точках Ло, Во и Со соответственно. Точки Бо и Со являются соответственно серединами дуг АС и АВ. Проведем через точку А прямую, параллельную В0С0, пересекающую биссектрисы в точках 1В и 1С (рис. 38). Имеем ZAIB0 = ZABI + ZBAI = = ZABB0 + ZBAA0 = ZBqBC + ZCAA0 = ZB0AI, поэтому треугольник B0AI равнобедренный (В0А = BI). Рис. 38 Аналогично С0А = CI, поэтому треугольники В0АС0 и BqICq равны. Далее, отрезок В0С0 является серединным перпендикуляром к AI, a AI — высота в треугольнике IIBIC. Отсюда следует, что В0С0 — средняя линия треугольника IBIIC- Получаем следующие равенства для радиусов описанных окружностей: R (1ВПС) = 2R (BoICo) = 2R (В0АС0) = 2R (ABC). Теперь достаточно доказать, что точки М и N лежат на описанной окружности треугольника IBIIC' Заметим, что ZAIBI = ZCqBqI = ZCqB0A = ZC0CA = ZICA, V ЭТАП. 2005/2006 учебный год 105
значит, точки А, J, С, 1В лежат на одной окружности, отсюда ВХА • ВХС = BJ • ВХ1В. С другой стороны, ВХА • ВХС = ВХМ • BtN, так как точки А, М, С, ЛГ лежат на одной окружности. Следовательно, ВХМ • BXN = = BJ - В^в, и точка 1В лежит на описанной окружности треугольника IMN. Аналогично на ней лежит точка Jc, что и требовалось. 11.5. Очевидно, начиная со второго члена, наши последовательности возрастают: хп + 2> х2п + 1> xn + lJ yn + 2>yn + i- Так как xs > 1 + I2 = 2, ys > I2 + 1 = 2, все члены каждой из последовательностей, начиная с третьего, больше 2. Аналогично при п > 3 получим хп > 3, уп > 3. Заметим теперь, что С другой стороны, уп + < Зг/п < у\ при п > 3. Итак, при п > 3 имеем *"п + 2 " — »#2 > х\ при п > 1. — j#2 + 2 */п-1 а^^> > 77 При достаточно большом k правая часть последнего неравенства больше 1, а значит, X2k > У2k-» что и требовалось доказать. 11.6. Из теоремы о трех перпендикулярах следует, что SD — высота в грани SAB. Так как SS' — диаметр окружности, проходящей через точки S, S' и А, то ZSAS' = 90° (рис. 39). Обозначив через R и г соответственно радиусы описанной сферы пирамиды и вписанной окружности треугольника АВС9 имеем Рис. 39 106 XXXII Всероссийская математическая олимпиада школьников
S'A'2 = S'A2 + AA'2 = (SS'2 - SA2) + AD2 = = SS'2 - (SA2 - AD2) = SS'2 - SD2 = SS'2 - (SI2 + ID2) = = (2R)2 - SI2 - г2. Аналогично, вычисляя S'B' и S'C, получаем, что S'A' = S'B' = S'C = J(2R)2 - SI2 - r2 . 11.7. Перепишем условие задачи в виде равенства (х + 1)п - 1 = Р (х) Q (х), где Р (х) — многочлен с нечетными коэффициентами. Будем называть два многочлена / (х) и g (x) похожими и обозначать f (x) = g (x), если коэффициенты при одинаковых степенях у многочленов / (х) и g (x) имеют одинаковую четность. Тогда если в верном равенстве мы заменим некоторые коэффициенты одного или нескольких многочленов на их остатки по модулю 2, то мы получим два похожих многочлена. Следовательно, (х + 1)п - 1 = (xk + xk~1 + ... + 1) Q (x). (1) Заменив в последнем равенстве переменную х на — и, домножив обе части нахл, получаем (у _1- 1 \п уп = (yk 4- yk ~ 1 4- _l_ 1 ^ Уп ~ k/~} I _zl I O\ Л> i л. у А> — \А> i A> i ... i JL j A> 4qf I I. \^/ При этом xn~kQ — — это некоторый многочлен степени, не превосходящей п — ft, от переменной х. Вычитая из (1) формулу (2), имеем хп - 1 = (xk + xk~l + ... + 1) R(x) для некоторого многочлена R (х). Пусть п не делится на k + 1, тогда п = q (k + 1) + г, 0<r<fc + l. Тогда многочлен хп - xr = xr (xq (/г +1} - 1) делится на xk + 1 - 1 = (xk + ... + 1) (х - 1), а значит, хг - 1 = = (хп - 1) - (хп - xr) = (xk + ... + 1) i?! (jc) для некоторого многочлена i?x (x). Это невозможно, ибо степень многочлена хг — 1 не больше степени многочлена xk + ... + 1, и они непохожи. 11.8. В решении мы будем пользоваться следующей известной теоремой. Теорема Холла. Пусть дан двудольный граф G (т. е. его вершины разбиты на два подмножества А и В, таких, что любое ребро соединяет вершины из разных подмножеств). Предположим, что для любого V ЭТАП. 2005/2006 учебный год 107
подмножества вершин A1<zzA количество вершин в Аг не больше, чем количество вершин, соединенных хотя бы с одной вершиной из At. Тогда в графе найдется паросочетание (т.е. набор ребер с различными концами), содержащее все вершины множества А. Перейдем к решению задачи. Построим граф, вершины которого соответствуют ребятам, а ребра — знакомствам. Степени вершин этого графа не менее 50 и не более 100. Докажем вспомогательное утверждение. Лемма 1. Пусть k ^ п < т — натуральные числа. Тогда из графа, степени вершин которого не менее п и не более т, можно удалить несколько ребер так, чтобы степени всех вершин стали не менее п - k и не более т - k. Доказательство. Понятно, что достаточно доказать утверждение леммы для k = 1. До тех пор, пока есть ребра, соединяющие пары вершин степени т, будем удалять такие ребра. Пусть таких ребер больше нет, обозначим через А множество всех вершин степени т в полученном после удаления ребер графе G, а через В множество всех остальных вершин. Рассмотрим двудольный граф G' на тех же вершинах, в котором останутся лишь ребра между А и В. Проверим выполнение условия теоремы Холла для этого графа. Рассмотрим множество Ах а А, пусть Вх — множество вершин, смежных с вершинами из Аг. Из Ах выходит не менее т \ Ах | ребер к вершинам множества В19 а в каждую вершину из Вх входит менее т ребер, следовательно, | Вх | ^ | Ах | (через | X \ мы, как обычно, обозначаем количество элементов в множестве X). Таким образом, по теореме Холла существует паросочетание, содержащее все вершины из А. Удалив из графа G ребра этого паросочетания, мы получим граф Gl9 степени вершин которого не менее п — 1 и не более т - 1. Лемма 1 доказана. ■ Перейдем к решению задачи. Применив лемму 1 для исходного графа и k = 30, мы получим граф if, степени вершин которого не менее 20 и не более 70. Сделаем его ребра красными. Для каждой вершины этого графа отметим 20 вершин среди ее соседей и попарно соединим эти 20 вершин зелеными ребрами. Так как 108 XXXII Всероссийская математическая олимпиада школьников
из каждой вершины выходит не более 70 красных ребер, то из нее выходит не более чем 70 ■ 19 = 1330 зеленых ребер. Рассмотрим граф Н' с зелеными ребрами на вершинах графа Н. Несложно по очереди покрасить эти вершины в 1331 цвет так, чтобы соседние вершины были разноцветными: рассматривая каждую следующую вершину, покрасим ее в любой не использованный среди ее соседей цвет. Теперь опять рассмотрим граф Н с красными ребрами. Среди соседей каждой его вершины есть 20 выделенных, и все они покрашены в разные цвета. Замечание. Можно покрасить бейсболки ребят всего в 761 цвет. (Это наблюдение принадлежит И. Богданову.) Для доказательства этого факта заменим лемму 1 на более сильную лемму 2, доказательство которой предоставляется читателю. Лемма 2. Пусть k < п — натуральные числа. В графе G степени всех вершин не менее п и не более 2п. Тогда можно удалить несколько ребер так, чтобы степени всех вершин стали не менее k и не более 2k.
ГЛАВА 5 (Олимпиада 2006/2007 учебного года II ЭТАП (районный) УСЛОВИЯ ЗАДАЧ* у) 6 класс 6.1. Пять шестиклассников на городской олимпиаде по математике в сумме решили 20 задач, причем один из них решил в два раза больше задач, чем другой. А сколько задач решил каждый из шестиклассников? Объясните свой ответ. (На олимпиаде было 5 задач). 6.2. Закрасьте в квадрате 7x7 четыре фигурки вида, изображенного на рисунке 40, состоящие из 4 клеток, так, чтобы в любом квадрате 2x2 была закрашена хотя бы одна клетка. Рис. 40 6.3. Петя в течение одного часа дважды сфотографировал электронные часы, показывающие время от 00:00:00 до 23:59:59. Покажите, что на этих двух фотографиях могли оказаться все цифры от 0 до 9. * Авторы задач и составители: Н. X. Агаханов, О. К. Подлипский, Б. В. Трушин. Задача 8.1 предложена И. С. Рубановым, 10.4 — М. В. Му- рашкиным. 110 XXXIII Всероссийская математическая олимпиада школьников
6.4. Шестиклассники Школы Сладкоежек собирают конфетные фантики трех цветов: зеленого, синего и красного — и обмениваются ими по правилам: либо меняют 3 синих фантика на 5 зеленых (и наоборот, 5 зеленых на 3 синих), либо 7 красных фантиков на 11 синих (и наоборот, 11 синих на 7 красных). Могло ли у ребят в конце месяца оказаться 1111 фантиков, если в начале месяца у них было 1000 фантиков? 6.5. Коля выложил на столе из цифр пятизначное число N9 а затем еще четыре числа: сумму первых двух цифр числа N, сумму первых трех, первых четырех, наконец — сумму всех пяти цифр числа N. В итоге на столе оказались: одна цифра 1, шесть цифр 2, одна цифра 4, три цифры 6, две цифры 8. Чему равно число N? (Объясните свой ответ). ▼> 7 класс 7.1. К бабушке в гости приехали 11 внуков — все дети двух ее дочерей. Одна из внучек сказала: «Здесь у меня в два раза больше сестричек, чем дома», а другая ответила: «А у меня здесь в три раза больше сестричек, чем дома». Сколько внуков и внучек у бабушки? 7.2. Петя с интервалом в целое число часов дважды сфотографировал электронные часы, показывающие время от 00:00:00 до 23:59:59. Оказалось, что на второй фотографии цифры идут в обратном порядке по сравнению с первой. Какое время могло пройти между снимками? (Объясните свой ответ). 7.3. Детский конструктор состоит из квадратов 2 х 2 и равнобедренных прямоугольных треугольников с гипотенузой 3. Какое наибольшее количество «домиков» (рис. 41) можно собрать из деталей конструктора, уложенных в один слой (без наложений) в коробку 7x7? Рис. 41 II ЭТАП. 2006/2007 учебный год 111
7.4. Петя и Коля копили монеты достоинством в 1, 2, 5 рублей, причем оказалось, что в Петиной копилке нет монет того же достоинства, что в Колиной. Могут ли ребята заплатить по 2006 рублей из своих копилок одинаковым числом монет? 7.5. Коля составил из различных цифр, отличных от нуля, пятизначное число А и прибавил к нему число, получаемое из А перестановкой цифр в порядке убывания. У него получилась сумма 171 540. Затем он прибавил к А число, получаемое из А перестановкой цифр в порядке возрастания, и у него получилась сумма 85 608. Какое число составил Коля? 0 8 класс 8.1. Когда одно из двух целых чисел увеличили в 2006 раз, а другое уменьшили в 2006 раз, их сумма не изменилась. Докажите, что эта сумма делится на 2007. 8.2. На доске написаны числа 1, 2, 3, ..., 10. Разрешается выбрать любое нецелое число t и ко всем числам, меньшим t, прибавить 1, а из всех чисел, больших t, вычесть 1. Можно ли несколькими такими операциями получить только единички и четверки? 8.3. На доске выписаны числа V2 и V5. Разрешается дописать на доску сумму, разность или произведение любых двух различных чисел, уже выписанных на доске. Докажите, что можно написать на доске число 1. 8.4. Петя с интервалом в 1000 секунд дважды сфотографировал электронные часы, показывающие время от 00:00:00 до 23:59:59. Могли ли на двух фотографиях оказаться все цифры от 0 до 9? 8.5. Докажите, что любой прямоугольный треугольник с гипотенузой 4с можно закрыть тремя одинаковыми кругами радиуса с. ▼ 9 класс 9.1. В ряд выписаны в порядке возрастания все простые числа. Может ли сумма шести подряд идущих чисел в этом ряду равняться сумме пяти подряд идущих чисел в этом ряду? (Наборы чисел могут пересекаться). 9.2. Верно ли, что если квадратные уравнения х2 + ах + Ь = 0 и х2 + сх + d = 0 не имеют корней, то и уравнение 2 а + с Ъ + d дг + х + —— = 0 также не имеет корней? 112 XXXIII Всероссийская математическая олимпиада школьников
9.3. Компьютерная программа преобразует набор из натуральных чисел по следующему правилу: каждое четное число делится на два, а из каждого нечетного числа вычитается единица. Докажите, что если начальный набор состоял из пяти последовательных натуральных чисел, то после двух преобразований в нем появятся хотя бы два равных числа. 9.4. Пусть Р и Q — проекции точки Н, лежащей на стороне АС остроугольного треугольника АВС9 соответственно на стороны АВ и СВ. Докажите, что если точки А, Р, Q, С лежат на одной окружности, то ВН — высота треугольника ABC. 9.5. В каждой клетке доски 5 х 10 стоит по одной шашке. За один ход можно выбрать какие-то 2 шашки, и каждую из них подвинуть на соседнюю по стороне клетку. Если на какой-то клетке оказалось хотя бы две шашки, то с этой клетки можно снять ровно две из этих шашек. Можно ли при помощи таких операций снять с доски все шашки? ▼ 10 класс 10.1. На доске написали квадратный трехчлен с положительным старшим коэффициентом. Каждую минуту на доске дописывают квадратный трехчлен, причем у каждого следующего трехчлена все три коэффициента на 1 больше соответствующих коэффициентов предыдущего. Докажите, что когда-нибудь на доске появится трехчлен, не имеющий корней. 10.2. Все магистры трех Тайных Орденов собрались на встречу. После встречи магистр первого Ордена сказал: «Теперь я знаю в два раза больше магистров, чем вчера». Магистр второго Ордена сказал: «Теперь я знаю в три раза больше магистров, чем вчера». Магистр третьего Ордена сказал: «Теперь я знаю в четыре раза больше магистров, чем вчера». Докажите, что кто-то из магистров обсчитался. Предполагается, что до встречи каждый магистр знал магистров только из своего Ордена, а после — всех трех орденов. 10.3. Существует ли такое число х, что все три числа 2х - Jx2 + 2, V*2 +2 - Jx2 + 2006 и Jx2 + 2006 - х являются целыми? II ЭТАП. 2006/2007 учебный год 113
10.4. Серединный перпендикуляр к стороне ВС треугольника ABC пересекает сторону АС в точке D, а биссектрису угла ABD — в точке К. Докажите, что точки А у Б, С, К лежат на одной окружности. 10.5. Могут ли для остроугольного треугольника с углами а, р, у — одновременно выполняться неравенства sin a < sin (2P), sin Р < sin (2у), sin 7 < sin (2а)? Jf) 11 класс 11.1. Когда одно из двух натуральных чисел возвели в квадрат, а из другого извлекли корень, их сумма не изменилась. Докажите, что эта сумма четна. 11.2. Существует ли такое число ху что все три числа х , - и — 2х являются целыми? X X Х2 + 1 Х2 + 1 11.3. Произведение производных двух квадратных трехчленов при всех значениях переменной больше суммы этих трехчленов. Докажите, что хотя бы один из трехчленов будет принимать и отрицательные значения. 11.4. Восемь бегунов одновременно стартуют в одном направлении из разных точек беговой дорожки и, пробежав с постоянной скоростью ровно один круг, каждый из них останавливается. (Скорости бегунов могут быть различными). Каждый раз, когда бегун обгоняет бегущего или стоящего бегуна, ему начисляется один балл, а каждый раз, когда его (бегущего или стоящего) обгоняет другой бегун, с него снимается один балл. Какое наибольшее число баллов мог заработать бегун после финиша всех участников? 11.5. Докажите, что если ос, р, у — плоские углы трехгранного угла (углы между его ребрами), то Y Р .а sin£ + sin-> sin-. РЕШЕНИЯ ЗАДАЧ 0 6 класс 6.1. Ответ. 2, 4, 4, 5, 5. Пусть Вася решил в два раза больше задач, чем Петя. Остальные трое ребят вместе решили не больше 3*5 = 15 задач, так как каждый может решить не бо- 114 XXXIII Всероссийская математическая олимпиада школьников
6.2. Ответ. 6.3. Ответ. Например, Рис. 42 лее 5 задач. Значит, Петя, и Вася вместе решили не меньше 5 задач. Если Петя решил 1 задачу (или меньше), то Вася решил 2 (или меньше), и вместе они решили 3 задачи (или меньше). Значит, этот случай не подходит. Если Петя решил 3 задачи (или больше), то Вася должен был решить 6 задач (или больше). Но на олимпиаде только 5 задач. Поэтому Петя решил ровно 2 задачи, а Вася — 4 задачи. Остальные трое ребят вместе решили 20-4 — 2 = 14 задач. Это возможно, только если один из них решил 4 задачи, а остальные двое — по 5 задач. Замечание. За ответ без объяснения — 4 балла. Не рассмотрен случай 1—2 задачи — снять 2 балла. См. рисунок 42. 07:18:29 и 07:34:56. Замечание. Любой правильный пример — 7 баллов. 6.4. Ответ. Не могло. Заметим, что после каждого обмена количество фантиков у ребят из 6А класса изменяется на четное число (либо на 2, либо на 4). Но так как у них изначально было ровно 1000 фантиков — четное число, то после каждого обмена общее количество фантиков у ребят должно оставаться четным числом. А число 1111 — нечетное. Замечание. За ответ без объяснения — 0 баллов. 6.5. Ответ. 88 622. Количество цифр, выложенных на столе, равно 13. Суммы — не более чем двузначные числа, поэтому они вместе могли дать не более 6-и цифр. Следовательно, все четыре суммы — двузначные числа. При прибавлении цифры нельзя перескочить через десятичный разряд, поэтому ровно одна из сумм (первая) начинается с 1, а остальные суммы начинаются с 2. Но 8 + 8 + 4 = 20, а цифра 0 отсутствует. Значит, первые три цифры числа N — это 8, 8 и 6. Значит, вторая сумма — 22 (6 + 8 + 8), третья — 24, четвертая — 26, так как третья и четвертая суммы начинаются с 2, и для сумм остались цифры 2, 4 и 6. II ЭТАП. 2006/2007 учебный год 115
Цифра 4 на столе одна, поэтому число N не может начинаться с 86 или 68, Поэтому N = 88 622 (2 = 24 - 22, 2 = 26 - 24). Замечание. Отдельные продвижения (баллы за продвижения суммируются): Доказано, что все суммы — двузначные числа, начинающиеся с 1 и 2, — 1 балл. Получено, что первая сумма — 14 или 16, — 1 балл. Доказано, что первые три цифры числа — 8, 8 и 6, — 2 балла. Только ответ — 3 балла. 7 класс 7.1. Ответ. 4 внука и 7 внучек. Количество сестер у первой девочки делится на 2, а у второй девочки делится на 3. Значит, если от количества внучек отнять единицу, то получится число, делящееся на 2 • 3 = 6. Но от 1 до 10 только 6 является таким числом. Значит, внучек 7, а внуков 4. Замечание. За ответ без объяснения — 4 балла. 7.2. Ответ. Кратное 24 часам. Заметим, что через целое число часов ни минуты, ни секунды не изменятся (если было ху : аЪ : cd9 то стало dc : ab : cd ). С другой стороны, должно получиться по условию dc : Ъа : ух. То есть ух совпадает с cd, а тогда ху совпадает с dc. Значит, время на фотографиях одинаковое. Следовательно, прошло время, кратное 24 часам. Замечание. За ответ без объяснения — 0 баллов. За ответ «24 часа» с объяснением — 7 баллов. 7.3. Рисунок 43 показывает, что детали для 8 «домиков» в коробку не помещаются (это также следует из того, что суммарная площадь 8 «домиков» равна 50). 8 то же время детали для 7 «домиков» уложить в коробку можно: достаточно на рисунке 43 убрать один из перекрывающихся квадратиков 2 х 2 и одну «кры- ШУ»- Рис-43 \ / х \ ч х / \ 116 XXXIII Всероссийская математическая олимпиада школьников
Замечание. За ответ с примером без объяснения максимальности — 3 балла. За ответ с объяснением максимальности без примера — 3 балла. 7.4. Ответ. Не могут. Предположим, что ребята могут заплатить по 2006 рублей из своих копилок одинаковым числом монет. Тогда у кого-то из ребят будут монеты только одного достоинства (только в 1, 2 или 5 рублей), иначе будет противоречие с условием. Если у кого-то в копилке есть монеты достоинством в 5 рублей, то у него должны быть и монеты другого достоинства, поскольку 2006 на 5 не делится. Также не может быть варианта, что у кого-то есть монеты достоинством в 1 рубль. В этом случае, чтобы набрать 2006 рублей, потребуется 2006 монет для одного из мальчиков. А для другого мальчика 2006 монет другого достоинства в сумме составит больше 2006 рублей. Значит, у кого-то (для определенности у Пети) в копилке только монеты достоинством в 2 рубля. Тогда у Коли в копилке должны быть монеты достоинством и в 5 рублей, и в 1 рубль. Чтобы набрать 2006 рублей, Пете потребуется ровно 1003 монеты. Однако Коля не сможет набрать 2006 рублей при помощи 1003 монет достоинством в 1 или в 5 рублей, поскольку нечетное количество (1003) нечетных чисел (1 или 5) в сумме дают нечетное число, а 2006 — четное. Замечание. За ответ без объяснения — 0 баллов. Указано, что у одного монеты достоинством в 1 и 5 рублей, а у другого — в 2 рубля, без объяснения получения — 1 балл, с объяснениями — 3 балла. 7.5. Ответ. 72 819. Пусть ABCDE — число, полученное из исходного перестановкой цифр в порядке убывания. Тогда ABCDE - EDCBA = 171 540 - 85 608 = 85 932. Цифра 8 разности может быть получена, только если А = 9, откуда Е = 1 (Е Ф 0). Мы получили ребус 9BCD1 - 1DCB9 = 85 932, следовательно, BCD1 - - DCB9 = 5932. При вычитании мы занимали одну единицу у числа Б, поскольку CD1 < СБ9, поэтому В - D = 6. Но Б < 8, D>2 (цифры 1 и 9 уже встретились в числе), значит, Б = 8, D = 2. Осталось найти цифру С. При перестановке цифр остаток г от деле- II ЭТАП. 2006/2007 учебный год 117
ния числа на 9 не изменяется (по признаку делимости числа на 9). Но 85 608 делится на 9, следовательно, г=0. Итак, 12С89 i 9, поэтому С = 7. Тогда искомое число есть 85 608 - 12 789 = 72 819. Отметим, что С можно найти и по-другому. Число, составленное Колей, равно 85 608 - 12С89. Независимо от значения С эта разность начинается на 7. Поскольку семерка среди найденных нами цифр в числе не встречается, то С = 7, а искомое число равно 85 608 - 12 789 = 72 819. Замечание. Отдельные продвижения (баллы за продвижения суммируются): Получен ответ (без объяснения) — 3 балла. Найдены (с обоснованием) все цифры, кроме средней (С), — 2 балла. 0 8 класс 8.1. Пусть тип — искомые числа. По условию т + п = = 2006т + п : 2006, откуда 2005л : 2006 = 2005т и п = 2006т. Значит, т + п = 2007т, откуда и следует утверждение задачи. 8.2. Ответ. Нельзя. Достаточно заметить, что при указанных операциях из пары чисел, отличающихся на 1, получается пара чисел, отличающихся на 1. 8.3. Получим сначала числа V5 - V2 и V5 + V2. Перемножив их, получим число 3. Последовательным домно- жением числа V5 на V2 и V5 мы получим числа VlO, V20, VlOO = 10. И наконец, 10-3-3-3 = 1. Замечание. Приведенный способ не единственный. 8.4. Ответ. Могли. Например, Петя сфотографировал часы в 08:56:47 и 09:13:27. Между снимками прошло 16 минут 40 секунд, т. е. 1000 секунд. Замечание. Любой правильный пример — 7 баллов. 8.5. Проведем в прямоугольном треугольнике ABC медиану СМ из вершины прямого угла С и средние линии MN и МК9 параллельные катетам (рис. 44). Они рассекут его на четыре равных прямоугольных треугольника с гипотенузами, равными 2с, причем у двух из 118 XXXIII Всероссийская математическая олимпиада школьников
них (AMNC и АМСК) общей гипотенузой будет медиана СМ исходного треугольника. Построим три круга радиуса с с центрами в серединах гипотенуз этих четырех треугольников. Каждый из них будет описанным Рис. 44 Для соответствующих треугольников и потому целиком закроет их. Поэтому вместе эти круги целиком закроют исходный треугольник (см. рис. 44). Замечание 1. За верную схему размещения кругов — 4 балла. Замечание 2. Схема на рисунке 44 не единственная возможная! Например, на ней можно немного сдвинуть самый верхний круг влево так, что треугольник останется целиком закрыт кругами. ■■▼> 9 класс 9.1. Ответ. Не может. Пусть такие наборы из шести и пяти чисел существуют. Заметим, что все простые числа, кроме 2, нечетные. Если первая сумма не содержит число 2, то она четна (сумма шести нечетных чисел). Но тогда вторая сумма, которая тем более не начинается с 2 (иначе сумма шести последовательных простых чисел будет больше суммы первых пяти простых чисел), будет нечетна и данные суммы не могут быть равны. Если первая сумма есть 2 + 3 + 5 + 7 + 11 + 13, а вторая сумма начинается с 3, то 2 + 3 + 5 + 7 + 11 +13 > >3 + 5 + 7 + 11 + 13. Значит, вторая сумма должна быть не меньше, чем 5 + 7+11 + 13 + 17. Сравнивая эти суммы, видим, что 17 > 2 + 3, поэтому вторая сумма больше. 9.2. Ответ. Верно. Первое решение. По условию а2 < 46, с2 < 4d. Пока- жем, что тогда < 4 —— . Имеем у Z J \ Z ) \{а2 +2ас + с2)^\{а2 +(а2 +с2) + с2) = \ (а2 = i (а2 +2ас + с2) < ± (а2 +(а2 +с2) + с2) = | (а < - (4Ь + Ad). (Мы использовали неравенство 2ас^а2 + с2 <=> 0 ^ (а - с)2). II ЭТАП. 2006/2007 учебный год 119
Второе решение. Пусть fx = х2 + ах + Ь, f2 = х2 + сх + d. По условию при всех х ^ > О и /2 > О, так как Dx < О и £>2 < 0. Но тогда, и f = х2 + х+ = = г 2 > 0, значит, уравнение / = 0 не имеет корней. z 9.3. Ответ. Первое (наименьп1ее) число исходного набора может иметь вид только 4k - 2, или 4ft - 1, или 4ft, или 4ft + 1 (соответственно это числа, дающие остатки 2, 3, 0 и 1 при делении на 4). А каждый из таких наборов заканчивается числом, не меньшим 4ft + 2. Значит, все возможные наборы из пяти последовательных чисел обязательно содержат в себе два соседних числа 4ft + 1 и 4ft + 2. Теперь заметим, что после двух преобразований оба числа такого вида станут равными 2ft: 4ft + 1 —► 4ft —► 2ft; 4ft + 2 ^ 2ft + 1 ^ 2ft. 9.4. Утверждение задачи равносильно тому, что ZPAC + + ZPQC = 180°, т. е. ZA + ZPQH = 90° (рис. 45). Рис. 45 Но четырехугольник PBQH вписанный, так как ZHPB + ZHQB = 90° + 90° = 180°. Поэтому ZPQH = = ZPBH (как вписанные углы, опирающиеся на одну и ту же дугу) и, значит, ZA + ZPQH = = ZA + ZPBH = 90°, т. е. ZBAH + ZABH = 90°. Это и означает, что ZBHA = 90°. Утверждение доказано. 9.5. Ответ. Нельзя. Рассмотрим шахматную раскраску нашей доски. Заметим, что изначально на черных клетках стояло 25 шашек (нечетное число). Покажем, что после каждого хода количество шашек, стоящих на черных клетках, будет нечетным. Действительно, после выполнения хода либо две шашки ушли с черных клеток, либо две шашки пришли на черные клетки, либо одна шашка пришла, а другая ушла. Также при снятии с доски пары шашек количество шашек, стоя- 120 XXXIII Всероссийская математическая олимпиада школьников
щих на черных клетках, либо не изменяется, либо уменьшается (увеличивается) на 2. Поэтому хотя бы одна шашка останется на черной клетке, а значит, и на доске. ▼) 10 класс 10.1. Пусть сначала на доске был написан трехчлен ах2 + Ъх + с. Тогда через п минут будет выписан трехчлен (а + п) х2 + (Ь + п) х + (с + п). Его дискриминант D = (Ь + п)2 - 4 (а + я) (с + л) = -Зп2 + (2Ь - 4а - 4с) л + + (Ь2 - 4ас) является квадратным трехчленом (относительно переменной п) с отрицательным старшим коэффициентом. Значит, при некотором натуральном п дискриминант будет отрицательным. Поэтому на доске появится трехчлен, не имеющий корней. 10.2. Пусть в трех орденах суммарно N магистров. Если первый магистр до встречи знал х магистров, то после встречи он стал знать 2х магистров. То есть N - 1 2х + 1 = N. Значит, в первом Ордене х +1 = — ь 1 магистр. Аналогично во втором Ордене — Ь1 ма- о ЛГ-1 __ гистр, в третьем — +1 магистр. Но тогда во всех ЛГ-1 . ЛГ-1 . N-1 л трех орденах магистров — hi Ч hi Ч h 1 = Л о 4 23 АГ о л > N. оначит, кто-то из магистров оосчи- л. А тался. 10.3. Ответ. Не существует. Предположим, что существует такое число ху что все три числа, данные в условии, целые. Тогда их сумма, равная Ху тоже целая. Первое число 2х — Jx2 +2 будет целым тогда, когда х — целое и из равенства х2 + 2 = у2 следует у2 - х2 = 2. Однако разность между двумя квадратами целых чисел не может равняться 2, поскольку между О2 и I2 разность равна 1, а между любыми двумя другими квадратами она хотя бы равна 3. Таким образом, первое число не может быть целым. 10.4. Утверждение задачи равносильно тому, что ZABK = = ZACK (рис. 46 на с. 122). Но по свойству середин- II ЭТАП. 2006/2007 учебный год 121
ного перпендикуляра DB = DC и KB = КС. Значит, ABKD = ACKD по трем сторонам. Отсюда ZACK = = ZDC.fi: = ZDBK. Но ZDBK = ZABK, т. е. ZACK = = ZABK. Значит, точки А, В, С, К лежат на одной окружности. Утверждение доказано. Рис. 47 10.5. Пусть такой треугольник существует, и у — его наибольший угол. Тогда у^60°, и, значит, 2у^ 120°. Неравенство sin (3 < sin (2y) может выполняться, если точка С тригонометрической окружности имеет большую ординату, чем точка А (рис. 47). Но это означает, что Р + 2у< 180°. С другой стороны, у^ а, поэтому 180° = а+Р + у<Р + 2у< 180° — противоречие. ▼ 11 класс 11.1. По условию а2 + 4b = а + Ь, откуда а2 - а = b - 4b. То есть а (а - 1) = л[Ь (Vb -1). Заметим, что а - 1 ^ 0, V& —1^0. Значит, а = V& (иначе каждый из сомножителей в одной из частей равенства был бы больше соответствующего сомножителя в другой части). То есть а2 = by и числа а и b — одной четности, откуда и следует утверждение задачи. 11.2. Ответ. Не существует. Предположим, что существует такое число х, что все три числа, данные в условии, целые. Тогда их сумма, равная — ху тоже целая. Тогда первое число х должно быть целым, т. е. целое. Это возможно 122 XXXIII Всероссийская математическая олимпиада школьников
только при х = ± 1. Но в этом случае второе число не является целым. 11.3. Производная квадратного трехчлена — линейная функция, которая обращается в ноль в точке, абсцисса которой совпадает с вершиной параболы, являющейся графиком трехчлена. Поэтому в этой точке сумма трехчленов отрицательна. Отсюда следует, что в этой точке значение по крайней мере одного из трехчленов отрицательно. 11.4. Ответ. Ноль баллов (все бегуны получают по О баллов). Рассмотрим любую пару бегунов. Как в начале, так и в конце пути один и тот же бегун находится впереди другого. Тогда либо вообще ни один из них не обгонял другого, либо каждый (из этих двоих) обогнал другого по одному разу. Значит, на обгонах друг друга (в рассмотренной паре) бегуны заработали по О баллов. Таким образом, в итоге все бегуны получат по О баллов. Замечание. Из решения видно, что утверждение остается верным и в случае непостоянства скоростей бегунов, когда количество обгонов в паре может быть сколь угодно большим. 11.5. Отложим от вершины S трехгранного угла SABC на ребрах отрезки SA = SB = SC = 1 (рис. 48). Пусть Al9 Ви Сх — соответственно середины отрезков ВС, СА и АВ (см. рис. 48). Тогда из равнобедренного треугольника BSC ВАХ = BS • sin | ZBSC = sin ^, \А ) А т. е. ВС = 2 sin ^. Аналогично А £, АВ = 2 sin jr. А А Но из неравенства треугольника следует, что ВС < СА + АВ, т. е. 2 sin ^ < 2 sin § + 2 sin ^. Утверж- А А А дение доказано. Рис. 48 II ЭТАП. 2006/2007 учебный год 123
Ill ЭТАП (региональный) УСЛОВИЯ ЗАДАЧ (▼> 8 класс 8.1. Найдите наименьшее четырехзначное число, такое, что произведение его цифр, увеличенных каждая на 1, равно 21. (И.Рубанов) 8.2. Докажите, что если точка, которая делит одну из сторон треугольника в отношении 1:3, равноудалена от середин двух других сторон, то треугольник прямоугольный. (Н. Агаханов) 8.3. При каком наименьшем п на шахматную доску можно поставить п ладей и п слонов так, чтобы любая ладья била хотя бы двух слонов, а любой слон бил хотя бы две ладьи? (М.Мурашкин) 8.4. На координатной плоскости проведено 20 прямых — графиков линейных функций у = kxx + bu у = k2x + b2, ..., у = k2Ox + b20, гДе каждый из коэффициентов kl9 k2y ..., k20J Ъ1У Ь2, ..., Ь20 равен одному из чисел 1, 2, ..., 20. Известно, что любые две прямые пересекаются в точке, не лежащей на оси ординат, но никакие три не проходят через одну точку. Отмечены точки пересечения всех пар прямых. Докажите, что модуль произведения абсцисс всех отмеченных точек равен 1. (И.Рубанов) 8.5. Сколько решений имеет числовой ребус ABA • АА = АВ • ААА - А, где А и В — различные цифры, А Ф 0? (М. Мураш- кин) 8.6. Имеется 40 внешне одинаковых монет, среди которых 3 фальшивые — они весят одинаково и легче, чем настоящие (настоящие монеты также весят одинаково). Как с помощью трех взвешиваний на чашечных весах без гирь отобрать 16 настоящих монет? (О.Дмитриев) 8.7. Пусть ААХ и ВВ1 — высоты остроугольного треугольника ABC у Н — точка их пересечения. Через точку, симметричную середине отрезка ВН относительно прямой ВС у провели прямую, перпендикулярную сто- 124 XXXIII Всероссийская математическая олимпиада школьников
роне АС. Докажите, что она пересекает прямую ВС в точке Ах. (Л. Емельянов) 8.8. По кругу расставлены п чисел (п > 2007), не все из которых равны. Известно, что сумма любых 13 стоящих подряд чисел не превосходит 13, а сумма любых 21 стоящих подряд чисел не превосходит 21. Докажите, что сумма всех чисел строго меньше п. (В. Дольников) Ь 9 класс 9.1. На столе лежат семь карточек. За один ход разрешается перевернуть любые пять карточек. Какое наименьшее число ходов необходимо совершить, чтобы перевернуть все карточки? (Л.Емельянов) 9.2. Корни квадратного уравнения ах2 + Ьх + с = 0 в 2007 раз больше корней квадратного уравнения сх2 + dx + а = 0. Докажите, что Ъ2 = d2. (Н. Агаханов) 9.3. На стороне АС треугольника ABC взята точка Вг. Пусть J — центр вписанной окружности треугольника. Окружность, описанная около треугольника АВХ19 вторично пересекает сторону АВ в точке Сх. Окружность, описанная около треугольника СВХ1У вторично пересекает сторону ВС в точке Ах. Докажите, что центр описанной окружности треугольника А1В1С1 не зависит от положения точки Вх на стороне АС. (Т. Емельянова) 9.4. В стране 20 городов. Авиакомпания хочет организовать двусторонние рейсы между ними так, чтобы из любого города можно было добраться в любой другой не более чем за k пересадок. При этом количество авиалиний из любого города не должно превышать четырех. При каком наименьшем k это возможно? (П. Мартынов) 9.5. В наборе из пяти палочек ни из каких трех палочек нельзя составить треугольник. Могло ли так оказаться, что, разломав одну из палочек на две, мы получим шесть палочек, из которых можно составить два равнобедренных треугольника? (С.Волченков) 9.6. Имеется 40 внешне одинаковых монет, среди которых 3 фальшивые — они весят одинаково и легче, чем настоящие (настоящие монеты также весят одинаково). Как с помощью трех взвешиваний на чашеч- III ЭТАП. 2006/2007 учебный год 125
ных весах без гирь отобрать 16 настоящих монет? (О.Дмитриев) 9.7. Пусть каждое из натуральных чисел я, п + 1, п + 2 делится на квадрат любого своего простого делителя. Докажите, что число п делится на куб некоторого своего простого делителя. (В. Сендеров) 9.8. Около треугольника ABC описана окружность. Пусть Ао и Со — соответственно середины ее дуг ВС и АВ, не содержащих вершин А и С. Оказалось, что отрезок А0С0 касается окружности, вписанной в треугольник ABC. Найдите угол В. (В. Филимонов) ф 10 класс 10.1. Существуют ли восемь натуральных чисел, среди которых ровно одно делится на 8, ровно два делятся на 7, ровно три — на 6, ..., ровно семь — на 2? (М. Мурашкин) 10.2. Даны числа а, Ь, с. Известно, что для любого х выполнено неравенство ах2 + Ьх + с ^ Ьх2 + сх + а ^ сх2 + ах + Ь. Докажите, что а = Ь = с. (И. Богданов) 10.3. Дан треугольник ABC. Через точку X, лежащую внутри него, проводятся отрезок сх> параллельный АВ, с концами на сторонах АС и ВС, и отрезок Ьх, параллельный АС, с концами на сторонах АВ и СВ. Докажите, что все точки X, для которых длины отрезков Ьх и сх равны, лежат на одной прямой. (Л. Емельянов) 10.4. На доске записано число 111... 11. Петя и Вася игра- 99 единиц ют в следующую игру, делая ходы по очереди. Начинает Петя. За ход игрок либо записывает ноль вместо одной из единиц, кроме первой и последней, либо стирает один из нулей. Проигрывает тот, после чьего хода на доске в первый раз появится число, делящееся на 11. Кто выигрывает при правильной игре? (М. Мурашкин) 10.5. В строку выписываются друг за другом без пробелов все натуральные числа в порядке возрастания: 1234567891011.... Какая из последовательностей цифр встретится в строке раньше: последовательность ров- 126 XXXIII Всероссийская математическая олимпиада школьников
но из 2006 подряд идущих шестерок (слева и справа от которой стоят не шестерки) или последовательность ровно из 2007 семерок (слева и справа от которой стоят не семерки)? (М. Мурашкин) 10.6. Пусть АВ и CD — две перпендикулярные хорды окружности с центром О, пересекающиеся в точке Е; пусть также N и Т — середины отрезков АС и BD соответственно. Докажите, что четырехугольник ENOT — параллелограмм. (В. Филимонов) 10.7. На доске написаны натуральные числа 1, 2, 3, ..., 10. Разрешается выписать число а2, если на доске уже имеется число а, или выписать наименьшее общее кратное чисел а и Ь, если числа а и & уже записаны. Можно ли с помощью таких операций получить число 1 000 000? (А. Голованов) 10.8. В стране некоторые пары городов соединены дорогами. Известно, что нет трех городов, попарно соединенных дорогами. Кроме того, для любых п дорог найдется город, из которого выходят хотя бы две из них. Докажите, что города можно так разбить на п округов, чтобы любая дорога соединяла города из различных округов. (В.Астахов) (▼► 11 класс 11.1. Существуют ли восемь натуральных чисел, среди которых ровно одно делится на 8, ровно два делятся на 7, ровно три — на 6, ..., ровно семь — на 2? (М. Мурашкин) 11.2. Даны числа а, Ь, с. Известно, что для любого х выполнено неравенство ах2 + Ьх + с ^ Ьх2 + сх + а> сх2 + ах + Ь. Докажите, что а = Ь = с. (И. Богданов) 11.3. Боковое ребро четырехугольной пирамиды назовем хорошим, если медианы двух содержащих его граней, проведенные в середину этого ребра, равны. Докажите, что если в пирамиде три боковые ребра хорошие, то четвертое боковое ребро также является хорошим. (Н. Агаханов) 11.4. В стране п городов, некоторые пары из которых соединены непересекающимися дорогами. Известно, что из любого города можно добраться по дорогам до III ЭТАП. 2006/2007 учебный год 127
любого другого, причем единственным способом (если не проезжать по одной дороге более одного раза). Докажите, что министр может объявить не более чем £г городов закрытыми (и запретить въезд в них и вы- езд из них) так, чтобы после этого для любой пары городов X, У выполнялось одно из двух условий: либо из X нельзя добраться до У, либо из X можно добраться до У, проехав не более чем по 49 дорогам. (В. Дольников) 11.5. Докажите, что при О < х < ^ выполнено неравенство (tg x)sin x + (ctg x)cos x^2. (H. Агаханов) 11.6. Пусть АВ и CD — две перпендикулярные хорды окружности с центром О, пересекающиеся в точке Е; пусть также N и Т — середины отрезков АС и BD соответственно. Докажите, что четырехугольник ENOT — параллелограмм. (В. Филимонов) 11.7. На клетчатой полоске 1 х п двое играют в следующую игру. Каждым своим ходом первый игрок закрашивает одну незакрашенную клетку, а второй — две рядом стоящие незакрашенные. Если игрок не может сделать хода — он выиграл. Кто выиграет при правильной игре? (И.Богданов, М.Исаев) 11.8. Найдите все четверки целых чисел (х, z/, z, t)y таких, что их сумма равна 0, а число х4 + у4 + z4 + t4 + 4xyzt является квадратом целого числа. (В. Сендеров) РЕШЕНИЯ ЗАДАЧ ▼ 8 класс 8.1. Ответ. 2006. 21 = 3 • 7, значит, в получившемся произведении могут встречаться только цифры 3, 7 и два раза 1. Поэтому в исходном числе были цифры 2, 6 и два раза 0. Так как число не может начинаться с 0, то, чтобы быть минимальным, оно должно начинаться с 2. А из чисел 2600, 2060 и 2006 наименьшим является 2006. 128 XXXIII Всероссийская математическая олимпиада школьников
8.2. Пусть М vl N — середины сторон АС и АВ треугольника ABC у К — точка на стороне ВС, такая, что СК : KB = = 1:3, причем МК = NK (рис. 49). Покажем, что ZACB = 90°. Соединим точки М и N и проведем в равнобедренном треугольнике MKN высоту КР. Она является медианой этого треугольника, поэтому 8.3. 8.4. МР = ± MN = -л (последнее равенство следует из того, что MN — средняя линия треугольника ABC). Таким образом, МР = КС. Кроме того, MN \\ СВУ поэтому ZPMK = ZMKC, и треугольники РМК и СКМ равны по двум сторонам (МР = КС, МК — общая) и углу между ними. Тогда ZMCK = ZKPM = 90°, что и требовалось доказать. Ответ, п = 4. Назовем пару «ладья — слон» ладейной у если в ней ладья бьет слона, и слоновой, если в ней слон бьет ладью. Заметим, что слон и ладья не могут бить друг друга, поэтому пара не может являться одновременно слоновой и ладейной. По условию ладейных пар не меньше 2п и слоновых тоже не меньше 2п. С другой стороны, общее количество пар «ладья — слон» равно п2. Поэтому 2п + 2п ^ п2, откуда п ^ 4. В случае п = 4 искомая расстановка ладей и слонов существует (рис. 50). Если для некоторой пары различных тип коэффициенты Ьт и Ьп равны, то прямые у = kmx + Ьт и у = knx + Ьп пересекаются на оси ординат, что противоречит условию. Следовательно, никакие два коэффициента из набора {bu b2, ..., Ь20} не равны, поэтому каждое из чисел 1, 2, ..., 20 встречается в наборе {&!, Ь2, ..., Ь20} ровно по разу. Рис. 50 III ЭТАП. 2006/2007 учебный год 129J
Так как никакие две прямые не параллельны, то никакие два коэффициента из набора {ku k29 ..., k20} не равны, значит, каждое из чисел 1, 2, ..., 20 встречается в наборе {ku k2J ..., k20} ровно по разу. Вычислим абсциссу точки пересечения прямых и у = кх + Ъ (тФп): х = т~ п кпх + Ъп (тФп): хтп = . кп - кт Отсюда вытекает, что произведение П всех чисел хтп равно дроби, числитель которой равен произведению всех попарных разностей чисел 1,2, ..., 20. В знаменателе дроби находится то же самое произведение разностей (возможно, соответствующие разности в числителе и знаменателе отличаются знаком). Отсюда и следует, что П равно +1 или —1. 8.5. Ответ. 8. Перепишем левую часть равенства как ABA • АА = = (10JLA + 10Б) • НА = 1111А2 + 110АВ, а правую — как АВ • ~ААА - А = (10А + В) • 111А - А = 1110А2 + + 111АВ - А, 1111А2 + 110АВ = 1110А2 + 111АВ - А, откуда А2 = АВ — А. Поделив обе части равенства на А Ф 0, получим А = В - 1. Так как В < 9, то решение существует для каждого А = 1, 2, ..., 8. 8.6. Первое решение. Разделим все монеты на две части по 20 монет и взвесим. Так как фальшивых монет нечетное число, то одна из кучек перевесит. Значит, в ней не более одной фальшивой монеты. Разделим ее на две кучки по 10 монет и взвесим их. Если чашки весов оказались в равновесии, то все 20 взвешиваемых монет настоящие. Если одна из чашек перевесила, то на ней 10 настоящих монет, а среди других 10 монет ровно одна фальшивая. Разделим эти 10 монет на три кучки, состоящие из 4, 4 и 2 монет. Третьим взвешиванием сравним две кучки по 4 монеты. Если они уравновесятся, то все 8 монет настоящие и мы нашли 18 настоящих монет. Если одна из кучек перевесит, то в ней 4 настоящие монеты, в другой кучке есть фальшивая, а 2 отложенные монеты — настоящие. Всего найдено 16 настоящих монет. Второе решение. Разделим все монеты на пять равных кучек, в каждой из которых по 8 монет, и пронумеруем их. Положим на одну чашку весов 1-ю и 2-ю кучки, а на другую — 3-ю и 4-ю. Рассмотрим первый случай — весы уравновесились. 130 XXXIII Всероссийская математическая олимпиада школьников
Тогда либо на каждой чашке находится по одной фальшивой монете, либо все монеты во взвешивании настоящие. Тогда возьмем и взвесим 1-ю и 2-ю кучки. Если они уравновесились, то все 16 монет настоящие. Если одна из кучек перевесила, то в ней 8 настоящих монет. Третьим взвешиванием сравниваем 3-ю и 4-ю кучки и определяем следующие 8 настоящих монет. Теперь рассмотрим второй случай — весы не уравновесились. Пусть для определенности перевесили 1-я и 2-я кучки, тогда среди них не более одной фальшивой монеты. Вторым взвешиванием сравним 1-ю и 2-ю кучки. Если они уравновесились, то все 16 монет настоящие. Если одна из кучек перевесила, то в ней 8 настоящих монет, а в другой ровно одна фальшивая. Следовательно, в 3-й и 4-й кучках ровно две фальшивые монеты, а в 5-й кучке 8 настоящих монет. Значит, всего найдено 16 настоящих монет. 8.7. Первое решение. Обозначим середину отрезка ВН через К, а точку, симметричную ей относительно ВС, через L (рис. 51). Так как KL и НАг перпендикулярны ВС, то KL параллельна НА19 а в силу того, что К — середина ВН9 KL содержит среднюю линию треугольника ВНА1. Значит, в четырехугольнике BLAJZ диагонали ВАг и KL делятся пополам точкой их пересечения. Тогда BLAXK — параллелограмм, и ЬАг || ВКУ а следовательно, LAX ± АС. Значит, перпендикуляр к АС, проведенный через точку L, и есть LAX. Второе решение. Обозначим середину отрезка ВН через К, а точку, симметричную ей относительно ВС, через L. Так как АгК — медиана прямоугольного треугольника ВАХН, то КАг — KB, откуда /.КВАХ = Рис. 51 А В ЭТАП. 2006/2007 учебный год 131
= ZKArB. В силу симметрии точек К и L относительно прямой ВАг ZBAXK = ZLAXB. Значит, ZBlBAl = = ZBAXK = ZBAXL9 откуда следует параллельность прямых ВВг и LAU т. е. перпендикулярность прямых ЬАг и АС. 8.8. Пусть по окружности расставлены числа а19 а2, ..., ал. Каждое слагаемое в сумме Т = (ах + а2 + ... + а13) + + (а2 + а3 + ... + а14) + ... + (ап + ах + а2 + ... + а12) не превосходит 13, и каждое из чисел ах, а2, ..., ал входит в эту сумму 13 раз. Поэтому S = ах + а2 + ... + ап = Для рекгения задачи теперь достаточно доказать, что если S = п, то все числа равны 1. Будем говорить, что для расставленных чисел выполнено С (ft), если сумма любых k подряд идущих чисел равна k. Если S = п, то каждая скобка в сумме Т равна 13, т. е. выполнено условие С (13). Аналогично если S = п, то выполнено и условие С (21). Заметим, что если при натуральных а > Ь выполняются условия С (а) и С (ft), то выполняется и условие С (а - Ь). Действительно, возьмем любые а - Ь подряд идущих чисел и следующие за ними Ь чисел. Сумма Ь чисел равна Ь, а сумма всех а чисел равна а, значит, сумма данных а - Ъ чисел равна а -Ь. Значит, из условий С (21) и С (13) последовательно выводим условия С (21 - 13) = С (8), С (13 - 8) = С (5), С (8 - 5) = С (3), С (5 - 3) = С (2), С (3 - 2) = С (1). Но условие С (1) и означает, что все числа на окружности равны 1, что противоречит условию, следовательно, S < п. ® 9 класс 9.1. Ответ. 3 хода. Очевидно, что одного хода не хватит. После двух ходов найдутся по крайней мере три карточки, перевернутые по два раза, а значит, эти карточки будут в исходном положении. Приведем пример переворачивания всех карточек за три хода. Пронумеруем карточки цифрами от 1 до 7 и перевернем за первый ход карточки с номерами 1, 2, 3, 4, 5, за второй ход — 1, 3, 4, 5, 6, а за третий ход — 1, 3, 4, 5, 7. 132 XXXIII Всероссийская математическая олимпиада школьников
9.2. Пусть xl9 x2 — корни уравнения сх2 + dx + а = 0, тогда пх1 и пх2 — корни уравнения ах2 + Ъх + с = О, где п = 2007. Тогда по теореме Виета хг+х2 = —, хгх2 = ~> njci + п*2 = —» пх1пх2 = -, т. е. *. (**) с2 Из равенства (**) получаем п2 = —, а из равенства (*) d2n2 b2 b2 d2 с2 d2 имеем —г— = —, откуда — = —— = —, значит, cz az a* cz az a* b2 = d2. Замечание. Ясно, что числа — и - имеют одинаковые зна- с а ки — они различаются в 2007 раз. Поэтому, если знаки а и с совпадают, то Ь = d, иначе Ь = -d; оба случая возможны. 9.3. Заметим, что AI — биссектриса угла ВАС. Из равенства углов СгА1 и ВгА1 следует равенство хорд Сг1 и Вг1. Аналогично Вг1 = АХ1 (рис. 52). Значит, точка J является центром описанной окружности для всех треугольников А^В^^ В Рис. 52 9.4. Ответ, k = 2. Заметим, что потребуется сделать не менее двух пересадок. Действительно, из произвольного города А без пересадок можно добраться не более чем в 4 города, а ровно с одной пересадкой — не более чем в 4 • 3 = 12 городов (так как один из рейсов ведет из каждого из этих городов в А). Итак, если использовать не более одной пересадки, то из любого города можно долететь не более чем в 16 других городов, III ЭТАП. 2006/2007 учебный год 133
12 Рис. 53 а требуется — в 19. На рисунке 53 показано, как можно организовать рейсы, чтобы было не более двух пересадок. Картинка симметрична, поэтому достаточно показать, как можно добраться из первого города в любой другой. Из него без пересадок можно добраться до городов 2, 3, 4, 5. Затем с одной пересадкой — в города 6, 7, 8 (из 5), 9 (из 2), 13 (из 3) и 17 (из 4). И с двумя пересадками — в города 10, 11, 12 (из 9), в 14, 15, 16 (из 13), в 18, 19, 20 (из 17). 9.5. Ответ. Могло. Например, подойдет набор палочек с длинами 1,1,2, 3, 5. Разломив палочку длины 5 на две палочки длиной 2 и 3, мы сможем составить два равнобедренных треугольника со сторонами 1, 2, 2 и 1, 3, 3. 9.6. См. решение задачи 8.6. 9.7. Предположим противное: число п делится ровно на вторую степень каждого своего простого делителя. Тогда п = ft2, где ft — произведение всех простых делителей р. Следовательно, достаточно доказать, что п не может являться точным квадратом. Предположим противное: п = т2, где т — натуральное. Если п четно, то п + 2 также четно, и одно из них не делится на 4, что противоречит условию. Если п нечетно, то и т нечетно, т — 2ft — 1. Тогда п + 1 = (2ft - I)2 + 1 = 4 (ft2 - ft) + 2 делится на 2, но не делится на 4, что опять противоречит условию. 9.8. Обозначим вписанную окружность треугольника ABC через со, ее центр — через /, точки касания со с АС и А0С0 — через К и L соответственно. Тогда в треугольниках IKC и ILA0 углы IKC и ILA0 прямые (как 134 XXXIII Всероссийская математическая олимпиада школьников
углы между радиусом и касательной) и IK = IL, так как оба этих отрезка — радиусы со (рис. 54). Также, поскольку точки А, /, Ао лежат на биссектрисе угла А, а точки С, J, Со — на биссектрисе угла С, то ZICK = ZC0CA = ZC0A0A = = ZLAqI (так как углы ZC0A0A и ZC0CA опираются на одну дугу описанной окружности Рис" 54 С0А). Поэтому прямоугольные треугольники IKC и ILA0 равны по катету и острому углу, следовательно, 1А0 = /С, т. е. треугольник 1А0С 1 / . ~^ -\ равнобедренный. Далее, ZA0IC= - \АС0 + А0С) = = \ (С^}+ А^В) = \ А^ВС0 = ZA0CC0 =» ZA0IC = = ZA0CI => AqI = A0C. Значит, AAqIC — равносторонний. Отсюда ZABC = ZAA0C = 60°. Замечание. Если рассмотреть щюизвольный треугольник ABC, в котором ZB = 60°, то для него отрезок А0С0 касается со. Действительно, проведя все рассуждения в обратном порядке (приняв за L проекцию / на А0С0), получаем равенство треугольников IKC и ILA0 по гипотенузе и острому углу, из которого следует, что IK = IL, поэтому L лежит на со, и, следовательно, А0С0 касается со. 3 класс 10.1. Ответ. Не существуют. Предположим, что такие восемь чисел найдутся. Из условия следует, что ровно одно из них не делится на 2 и ровно два из них не делятся на 3. Значит, среди рассматриваемых чисел не менее пяти чисел делятся на 2 и на 3, т. е. делятся на 6. Но по условию чисел, делящихся на 6, должно быть ровно три. Противоречие. 10.2. Из неравенств в условии получаем, что трехчлены (ах2 + Ьх + с) - (Ьх2 + сх + а) и (Ьх2 + сх + а) - — (сх2 + ах + Ъ) принимают неотрицательные значения при всех значениях х. Отсюда следует, что их III ЭТАП. 2006/2007 учебный год 135
старшие коэффициенты а — Ь и Ь — с неотрицательны, т. е. а ^ Ь ^ с. С другой стороны, подставив в исходные неравенства х = О, получаем с ^ а ^ Ь, откуда а ^ b ^ с ^ а, что возможно лишь в случае равенства всех коэффициентов. 10.3. Рассмотрим некоторую точку X, лежащую внутри треугольника ABC, для которой длины отрезков Ъх и сх равны. Обозначим через К, L, М, N концы отрезков Ьх и сх (рис. 55). Достроим треугольник КХМ до параллелограмма КХМТ. Пусть ТК и АВ пересекаются в точке Р, а ГМ и АС — в точке Q. По построению APTQ — параллелограмм, причем ТР = MN = = KL = TQ, т. е. APTQ — ромб. Отсюда вытекает, что Рис. 55 точка Т лежит на биссектрисе АА1 угла ВАС. Из подобий ААВАХ ™ АТМАг и ААСАг ™ АТКАХ следует: АгК _ АгТ _ АгМ Проведем через точку В прямую, параллельную АС, до пересечения с прямой ХАХ в точке R. Также проведем через точку С прямую, параллельную АВ, до пересечения с прямой ХАХ в точке S. Из подобий ААгМХ со AAXBR и ААгКХ ™ AAXCS вытекает, АХХ АХМ АгК АХХ A Ft- А Я чит, точки S и R совпадают с такой точкой А', что 136 XXXIII Всероссийская математическая олимпиада школьников
ABA'С — параллелограмм. Получаем, что точка X лежит на фиксированной прямой АХА\ 10.4. Ответ. Петя. Заметим, что число Ал = 1011...11 делится на 11. п единиц Действительно, если п четно, то Ап = 1000 ... 00 + п + 1 нулей + 11... 11, а если п нечетно, то Ап = 9000... 00 + п единиц п нулей + 111... 11. В обоих случаях второе слагаемое делит- п + 1 единиц ся на 11, а первое нет. Поскольку с каждым ходом число на доске уменьшается, рано или поздно один из игроков проиграет. Предъявим беспроигрышную стратегию для Пети, действуя по которой он сможет после каждого своего хода получать число вида Ап. Первым ходом Петя заменит вторую слева единицу на ноль, а в дальнейшем он либо стирает ноль, появившийся на предыдущем ходе Васи (т. е. число вида Ат)9 либо, если тот стер единственный ноль, опять заменяет вторую слева единицу на ноль. Если Петя не может сделать последнего действия, то перед ним число 11, т. е. Вася уже проиграл на предыдущем ходу. 10.5. Ответ. Последовательность ровно из 2007 семерок. Последовательность ровно из 2007 семерок появится, например, при выписывании чисел N = 777 ... 77 и 1004 семерки N + 1 = 777 ... 78, так как числу N предшествует чис- 1003 семерки л о N — 1, оканчивающееся на цифру 6. Предположим, что последовательность р из 2006 подряд идущих шестерок встретилась раньше числа N. Подчеркнем в строке у каждого из чисел 1, 2, ..., N— 1, N последнюю цифру. Между ближайшими подчеркнутыми цифрами не более 1003 цифр (так как N — 1004-значное число), поэтому в последовательности р есть подчеркнутая шестерка (т. е. последняя цифра некоторого числа М), причем ровно одна, так как ближайшие к ней подчеркнутые цифры 5 и 7: ... 5 6 7... . В числах МиМ+1 соответству- М М + 1 III ЭТАП. 2006/2007 учебный год 137
ющие цифры, кроме последних, равны. Поэтому количество шестерок между 5 и 7 — нечетное число, не превосходящее 2 • 1003 + 1 = 2007, т. е. между 5 и 7_ либо 2007 шестерок подряд, либо не более 2005 шестерок. Противоречие. 10.6. Продолжим отрезок ТЕ до пересечения с прямой АС в точке К (рис. 56). Покажем, что отрезок ЕК — высота треугольника СЕ А. Пусть ZCAB = ос. Из прямоугольного треугольника АЕС получаем ZACE = = 90° - а. Далее, ZCDB = ZCAB = а (как вписанные, опирающиеся на одну дугу). С другой стороны, ЕТ — медиана в прямоугольном треугольнике BED, поэтому ЕТ = DT и ZTED = ZTDE = ос, и ZKEC = а (как вертикальный с углом TED). Отсюда получаем ZEKC = 180° - ZKEC - ZKCE = 90°. Рис. 56 Таким образом, ТЕ JL АС и ON JL АС (так как п — середина АС), откуда ТЕ || ON. Аналогично ОТ || EN, что и требовалось. Замечание. Утверждение задачи остается в силе и в том случае, если в точке Е пересекаются не сами хорды АВ и CD, а их продолжения. Доказательство в этом случае полностью аналогично изложенному. 10.7. Ответ. Нельзя. Обозначим через д (п) такое наибольшее целое число й, что п делится на 5Л. Для каждого п из начального набора чисел д (п) = 0 или д (п) = 1 = 2°. Заметим, что д (а2) = 2д (а), а д (НОК (а, Ь)) равно д (а) или д (Ь). Отсюда вытекает, что для чисел п, выписанных в любой момент на доске, число д (п) является целой степенью двойки или равно 0. Число 1 000 000 не могло быть выписано, поскольку 138 XXXIII Всероссийская математическая олимпиада школьников
q (1 000 000) = 6 не является ни нулем, ни степенью двойки. 10.8. Лемма. Пусть в множестве М городов отмечено ровно 2k городов, а среди неотмеченных никакие два не соединены дорогой. Если в множестве М нет трех городов, попарно соединенных дорогами, то множество М можно разбить на (k + 1) округов так, чтобы любая дорога соединяла города из разных округов. Доказательство. Применим индукцию по k. Если k = 1, то отмечены два города X и Y. Если между X и У нет дороги, то отмеченные и неотмеченные города образуют нужные округа. Если X и У соединены дорогой, то среди неотмеченных нет городов, соединенных и с X, и с У. Поэтому в первый округ можно включить все города, в которые из X ведет дорога (в том числе город У), а во второй округ — все остальные города. Пусть k > 2. Среди отмеченных городов выделим округ из двух городов, не соединенных дорогой (такие найдутся). По предположению индукции оставшиеся 2ft - 2 отмеченных города и все неотмеченные города можно разбить на ft округов нужным образом. Лемма доказана. ■ Перейдем к решению задачи. На первом шаге отметим некоторые два города, соединенные дорогой. Далее, на очередном шаге среди еще не отмеченных городов выбираем два города, соединенные дорогой, и отмечаем их. Действуем так, пока это возможно. В конце концов (после ft шагов) получим 2ft отмеченных городов, и среди неотмеченных городов нет двух, соединенных дорогой. Из условия следует, что ft^n-1. По лемме все города можно разбить на ft + 1 ^ п округов нужным образом. ▼I 11 класс 11.1. Ответ. Не существует. См. решение задачи 10.1. 11.2. См. решение задачи 10.2. 11.3. Обозначим через А', В\ С", D' середины боковых ребер SA, SB, SC, SD пирамиды SABCD соответственно (рис. 57 на с. 140). Пусть для определенности ребра SB, SC и SD хорошие, т. е. АВ' = СВ', BCf = DC, III ЭТАП. 2006/2007 учебный год 139
Рис. 57 CD' = AD'. Геометрическое место точек, равноудаленных от А и С, — плоскость, перпендикулярная отрезку АС (и проходящая через его середину). Значит, точки В' и D' лежат в этой плоскости, поэтому B'D' JL АС. Так как B'D' — средняя линия треугольника SBD, то BD || B'D', откуда BD 1 АС. Так как А'С — средняя линия треугольника SAC, то АС || А'С1', откуда BD _1_ А'С, значит, точка А' лежит в плоскости, перпендикулярной BD и проходящей через точку С. Так как ВС = DC, то эта плоскость является геометрическим местом точек, равноудаленных от В и D. Отсюда ВА' = DA', т. е. SA — хорошее ребро. 11.4. Применим индукцию по п. При п ^ 50 утверждение верно. Докажем утверждение для некоторого п ^ 51, предполагая, что оно верно для 1, 2, ..., п — 1. Для каждой пары городов X, Y обозначим через р (X, Y) единственный путь от X до У, в котором ни одна дорога не пройдена дважды. Зафиксируем город А (столицу) (рис. 58). Для каждого города С путь р (А, С) содержит А, С и некоторое количество промежуточных городов (в частности, р (А, А) содержит только А). Если город С входит в путьр (A, D), то бу- Рис. 58 140 XXXIII Всероссийская математическая олимпиада школьников
дем говорить, что D подчинен С (в частности, С подчинен самому себе, а все города подчинены А). Если город D подчинен городу С и, кроме этого, имеется дорога из D в С, то скажем, что D — пригород С. Среди всех городов, у которых не меньше 51 подчиненных (таков, например, город А)9 рассмотрим город Б с наименьшим количеством I подчиненных. Пусть В19 В29 ..., Вг — все пригороды города В; у города Bt подчиненных меньше, чем у города В, поэтому их не больше 50 (иначе получилось бы противоречие с выбором Б). Сделаем город Б закрытым. После этого из города Х9 подчиненного городу Bi9 нельзя добраться до города У, не подчиненного Bi9 так как иначе путь р (Х9 Y) проходил бы через город Б. Если же города X и У подчинены Bi9 то путь р (Х9 У) содержит только города, подчиненные Bi9 и в нем не более 49 дорог. Далее, к множеству городов, не подчиненных Б (таких городов не более п - 51, и любые два из них по-прежнему соединены путем), можно применить предположение л л-51 л индукции и закрыть не более ——— городов, чтобы выполнялось условие задачи. Итого закрыто не более ——— +1 = -£т городов, и все условия выполнены. ( 1 Г ^ 11.5. Пусть а е 0, ^ , beR,f и а+Ъ = ^. Тогда V 4J L4 *) * (tg b)8inb = (ctg a)cosa, (ctg b)cosb = (tg a)sina. Таким образом, если требуемое неравенство выполняется для х = а, то оно выполняется и для х = Ь; поэтому достаточно доказать неравенство для х е 0, ^ . Если х е 0, ^ , то ctg х > 1, cos x > sin x9 поэтому (tg xf* х + (ctg x)cos х > (tg xf™ x + (ctg xfin x = (tg x)s + :— = А + — для положительного А. Но (tg x)sm x A /— I +2^2. у[А) 11.6. См. решение задачи 10.6. 11.7. Ответ. При п9 делящихся на 3, выигрывает первый; при всех остальных п — второй. Ill ЭТАП. 2006/2007 учебный год 141
Заметим, что после ходов первого и второго игроков количество закрашенных клеток увеличивается на 3. Поэтому после хода второго оно всегда кратно трем. Пусть п не кратно трем. Тогда второй может ходить, как ему заблагорассудится. Предположим, что после очередного хода второго первый не может сделать ход. Тогда все клетки закрашены. Это противоречит тому, что количество закрашенных клеток кратно трем. Поэтому первый всегда сможет сделать ход, и выиграет второй. Пусть п кратно трем. Приведем выигрышную стратегию для первого. Первым ходом он закрасит клетку, отстоящую на 3 от края; тогда полоска разбилась на не более чем два куска, один из которых имеет длину 2 (назовем такой кусок домино). Покажем, что первый сможет добиться выполнения следующего условия: После хода первого если полоска разбита на k кусков, то не менее - из них — домино. После хода вто- рого если полоска разбита на k кусков, то не менее k-1 из них — домино. В таком случае после любого хода первого останется хотя бы один кусок (так как до его хода количество клеток было кратно трем), а значит, останется хотя бы одно домино; поэтому второй всегда сможет сделать ход и проиграет. Итак, пусть после очередного хода первого условие выполняется. Второй своим ходом либо закрашивает домино (тогда число кусков k уменьшается на 1), либо закрашивает 2 клетки в большем куске (тогда k увеличивается не более чем на 1). В любом случае условие выполнено. Если еще остался кусок длины 3 или больше, то первый закрашивает в нем третью клетку от края; тогда количество домино увеличивается хотя бы на 1, a k — не больше чем на 1. Если остался кусок длины 1, то, закрасив его, первый просто уменьшает количество кусков. Наконец, если все куски — домино^ то их хотя бы три, так как их суммарная длина делится на 3. Тогда после закрашивания первым произвольной клетки остается хотя бы три куска, 142 XXXIII Всероссийская математическая олимпиада школьников
из которых только один не домино. Таким образом, после хода первого условие опять же выполнено. 11.8. Ответ. x = y = z = t = O. Первое решение. Возведем в квадрат обе части равенства х + у = - (z + t)9 получим у? + 2ху + у2 = z2 + 2zt + t2 => => х2 + у2 - z2 - t2 = 2 (zt - ху) => (х2 + y2-z2- t2)2 = = ±(zt- xy)2. Раскрыв скобки, получим из этого равенства Axyzt = = А - |, где А = х2у2 + x2z2 + x2t2 + y2z2 + y2t2 + z2*2, Обозначим искомое число через N. Отсюда 2N = = 2(x4 + y4 + z4 + t4 + 4xyzt) = I = (х2 + у2 + z2 + t2)2. Если х2 + у2 + z2 + t2* 0, то в разложение числа N на простые множители двойка входит в нечетной степени, значит, N не является квадратом. Итак, х2 + у2 + z2 + t2 = 0, что приводит к решению x = y = z = t = O. Второе решение. Пусть четверка чисел, не все из которых равны нулю, удовлетворяет условию задачи. Если каждое из них четно, разделим все числа на 2, получим четверку чисел, удовлетворяющих условию. Продолжим деление на 2 до тех пор, пока одно из чисел не станет нечетным. Так как сумма чисел нечетна, то среди них либо два нечетных, либо все они нечетные. Если среди чисел два нечетных, то N = х4 + у4 + z4 + + t4 + 4:Xyzt имеет остаток 2 при делении на 4 и поэтому не является точным квадратом. Пусть все числа нечетны: х = 2а + 1, z/ = 2b+l, z = 2с + 1, t = 2d + 1, где a + b + c + d = —2. Покажем, что N = х4 + у4 + z4 + t4 + 4xyzt делится на 8, но не делится на 16. Отсюда будет следовать, что N не является точным квадратом. Запишем N = 16 (а4 + Ъ4 + + с4 + d4) + 32 (а3 + б3 + с3 + d3) + 24 (а2 + Ь2 + с2 + d2) + + 8(a + 6 + c + d) + 4 + 4 (41Г + 2а + 2b + 2с + 2d + 1), где К — целое число. Далее, так как а + Ь + с + d = = -2, то а2 + Ь2 + с2 + d2 четно, откуда N = 16М + 8, где М — целое число. ЭТАП. 2006/2007 учебный год 143
IV ЭТАП (федеральный окружной) УСЛОВИЯ ЗАДАЧ 0 8 класс 8.1. В выпуклом четырехугольнике семь из восьми отрезков, соединяющих вершины с серединами противоположных сторон, равны. Докажите, что все восемь отрезков равны. (Н.Агаханов) 8.2. Петя задумал натуральное число и для каждой пары его цифр выписал на доске их разность. После этого он стер некоторые разности, и на доске остались числа 2, О, О, 7. Какое наименьшее число мог задумать Петя? (М.Мурашкин) 8.3. Существуют ли такие простые числа р19 р29 ..., Ргоо7> что р\ - 1 делится на р2, р\-1 делится на р3, ..., Р!оо7 ~ 1 делится на рх1 (В. Сендеров) 8.4. На шахматной доске расставлены во всех клетках 32 белых и 32 черных пешки. Пешка может бить пешки противоположного цвета, делая ход по диагонали на одну клетку и становясь на место взятой пешки (белые пешки могут бить только вправо-вверх и влево-вверх, а черные — только влево-вниз и вправо-вниз). Другим образом пешки ходить не могут. Какое наименьшее количество пешек может остаться на доске? (И.Богданов) 8.5. Среди 11 внешне одинаковых монет 10 настоящих, весящих по 20 г, и одна фальшивая, весящая 21 г. Имеются чашечные весы, которые оказываются в равновесии, если груз на правой их чашке ровно вдвое тяжелее, чем на левой. (Если груз на правой чашке меньше, чем удвоенный груз на левой, то перевешивает левая чашка, если больше — то правая.) Как за три взвешивания на этих весах найти фальшивую монету? (И.Рубанов) 8.6. В натуральном числе А переставили цифры, получив число Б. Известно, что А - В = 11... 1. Найдите наи- N единиц меньшее возможное значение N. (Н. Агаханов) 144 XXXIII Всероссийская математическая олимпиада школьников
8.7. Внутри равнобедренного треугольника ABC (АВ = ВС) выбрана точка М таким образом, что ZAMC = 2ZABC. На отрезке AM нашлась такая точка К> что ZBKM = = ААВС. Докажите, что ВК = КМ + МС. (С.Берлов) 8.8. В классе учатся 15 мальчиков и 15 девочек. В день 8 Марта некоторые мальчики позвонили некоторым девочкам и поздравили их с праздником (никакой мальчик не звонил одной и той же девочке дважды). Оказалось, что детей можно единственным образом разбить на 15 пар так, чтобы в каждой паре оказались мальчик с девочкой, которой он звонил. Какое наибольшее число звонков могло быть сделано? (С. Берлов) Ф 9 класс 9.1. Петя придумал 1004 приведенных квадратных трехчлена fl9 ..., /юо4> среди корней которых встречаются все целые числа от 0 до 2007. Вася рассматривает всевозможные уравнения ft = fj (i Ф у), и за каждый найденный у них корень Петя платит Васе по рублю. Каков наименьший возможный доход Васи? (И.Рубанов) 9.2, Существуют ли такие простые числа р19 р2, ..., Р2007» что р\ - 1 делится на р2> р\ - 1 делится на р3, ..., Р!оо7 ~ 1 делится на рг? (В. Сендеров) 9.3, 25 мальчиков и несколько девочек собрались на вечеринке и обнаружили забавную закономерность. Если выбрать любую группу не меньше чем из 10 мальчиков, а потом добавить к ним всех девочек, знакомых хотя бы с одним из этих мальчиков, то в получившейся группе число мальчиков окажется на 1 меньше, чем число девочек. Докажите, что некоторая девочка знакома не менее чем с 16 мальчиками. (С.Волчен- ков) 9.4. У двух треугольников равны наибольшие стороны и равны наименьшие углы. Строится новый треугольник со сторонами, равными суммам соответствующих сторон данных треугольников (складываются наибольшие стороны двух треугольников, средние по длине стороны и наименьшие стороны). Докажите, что площадь нового треугольника не меньше удвоенной суммы площадей исходных. (Н.Агаханов) IV ЭТАП. 2006/2007 учебный год 145
9.5. Среди 11 внешне одинаковых монет 10 настоящих, весящих по 20 г, и одна фальшивая, весящая 21 г. Имеются чашечные весы, которые оказываются в равновесии, если груз на правой их чашке ровно вдвое тяжелее, чем на левой. (Если груз на правой чашке меньше, чем удвоенный груз на левой, то перевешивает левая чашка, если больше — то правая.) Как за три взвешивания на этих весах найти фальшивую монету? (И.Рубанов) 9.6. На стороне ВС треугольника ABC выбрана произвольная точка D. В треугольники ABD и ACD вписаны окружности с центрами К и L соответственно. Докажите, что существует окружность со, такая, что при любой выбранной точке D описанные окружности треугольников BKD и CLD вторично пересекаются на со. (Л.Емельянов) 9.7. Бесконечная возрастающая арифметическая прогрессия, состоящая из натуральных чисел, содержит точный куб натурального числа. Докажите, что она содержит и точный куб, не являющийся точным квадратом. (И.Богданов, В.Сендеров) 9.8. Среди натуральных чисел от 1 до 1200 выбрали 372 различных числа так, что никакие два из них не различаются на 4, 5 или 9. Докажите, что число 600 является одним из выбранных. (Д.Храмцов) 0 10 класс 10.1. В 25 коробках лежат шарики нескольких цветов. Известно, что при любом k (I ^ k ^ 25) в любых k коробках лежат шарики ровно k + 1 различных цветов. Докажите, что шарики одного из цветов лежат во всех коробках. (С.Волченков) 10.2. Для действительных чисел х и у, х > у > 0, и натуральных чисел п и fe, n > k9 докажите неравенство (xk - yk)n < (хп - yn)k. (В. Сендеров) 10.3. При каком наименьшем п для любого набора А из 2007 множеств найдется такой набор В из п множеств, что каждое множество набора А является пересечением двух различных множеств набора В? (И.Богданов, Г.Челноков) 10.4. Окружность проходит через вершины В и С треугольника ABC и пересекает стороны АВ и АС в точках D 146 XXXIII Всероссийская математическая олимпиада школьников
и Е соответственно. Отрезки CD и BE пересекаются в точке О. Пусть М и N — центры окружностей, вписанных соответственно в треугольники ADE и ODE. Докажите, что середина меньшей дуги DE лежит на прямой MN. (М.Исаев) 10.5. В натуральном числе А переставили цифры, получив число В. Известно, что А — В = 11... 1. Найдите наи- N единиц меньшее возможное значение N. (Н. Агаханов) 10.6. Точка D на стороне ВС треугольника ABC такова, что радиусы вписанных окружностей треугольников ABD и ACD равны. Докажите, что радиусы окружностей, вневписанных в треугольники ABD и ACD, касающихся соответственно отрезков BD и CD, также равны. (Л.Емельянов) 10.7. Дано натуральное число п > 6. Рассматриваются натуральные числа, лежащие в промежутке (п (п - 1); п2) и взаимно простые с п (п - 1). Докажите, что наибольший общий делитель всех таких чисел равен 1. (В. Астахов) 10.8. В клетках таблицы 15 х 15 изначально записаны нули. За один ход разрешается выбрать любой ее столбец или любую строку, стереть записанные там числа и записать туда все числа от 1 до 15 в произвольном порядке — по одному в каждую клетку. Какую максимальную сумму чисел в таблице можно получить такими ходами? (М.Мурашкин) ▼ 11 класс 11.1. В 25 коробках лежат шарики нескольких цветов. Известно, что при любом k (I ^ k ^ 25) в любых k коробках лежат шарики ровно k + 1 различных цветов. Докажите, что шарики одного из цветов лежат во всех коробках. (С. Волченков) 11.2. Квадратные трехчлены fx (x) и /2 (х) таковы, что f[(x)f'2(x)>\f1(x)\ + \f2(x)\ при всех действительных х. Докажите, что произведение Д (х) f2 (x) равно квадрату некоторого трехчлена. (Н. Агаханов) 11.3. В треугольнике ABC на стороне ВС выбрана точка М так, что точка пересечения медиан треугольника АВМ IV ЭТАП. 2006/2007 учебный год 147
лежит на описанной окружности треугольника АСМ9 а точка пересечения медиан треугольника АСМ лежит на описанной окружности треугольника АВМ. Докажите, что медианы треугольников АВМ и АСМ из вершины М равны. (А. Бадзян) 11.4. На столе лежат купюры достоинством 1, 2, ..., 2п тугриков. Двое ходят по очереди. Каждым ходом игрок снимает со стола две купюры, большую отдает сопернику, а меньшую забирает себе. Каждый стремится получить как можно больше денег. Сколько тугриков получит начинающий при правильной игре? (Г. Челнокову И. Богданов) 11.5. При каких натуральных п найдутся такие целые а, Ъ9 с9 что их сумма равна нулю, а число ап + Ъп + сп — простое? (В. Сендеров) 11.6. На плоскости отмечено несколько точек, каждая покрашена в синий, желтый или зеленый цвет. На любом отрезке, соединяющем одноцветные точки, нет точек этого же цвета, но есть хотя бы одна другого цвета. Каково максимальное возможное число всех точек? (М.Мурашкин) 11.7. Назовем многогранник хорошим, если его объем (измеренный в м3) численно равен площади его поверхности (измеренной в м2). Можно ли какой-нибудь хороший тетраэдр разместить внутри какого-нибудь хорошего параллелепипеда? (М.Мурашкин) 11.8. Для положительных чисел х19 х29 ..., хп докажите неравенство (1 + хх) (1 + хх + х2) ... (1 + хх + х2 + ... + хп) > > <y](n + l)n + 1 ^хгх2 ... хп . (М. Мурашкин) РЕШЕНИЯ ЗАДАЧ 0 8 класс 8.1. Предположим, что восьмой отрезок не равен остальным, и это отрезок СК9 где ABCD — данный четырехугольник, а точки К9 L, М, N — середины сторон АВ9 ВС9 CD9 DA соответственно (рис. 59). Тогда в равнобедренных треугольниках ALD и BNC отрезок LN является медианой, а поэтому и высотой. Далее, прямоугольные треугольники ALN и BNL равны 148 XXXIII Всероссийская математическая олимпиада школьников
по гипотенузе и катету, отсюда AN = BL9 значит, ANLB — прямоугольник. Аналогично, DNLC — прямоугольник, и, значит, ABCD — прямоугольник. Из доказанного вытекает, что прямоугольные треугольники DKA и СКВ равны по двум катетам, отсюда С К = DK вопреки нашему предположению. Замечание 1. Можно завершить решение по-другому. После доказательства того, что LN — серединный перпендикуляр к AD и ВС у ясно, что четырехугольник ABCD симметричен относительно LN; тогда все 8 рассматриваемых отрезков разбиваются на пары симметричных, и количество равных отрезков должно быть четно. Замечание 2. Из условия вытекает, что данный четырехугольник — квадрат. Утверждение задачи можно усилить: равенство всех восьми отрезков следует из равенства пяти из них. 8.2. Ответ. 11 138. Число 11 138 могло быть задумано: 2 = 3-1,0 = 1-1, 7 = 8-1. Предположим, что задумано число N < 11 138. Поскольку выписаны разности 2 и 7, то различных цифр в числе N не менее трех. Так как выписаны два нуля, то среди цифр найдутся либо три одинаковых, либо две пары равных. Так как N < 11 138, то в числе N ровно пять цифр, среди которых ровно три различные цифры, и первая цифра равна 1. Если в записи числа N встречаются цифры 0, 1 и с (а > 1), то среди разностей цифр встречаются лишь числа 0, 1, а, а — 1, что невозможно. Иначе в записи числа N нет нулей, и N = lllbCy где 6 = 2 или 6 = 3 (так как N < 11 138). Тогда с > 1 + 7 = 8, откуда 6 Ф 3. Но если 6 = 2, то среди разностей цифр встречаются лишь числа 0, 1, с - 1, с - 2. Противоречие. 8.3, Ответ. Не существуют. Предположим, что такие числа существуют. Пусть одно из чисел равно 2, скажем, рх = 2. Тогда последовательно получаем, что р2 = 3, р3 = 2, р4 = 3, р5 = 2, ..., р2007 = 2, рх = 3 — противоречие. IV ЭТАП. 2006/2007 учебный год 149
Пусть все числа — нечетные простые и рг — наибольшее из них. Пусть р2оо7 = Я. ^Pi- Тогда q2 - 1 = 2 • 2 х q - 1 q + 1 х —-— • —-—, где все сомножители натуральные и не превосходят <g<px. Получаем противоречие с тем, что q2 — 1 делится на рг. 8.4. Ответ. Две. Заметим, что пешка, стоявшая на черной клетке, все время будет перемещаться только по черным клеткам. Тогда после каждого хода (по черным клеткам) на черных клетках всегда будет оставаться хотя бы одна пешка — та, которая делала ход. Аналогично, хотя бы одна пешка будет оставаться на белых клетках, и всего останется не меньше двух пешек. А А Ъ и 1 Аи Д Рис. 60 На рисунке 60 показано, как расставить пешки на черных клетках и как ходить ими так, чтобы осталась только одна. Расстановка и действия на белых клетках доски аналогичны. 8.5. Приведем один из возможных способов. Сначала покажем, как за два взвешивания найти фальшивую монету среди пяти, четыре из которых настоящие. Положим на левую чашку одну монету, а на правую — две. Если перевесила левая чашка, то фальшивая монета на ней. Если перевесила правая чашка, то фальшивая монета — одна из двух на правой чашке; если весы в равновесии, фальшивая монета — одна из двух оставшихся. В любом случае у нас есть две «подозреваемые» в фальшивости монеты и известны три настоящих. Положим на левую чашку одну из «подозреваемых», а на правую — две настоящих. Если левая чашка перевесила, то фальшивая монета на ней; если весы в равновесии, то фальшивая — оставшаяся из двух «подозреваемых». 150 XXXIII Всероссийская математическая олимпиада школьников
Пусть теперь у нас 11 монет. Положим на правую чашку весов любые 4 из них, а на левую — любые две. Если весы в равновесии, фальшивая монета — среди пяти не лежащих на весах, и мы находим ее за оставшиеся два взвешивания. Если перевесила одна из чашек — фальшивая монета на ней, и мы сузили круг «подозреваемых» монет до двух или четырех. Добавляя к ним соответственно три или одну монету с другой чашки, снова сводим задачу к поиску одной фальшивой монеты среди пяти за два взвешивания. Замечание. На самом деле за три взвешивания на таких весах можно выявить фальшивую монету даже из двадцати одной. 8.6. Ответ. 9. Как известно, любое число имеет тот же остаток от деления на 9, что и его сумма цифр. Поэтому числа, получаемые друг из друга перестановкой цифр, имеют одинаковый остаток от деления на 9, т. е. их разность делится на 9. Поэтому и сумма цифр разности, равная N, должна делиться на 9, откуда N ^ 9. Значение N = 9 получается, например, так: 9 012 345 678 - 8 901 234 567 = 111 111 111. 8.7. Продлим отрезок СМ до пересечения с ВК в точке L (рис. 61). Поскольку в треугольнике внешний угол равен сумме двух несмежных с ним внутренних, то ZKLM = ZAMC - ZBKM = 2ZABC - ZABC = ZABC, откуда МК = ML. Далее, АВ = ВС, ZLCB = ZKLM - - ZLBC = ZABC - ZKBC = ZABK, ZBAK = ZBKM - - ZABK = ZKLM - ZBCL = ZLBC; поэтому треугольники ABK и BCL равны. Значит, ВК = CL = = CM + ML = CM + МК, что и требовалось доказать. Рис. 61 IV ЭТАП. 2006/2007 учебный год 151
8.8- Ответ. 120. Обозначим мальчиков М19 М29 ..., М15, а девочек — Dl9 D29 ..., D15 так, чтобы Мх - Dl9 М2 - D29 ..., М15 - D15 было единственным разбиением на пары из условия задачи. Предположим, что каждый мальчик позвонил хотя бы двум девочкам. Нарисуем стрелку от каждой девочки Dt к мальчику Mi9 с которым она находится в паре, а от каждого мальчика Mt — к другой (отличной от Dt) девочке, которой он звонил. Тогда от каждого ребенка идет по стрелке. Если мы будем двигаться по стрелкам (начав от произвольной девочки), то рано или поздно мы попадем к девочке, которая уже встречалась в строящейся цепочке. Таким образом, найдется цикл Dix-+ М41-> Д2-^ Mi2~^ ■•• -* Dik -* —► Mik-^ Dtl (здесь k > 1, так как от Mik идет стрелка не к Dik). Объединим в этом цикле каждого мальчика Mid с девочкой Did + l9 к которой от него идет стрелка (Mik мы объединим с Dtl); остальные пары оставим без изменения. Мы получили другое разбиение на пары, что противоречит условию. Следовательно, найдется мальчик Mi9 который звонил ровно одной девочке Dt. Тогда, если отбросить эту пару, число звонков уменьшится не больше чем на 15 — максимально возможное количество звонков этой девочке. После этого снова найдется мальчик, сделавший ровно один звонок одной из оставшихся девочек. Отбросив эту пару, уменьшим количество звонков не более чем на 14, и т. д. Итого было сделано не более 15 + 14 + ... + 2 + 1 = 120 звонков. Ровно 120 звонков получается, например, если девочке Dt звонили мальчики М19 М2, ..., Mt (i= 1, 2, ..., 15). ф 9 класс 9.1. Ответ. 0. Положим Д (х) = х(х- 2007), f2 (x) = (х-1)(х- 2006), ••■» Аоо4 (х) = (х - 1003) (х - 1004). Все эти многочлены попарно различны, потому что у них разные корни, но коэффициент при х у каждого из них равен -2007. Поэтому разность любых двух данных трех- XXXIII Всероссийская математическая олимпиада школьников
членов равна константе, отличной от 0, и ни одно из уравнений fn (x) = fm (x) не имеет решений. 9.2. Ответ. Не существуют. См. решение задачи 8.3. 9.3. По условию имеется ровно 26 девочек, знакомых хотя бы с одним мальчиком из 25. Выберем произвольно мальчика М. Для оставшихся 24 мальчиков ровно 25 девочек знакомы хотя бы с одним из них. Тогда девочка D, не входящая в эти 25, знакома только с одним мальчиком М. Значит, для каждого мальчика есть девочка, знакомая только с ним. Обозначим мальчиков М19 ..., М25; обозначим девочку, знакомую только с Mi9 через Dt. Рассмотрим оставшуюся девочку (отличную от Dl9 ..., D25). Если она знакома менее чем с 16 мальчиками, то незнакома не менее чем с 10 мальчиками. Так как каждая из остальных 25 девочек знакома ровно с одним мальчиком, то для этих k (k > 10) мальчиков найдется ровно k знакомых хотя бы с одним из них девочек. Это противоречит условию, что таких девочек должно быть k + 1. 9.4. Пусть ax^bx^c, a2^b2^ с — длины сторон данных треугольников, а — их общий наименьший угол, otx — наименьший угол в построенном треугольнике со сторонами ах + а29 Ьх + Ь2, с + с. В этом треугольнике ах + а2 — наименьшая сторона, поэтому угол щ лежит против нее и является острым. Рассмотрим треугольник AML с углом а при вершине А и боковыми сторонами AM = Ьх + b29 AL = с + с (рис. 62). ^V ^ Рис. 62 А с в с L Покажем, что ах ^ а; для этого достаточно доказать, что ML ^ ах + а2. Пусть В и С — точки на сторонах AL и AM соответственно такие, что АС = bu AB = с. Выберем точку К так, что BCKL — параллелограмм. Тогда треугольники ABC и СКМ соответственно равны исходным треугольникам, поэтому в треугольнике MKL получаем МК = а2У KL = аи откуда по нера- IV ЭТАП. 2006/2007 учебный год 153
венству треугольника ML ^ ai + a29 что и требовалось. Отсюда SAML = -(Ь1 + Ь2) (с + с) sin ах > (Ьх + Ь2) с sin а = = 2 ^ Ьхс sin а + - Ъ2с sin а = 2 (SACB + SCMi^), что и требовалось доказать. 9.5. См. решение задачи 8.5. 9.6. Пусть описанные окружности с треугольников BKD и CLD вторично пересекаются в точке М (рис. 63). Рис. 63 Пользуясь тем, что четырехугольники BKDM и CLDM вписанные, получаем: /.ВМС = ZBMD + ZCMD = = (180° - ZBKD) + (180° - ZCLD) = ZKBD + ZKDB + + ZLDC + ZLCD = \ (ZABD + ZADB + ZADC + ZACD) = = ± (ZABD + 180° + ZACD) = 90° + £ (ZARD + ZACZ>). Величина угла ВМС фиксирована, поэтому точка М лежит на фиксированной окружности с хордой ВС. Замечание. В решении используется, что точки А и М лежат по разные стороны от ВС. Это нетрудно вывести из того, что углы BKD и CLD — тупые. 9.7. Для решения задачи докажем вначале лемму. Лемма. Если для некоторого натурального п число п3 является точным квадратом, то число п также является точным квадратом. Доказательство. Пусть простое число р входит в разложение числа п на простые множители в степени t9 тогда р входит в разложение числа п3 в степени 3£. По условию 3£ четно, поэтому t четно. В силу произвольности р получаем, что п — точный квадрат. Лемма доказана. ■ 154 XXXIII Всероссийская математическая олимпиада школьников
Пусть в прогрессии с разностью d > 0 содержится куб натурального числа т. Если т3 не является точным квадратом, то искомое число найдено. Иначе т3 — точный квадрат и, согласно лемме, т — точный квадрат, т- k2. Вместе с т3 прогрессия содержит точный куб А = (т + md2)3, поскольку А = т3 + ld9 где I — натуральное. Докажем, что А не является точным квадратом. Пусть это не так, тогда по лемме т + md2 = k2 (I + d2) — точный квадрат. Отсюда 1 + d2 — точный квадрат, 1 + d2 = х2 для натурального х. Получаем 1 = (х - d) (x + d)9 что невозможно. 9.8. Для решения задачи докажем вначале лемму. Лемма. Среди любых 13 подряд идущих натуральных чисел нельзя выбрать более четырех так, что никакие два из них не различаются на 4, 5 или 9. Доказательство. Разобьем 13 чисел а, а + 1, ..., а + 12 на 9 групп (из одного или двух чисел) и запишем группы по кругу в следующем порядке: {а + 4}, {а, а + 9}, {а + 5}, {а + 1, а + 10}, {а + 6}, {а + 2, а + 11}, {а + 7}, {а + 3, а + 12}, {а + 8}. Если выбрано 5 или более чисел, то некоторые два из них окажутся в одной группе или в соседних группах. Однако из двух соседних групп можно выбрать не более одного числа. Лемма доказана. ■ Отметим 4 средних числа 599, 600, 601, 602, а все остальные числа от 1 до 1200 разобьем на (1200 - 4) : 13 = 92 группы по 13 последовательных чисел. Из леммы следует, что в группах по 13 чисел можно выбрать не более 92 • 4 = (372 — 4) чисел требуемым в условии образом. А по условию было выбрано 372 числа. Значит, все четыре отмеченных числа были выбраны, а среди них и число 600. ф 10 класс 10.1. Обозначим коробки В19 ..., Б25. По условию общее число цветов равно 26. Если рассмотреть все коробки, кроме Bi9 то общее число цветов в них равно 25. Следовательно, есть цвет, присутствующий только в коробке Bi9 назовем его Ct. Поскольку общее число цветов 26, остался ровно один цвет С, отличный от всех Ct. Если в какой-то коробке Bk нет шариков этого цвета, то в ней есть толь- IV ЭТАП. 2006/2007 учебный год 155
ко шарики цвета Ck9 что противоречит условию (в Bk должны быть шарики двух цветов). Значит, шарики цвета С есть во всех коробках. у 10.2. Разделив на хпк и обозначив а = —, перепишем исходное неравенство в виде (1 - ak)n < (1 - an)k. Однако 0<l-afe<l-an<l (так как 0 < a < 1), поэтому (1 - ak)n < (1 - ап)п < (1 - an)k9 что и требовалось доказать. 10.3. Ответ, п = 2008. Пусть набор А содержит множества А19 А2, ..., А2007- Включим в набор В множества В1 = А19 В2 = А29 ..., Я2007 = А2007> ^2008 = A1UA2U...U А2007 U {х} (здесь х — элемент, не принадлежащий ни одному из множеств At). Тогда все множества Bt разные, и А = В2оо8 П В, для 4=1,2, ..., 2007. Далее докажем, что для множеств Ах = {1}, А2 = {1, 2}, А3 = {1, 2, 3}, ..., А2Оо7 = {!> 2, ..., 2007} в любом наборе В, удовлетворяющем условию, не менее 2008 множеств. Из условия вытекает, что для каждого i = 1, 2, ..., 2006 среди множеств набора В найдется множество В;, содержащее Ai9 но не содержащее At + x (иначе At не может быть пересечением множеств набора В). Заметим, что Bt также содержит все множества А19 ..., Д и не содержит ни одного из множеств Ai + U ..., Ап; поэтому все множества Bt разные. Кроме того, среди множеств набора В найдутся два множества В2007 и В20089 содержащие А2Оо7« Очевидно, множества В19 В29 ..., В2Оо8 различны. Замечание. Другим примером множеств, для которых п > 2008, являются множества At = A\{£}, где А = {1, 2, ..., 2007} (i = 1, 2, ..., 2007). 10.4. Середину меньшей дуги DE обозначим через К (рис. 64). Тогда Z.MEK = ZMED - ZKED = ZMED - - ZKCD = I ZAED - \ ZECD = \ (ZAED - ZECD) = 2 2^ = - ZCDE = ZEDN (в частности, точка К лежит внут- ри угла MED9 так как ZMED - ZKED > 0). Аналогично, ZMDK = ZDEN. 156 XXXIII Всероссийская математическая олимпиада школьников
Рис. 64 Пусть прямые DK и ЕК пересекают описанную окружность треугольника DEM соответственно в точках Р и Q (рис. 65). Так как DK = ЕК, то ZKED = = ZKDE = ZPDE = ZPQE, откуда PQ || DE. Далее, ZQPM = ZQEM = ZKEM = ZEDN и, аналогично, ZPQM = ZDEN. Отсюда вытекает, что треугольники DEN и PQM гомотетичны, причем точка К является центром гомотетии (как точка пересечения прямых QE и PD). Следовательно, прямая MN проходит через точку К. Рис. 65 В X со. Рис. 66 10.5. Ответ. 9. См. решение задачи 8.6. 10.6. Первое решение. Рассмотрим общую внешнюю касательную I (отличную от ВС) для окружностей сох и со2, вписанных в треугольники ABD и ACD (рис. 66). Из равенства окружностей сох и со2 имеем: I || ВС. Рассмотрим гомотетию с центром А, переводящую прямую I в прямую ВС. При выполнении этой гомотетии IV ЭТАП. 2006/2007 учебный год 157
окружность С0х перейдет в окружность, отличную от с^, вписанную в угол BAD и касающуюся прямой ВС, т. е. в соответствующую вневписанную окружность треугольника ABD. Аналогично, со2 перейдет во вневписанную окружность треугольника ACD. Отсюда вытекает утверждение задачи, так как при гомотетии равные окружности переходят в равные. Второе решение. Пусть высота треугольника ABC, опущенная из вершины А, равна h, площади треугольников ABD и ACD равны Sl9 S2, радиусы их вписанных окружностей равны г, радиусы вневписанных окружностей (о которых идет речь в задаче) — г1 и г2 соответственно. Положив АВ = с, AD = d, BD = х, на- __, 1 с + d - х c + d + x ходим 2S1 = r1(c + d-x) => - = —— = —^ х 12 — 2 —- = — —. Аналогично можно получить, что 2«Sj г h — = - - |, откуда гх = г2. 10.7. Докажем вначале лемму. Лемма. Пусть р — минимальное простое число, на которое не делится п (п - 1), при этом п > 6. Тогда р < п - 1. Доказательство. Рассмотрим числа п — 2, п - 3, я-4. Среди них не больше одного числа, делящегося на 3, и не больше одного числа, являющегося степенью двойки (так как п — 4 > 2). Таким образом, у одного из них есть нечетный простой делитель д, больший 3. Числа п и п — 1 не могут делиться на q9 поэтому p^q^n-2. Ш Вернемся к решению задачи. Ясно, что среди рассматриваемых чисел есть число А = п (п — 1) + 1. Из леммы следует, что среди них также есть число В = п (п - 1) +р. Тогда НОД (А, Б) = НОД (А, В-А) = = НОД (А, р — 1) = 1, так как согласно выбору р любой простой делитель числа р - 1 является делителем числа п (п — 1) = А — 1 и потому взаимно прост с А. Итак, НОД уже двух чисел А и Б равен 1, откуда и следует утверждение задачи. 10.8. Ответ. 2360. Первое решение. Докажем вначале лемму. Лемма. Пусть k — натуральное число. В прямоугольнике т х п (т, п> к) изначально все клетки белые. 158 XXXIII Всероссийская математическая олимпиада школьников
За один ход можно выбрать ряд (строку или столбец) и перекрасить клетки этого ряда так9 чтобы в нем оказалось не более k черных клеток (остальные клетки белые). Тогда в любой момент в прямоугольнике не менее (т - k) (n - k) белых клеток. Доказательство. Применим индукцию по числу клеток в прямоугольнике. Если т = k или п = fc, утверждение очевидно. Пусть т, n > k, и для всех рассматриваемых прямоугольников с менее чем тп клетками утверждение верно. Рассмотрим ситуацию после нескольких перекрашиваний. Пусть, для определенности, последней перекрашивалась некоторая строка; тогда в этой строке не менее (п — k) белых клеток. Исключив из рассмотрения эту строку, можно считать, что все перекрашивания, кроме последнего, происходили в прямоугольнике (т — 1) х п. По предположению индукции, в этом прямоугольнике не менее (т - 1 - k) (n - k) белых клеток. Итого в прямоугольнике т х п не менее (п - k) + (т - 1 - К) (п - К) - = (т - к) (п - к) белых клеток. Лемма доказана. ■ Перейдем к решению задачи. Положим п = т = 15. Пусть после нескольких ходов в таблице стоит аь чисел, равных i (I ^ i ^ 15), Применим лемму, считая черными клетки с числом 15. При этом k = 1, и значит, а15 ^ 152 - 142, Далее, применим лемму, считая черными клетки с числами 15 и 14. При этом k = 2, поэтому а15 + а14 ^ 152 — 132. Рассуждаем так и далее: применяем лемму, считая черными клетки с числами 15, 14, ..., s + 1 (s = 14, 13, ..., 0), При этом k = 15 — s, поэтому а15 + ... + as + 1 < 152 - s2. Складывая все эти неравенства, получаем 15а15 + 14а14 + 13а13 + ... + 2а2 + ах ^ ^ 15 • 152 - (142 + ... + О2) = 2360. Слева в этом неравенстве стоит, очевидчо, сумма всех чисел в таблице. Легко убедиться, что сумма всех чисел окажется равной 2360, если действовать, например, следующим образом. Числа будем записывать в порядке убывания: в столбцах — сверху вниз, в строках — слева направо. Будем последовательно изменять первый (слева) столбец, первую (сверху) строку, второй столбец, вторую строку, ..., 15-й столбец, 15-ю строку. Тогда в первой строке и первом столбце будет стоять IV ЭТАП. 2006/2007 учебный год 159
число 15, в остальных клетках второй строки и второго столбца — число 14 и т. д., т. е. все неравенства из предыдущего рассуждения обратятся в равенства. Второе решение. Приведем другое доказательство оценки. Будем называть столбцы и строки линиями. Рассмотрим алгоритм расстановки чисел, дающий максимальную возможную сумму. Мы немного упростим этот алгоритм. При этом сумма всех чисел не уменьшится, т. е. алгоритм останется оптимальным. Заметим, что делать два хода в одну и ту же линию не имеет смысла — второй ход уничтожает все числа, появившиеся в результате первого; поэтому первого хода можно просто не делать. Далее, если в какую-то линию не было хода, то можно было сделать произвольный ход в нее перед всеми остальными ходами — сумма чисел от этого не уменьшится. Таким образом, можно считать, что в каждую линию был сделан ровно один ход. Переставим строки таблицы так, чтобы первый горизонтальный ход был в первую (верхнюю) строку, второй — во вторую и т. д. Аналогично, можно считать, что столбцы, в которые делаются вертикальные ходы, идут слева направо. Назовем вкладом некоторого хода сумму чисел, поставленных на доску во время него и впоследствии не стертых. Тогда ясно, что сумма всех чисел таблицы — это сумма вкладов всех ходов. Рассмотрим некоторый вертикальный ход. Заменим порядок чисел в нем на убывающий порядок сверху вниз. Тогда вклад этого хода не уменьшился, так как не будут стерты ровно несколько верхних чисел этого столбца, а их сумма в новой расстановке — максимальная возможная. Таким образом, можно считать, что при каждом вертикальном ходе числа расставлялись по убыванию сверху вниз. Аналогично, при каждом ходе в строку числа можно расставлять по убыванию слева направо. Таким образом, за все ходы в каждую клетку будут поставлены два числа — ее номер справа в строке и ее номер снизу в столбце. Тогда в этой клетке будет стоять не больше чем максимальное из этих чисел. Заметим, что в примере (из первого решения) ровно так и происходит, поэтому он и дает максимальную сумму. XXXIII Всероссийская математическая олимпиада школьников
f) 11 класс 11-1- См. решение задачи 10-1. 11.2. Если х19 х2 — абсциссы вершин парабол у = f1 (x) и у = f2 (х), то Д (х) = а1(х- xxf + dl9 f2 (x) = a2 (x - x2f + + d2 (al9 a2 Ф 0). Тогда f[ (x) f2 (x) = 4axa2 (x - хг) х x (x — x2)9 и исходное неравенство переписывается в виде 4ага2 (х - хх) (х - х2) > >\аг(х- хг)2 + dl\ + \a2(x- х2)2 + d21. Подставляя в это неравенство х = х19 получаем \d1\ + |а2 (х1 - х2)2 + d21 ^ 0, откуда dx - 0 и а2 (хх - х2у + d2 = 0. По аналогичным соображениям d2 = 0, поэтому из последнего равенства имеем хх — х2. Так как 4axa2 (х - jcx)2 ^ 0 при всех х, то ага2 > 0. Поэтому Д (х) f2 (х) = аха2 (х - хг)* = {^аха2 (х - xtff, что и требовалось доказать. 11-3- Обозначим середины сторон АВ и АС через Сг и Вг соответственно, а точки пересечения медиан треугольников АВМу АСМ — через Gb9 Gc соответственно (рис. 67). Тогда AAGCBX = 180° - ZAGCM = ZABM9 так как AGCMB — вписанный четырехугольник. Рис.67 в М С Далее, ZABM = /.АСХВ19 так как С1В1 \\ ВС. Значит, ZAGCB1 = /AC1B1. Следовательно, четырехугольник AC1GCB1 вписан, т. е. точка Gc лежит на описанной окружности ААВ1С1. Аналогично, Gb лежит на описанной окружности ДАВ^х. Таким образом, точки A, Cl9 Gb9 Gc9 Вг лежат на одной окружности. Далее, GcGb \\ BxCl9 так как b =2= с. Получаем, что СгВгОсОь — вписанная IV ЭТАП. 2006/2007 учебный год 161
трапеция, значит, ZGbC1B1 = Z.GcBxCl9 т. е. МВ1 = МС1, что и требовалось доказать. 11.4. Ответ, п2 = 1 + 3 + ... + (2п — 1) при нечетном п, п (п + 1) = 2 + 4 + ... + 2п при четном п. Обобщим задачу. Пусть в начале игры на столе лежат купюры достоинством М1 > М2 > ... > М2п. Покажем индукцией по п, что игрок, делающий последний ход (назовем этого игрока последним), всегда может получить не менее М2 + М4 + ... + М2п тугриков, а игрок, делающий предпоследний ход (назовем этого игрока предпоследним)у — не менее Мх + Мг + ... + М2п_г тугриков. Сразу заметим, что сумма этих чисел равна суммарному достоинству всех купюр; поэтому при оптимальной игре игроки получат ровно такое количество. База при п = 1 очевидна. Пусть утверждение верно для п = k — 1, докажем его для п = ft. Пусть ft нечетно, тогда ходить должен последний игрок. Он может снять купюры Мг и М2; тогда в оставшейся игре он получит не меньше М4 + М6 + ... + М2Л, а за этот ход он получит М2; поэтому у него окажется не меньше М2 + (М4 + ... + M2k) тугриков, что и требовалось. Покажем, что при любом ходе последнего предпоследний получит не меньше Мх + М3 + ... + M2k _ г тугриков. Пусть последний взял купюры Mt > Mf, перенумеруем оставшиеся на столе купюры по убыванию: L1 > L2 > ... > L2k_2. Тогда по предположению индукции предпоследний сможет дальше играть так, чтобы получить не меньше чем Lx + L3 + ... + L2k_s. Поэтому нам достаточно показать, что Мг + (Lx +L3 + ... + L2k_3) > Мг + М3 + ... + М2А_!. (1) Пусть 1 ^ d ^ ft. Покажем, что в левой части неравенства присутствует не меньше d купюр из 2d - 1 наибольших купюр М19 М2, ..., M2d_1; это будет означать, что d-e по величине слагаемое слева не меньше M2d-u откуда и следует (1). Действительно, если i < 2d — 1, то M2d_1 = L2d_ 1 и в левой части содержится d купюр М19 М3, ..., M2d_x. Пусть i > 2d - 1. Тогда среди М19 М2, ..., Мы_х содержатся купюры Ll9 L2, ..., L2d_39 из которых d - 1 содержится в левой части; кроме того, в ней содержится Miy что и требовалось. Аналогично, если ft четно, то ходит предпоследний. 162 XXXIII Всероссийская математическая олимпиада школьников
Если он снимет купюры М2 и М3, то он получит не меньше М3 + (Мг + М5 + М7 + ... + М2к_ х), что и требовалось. Далее, пусть предпоследний снял купюры М; > Mj, оставив на столе купюры L1 > L2 > ... > L2k_2; тогда по предположению индукции последний сможет за дальнейшую игру получить не меньше чем L2 + L4 + ... + L2k_2. Поэтому достаточно показать, что Mt + (L2 + L4 + ... + L2k_2) > M2 + M4 + ... + M2k. Аналогично предыдущему случаю легко показать, что в левой части неравенства присутствует не меньше d купюр из наибольших 2d купюр М19 ..., M2d9 откуда сразу следует требуемое. 11-5- Ответ. При всех четных п. Первое решение. Если п четно, то 1П + (-1)п + 0л = 2 — простое число. Пусть х — целое, an — нечетное. Представив х в виде 3£ или 3* ± 1 для целого t9 в любом случае получаем, что хп — х делится на 3. Кроме того, хп — х четно. Сумма чисел а, Ь, с по условию равна нулю, поэтому ап + Ъп + сп = (ап - а) + (Ьп - Ъ) + (сп - с) делится на 2 • 3 и, значит, не является простым. Второе решение. Докажем, что при нечетном п число А = ап + Ъп + сп не является простым. Пусть А — простое, тогда п > 1 и числа а, Ь, с отличны от 0. Поскольку Ъп + сп делится на Ъ + с = -а, то число А делится на а. Аналогично, число А делится на Ъ и на с. Отсюда следует, что каждое из чисел а, Ь, с равно одному из чисел ±1, ±А. Так как среди чисел а, Ь9 с нет двух противоположных (иначе третье было бы нулем), то среди них найдутся два равных числа; пусть они равны d, тогда третье число равно -2d. Получаем, что А = 2dn - 2ndn — четное число, делящееся на 2п — 2 > 2. Противоречие. 11-6- Ответ. 6. а) Два возможных примера из шести точек показаны на рисунке 68 (существуют и другие примеры). Предположим, что точек хотя бы семь; тогда найдутся три точки одного цвета. Согласно условию, они не лежат на одной прямой, поэтому они образуют треугольник. Рис- 68 IV ЭТАП. 2006/2007 учебный год 163
Рассмотрим треугольник ABC наименьшей площади, у которого все три вершины одноцветны (пусть они синие; рис. 69). Тогда 1 внутри него нет синих точек. На каждой из его сторон ВС, АС> АВ есть по точке другого цвета (обозначим их А19 В19 Сг соответственно). Если все они одноцвет- Рис. 69 ны, то образовался одноцветный треугольник меньшей площади, что невозможно. Поэтому можно без ограничения общности считать, что Аг и Сг — желтые, а Вг — зеленая. Далее, на отрезке А1С1 есть не желтая точка В2. По замеченному выше она не может быть синей — значит, она зеленая. Тогда на отрезке ВХВ2 есть не зеленая точка X. Она также не синяя — значит, она желтая. Но тогда треугольник АХСХХ имеет желтые вершины и площадь, меньшую площади АВС9 — противоречие. 11.7. Ответ. Нельзя. Предположим, что хороший тетраэдр объема V с площадью поверхности S помещен внутри хорошего параллелепипеда объема V'9 площади граней которого равны Sl9 S2, S3 (Si > S2> S3), а соответствующие высоты равны hl9 h29 hs. По условию V=S и V = 2 (Sx + S2 + S3). Впишем в тетраэдр сферу со радиуса г. Так как V = - Sr, то г = 3. Сфера со лежит между парой па- о раллельных плоскостей, содержащих грани параллелепипеда, поэтому hx > 2r = 6. Отсюда V = SA > 6SX > 2 (Sx + S2 + S3) = V. Противоречие. 11.8. Первое решение. Возведя в квадрат обе части исходного неравенства и преобразовав, перепишем его в виде ххх ' 1 чп + * s = _ 12- п (X "f~ Х-^) ... (X "f~ JCj ~\~X2 Обозначим хх+х2У 164 XXXIII Всероссийская математическая олимпиада школьников
г +x2) (1 + хг +x2 +xs)' и = * n x2 + ... Уп + i- Тогда мы имеем уг • i/2 * ••• # Уп + i = & и По неравенству о средних получаем: J = что и требуется. Второе решение. Применим индукцию по п. При п = 1 исходное неравенство имеет вид 1 + хг ^ у[4х\ <= Предположив, что неравенство верно для п переменных, докажем его для п + 1 переменных. По пред- положению индукции для п чисел *1 Хп + 1 верно неравенство: 1 + Хл IV ЭТАП. 2006/2007 учебный год 165
Домножив неравенство на (1 + xl)n + 1y получим (1 + хг) (1 + хх + х2) ... (1 + х1 + х2 + ... + хп + 1 Для доказательства перехода индукции остается показать, что (п + l)n + 1 (1 + х)п + 2 > (п + 2)п + 2 х для любого положительного х = jcx. По неравенству о среднем арифметическом и среднем геометрическом для п + 2 чисел n + 1 слагаемое Возводя в (п + 2)-ю степень, получаем требуемое. V ЭТАП (заключительный) УСЛОВИЯ ЗАДАЧ ® 8 класс 8-1- Даны числа а, Ь, с. Докажите, что хотя бы одно из уравнений х2 + (а - Ъ) х + (Ь - с) = 0, jc2 + (b - с) х + + (с — а) = 0, jc2 + (с — а) х + (а — Ь) = 0 имеет решение. (О. Подлипский) 8.2. В клетках таблицы 10 х 10 произвольно расставлены натуральные числа от 1 до 100, каждое по одному разу. За один ход разрешается поменять местами любые два числа. Докажите, что за 35 ходов можно добиться того, чтобы сумма любых двух чисел, стоящих в клетках с общей стороной, была составной. (Н. Агаханов) 8.3. На стороне ВС ромба ABCD выбрана точка М. Прямые, проведенные через М перпендикулярно диагоналям BD и АС9 пересекают прямую AD в точках Р и Q соответственно. Оказалось, что прямые РБ, QC и AM пересекаются в одной точке. Чему может быть равно отношение ВМ : МС? (С. Берлов, Ф.Петров, А. Ако- пян) 8.4. Фокусник Арутюн и его помощник Амаяк собираются показать следующий фокус. На доске нарисована 166 XXXIII Всероссийская математическая олимпиада школьников
окружность. Зрители отмечают на ней 2007 различных точек, затем помощник фокусника стирает одну из них. После этого фокусник впервые входит в комнату, смотрит на рисунок и отмечает полуокружность, на которой лежала стертая точка. Как фокуснику договориться с помощником, чтобы фокус гарантированно удался? (А. Акопян, И. Богданов) 8.5. От Майкопа до Белореченска 24 км. Три друга должны добраться: двое из Майкопа в Белореченск, а третий из Белореченска в Майкоп. У них есть один велосипед, первоначально находящийся в Майкопе. Каждый из друзей может идти (со скоростью не более 6 км/ч) и ехать на велосипеде (со скоростью не более 18 км/ч). Оставлять велосипед без присмотра нельзя. Докажите, что через 2 ч 40 мин все трое друзей могут оказаться в пунктах назначения. Ехать на велосипеде вдвоем нельзя. (Фольклор) 8.6. Через точку / пересечения биссектрис треугольника ABC проведена прямая, пересекающая стороны АВ и ВС в точках М и N соответственно. Треугольник BMN оказался остроугольным. На стороне АС выбраны точки К и L так, что ZILA = ZIMB, ZIKC = ZINB. Докажите, что AM + KL + CN = АС. (С. Берлов) 8.7. Для натурального п (п > 3) будем обозначать через п? (n-вопросиал) произведение всех простых чисел, меньших п. Решите уравнение п? = 2п + 16. (В. Сен- деров) 8.8. В таблице 10 х 10 расставлены числа от 1 до 100: в первой строчке — от 1 до 10 слева направо, во второй — от 11 до 20 слева направо и т. д. Андрей собирается разрезать таблицу на прямоугольники 1x2, посчитать произведение чисел в каждом прямоугольнике и сложить полученные 50 чисел. Он стремится получить как можно меньшую сумму. Как ему следует разрезать квадрат? (А. Бадзян) 0 9 класс 9.1. Приведенные квадратные трехчлены / (х) и g (x) таковы, что уравнения / (g (х)) = 0 и g (f (x)) = 0 не имеют действительных корней. Докажите, что хотя бы одно из уравнений / (/ (х)) = 0 или g (g (x)) = 0 тоже не имеет действительных корней. (С. Берлов) V ЭТАП. 2006/2007 учебный год 167
9.2. На доске написали 100 дробей, у которых в числителях стоят все числа от 1 до 100 по одному разу и в знаменателях стоят все числа от 1 до 100 по одному разу. Оказалось, что сумма этих дробей есть несократимая дробь со знаменателем 2. Докажите, что можно поменять местами числители двух дробей так, чтобы сумма стала несократимой дробью с нечетным знаменателем. (Н. Агаханов, И. Богданов) 9.3. Два игрока по очереди проводят диагонали в правильном (2п + 1)-угольнике (п > 1). Разрешается проводить диагональ, если она пересекается (по внутренним точкам) с четным числом ранее проведенных диагоналей (и не была проведена раньше). Проигрывает игрок, который не может сделать очередной ход. Кто выигрывает при правильной игре? (К. Сухов) 9.4. В треугольнике ABC проведена биссектриса ВВХ. Перпендикуляр из Вг на ВС пересекает дугу ВС описанной окружности треугольника ABC в точке К. Перпендикуляр из точки В на АК пересекает АС в точке L. Докажите что точки К, L и середина дуги АС (не содержащей точку В) лежат на одной прямой. (В. Астахов) 9.5. В каждой вершине выпуклого 100-угольника написано по два различных числа. Докажите, что можно вычеркнуть по одному числу в каждой вершине так, чтобы оставшиеся числа в любых двух соседних вершинах были различными. (Ф.Петров) 9.6. Дан остроугольный треугольник ABC. Точки М и N — середины сторон АВ и ВС соответственно, точка Н — основание высоты, опущенной из вершины В. Описанные окружности треугольников AHN и СНМ пересекаются в точке Р (Р Ф Н). Докажите, что прямая РН проходит через середину отрезка MN. (В. Филимонов) 9.7. В таблице 10 х 10 расставлены числа от 1 до 100: в первой строчке — от 1 до 10 слева направо, во второй — от 11 до 20 слева направо и т. д. Андрей собирается разрезать таблицу на прямоугольники 1x2, посчитать произведение чисел в каждом прямоугольнике и сложить полученные 50 чисел. Он стремится получить как можно меньшую сумму. Как ему следует разрезать квадрат? (А. Бадзян) 9.8. Дима посчитал факториалы всех натуральных чисел 168 XXXIII Всероссийская математическая олимпиада школьников
от 80 до 99, нашел числа, обратные к ним, и напечатал получившиеся десятичные дроби на 20 бесконечных ленточках (например, на последней ленточке было напечатано число —у = 0,00... 0010715...). Саша 155 нулей хочет вырезать из одной ленточки кусок, на котором записано N цифр подряд и нет запятой. При каком наибольшем N он сможет это сделать так, чтобы Дима не смог определить по этому куску, какую ленточку испортил Саша? (А. Голованов) 3 Ю класс 10.1. Грани куба 9x9x9 разбиты на единичные клетки. Куб оклеен без наложений бумажными полосками 2x1 (стороны полосок идут по сторонам клеток). Докажите, что число согнутых полосок нечетно. (А. Полянский) 10.2. Дан многочлен Р (х) = аохп + аххп~х + ... + ап_гх + ап. Положим т = min {а0, а0 + а19 ..., а0 + аг + ... + ап}. Докажите, что Р (х) > тхп при х > 1. (А. Храброе) 10.3. В треугольнике ABC проведена биссектриса ВВг. Перпендикуляр из Вх на ВС пересекает дугу ВС описанной окружности треугольника ABC в точке К. Перпендикуляр из точки Б на АК пересекает АС в точке L. Докажите что точки К, L и середина дуги АС (не содержащей точку В) лежат на одной прямой. (В. Астахов) 10.4. Фокусник с помощником собираются показать такой фокус. Зритель пишет на доске последовательность из N цифр. Помощник фокусника закрывает две соседние цифры черным кружком. Затем входит фокусник. Его задача — отгадать обе закрытые цифры (и порядок, в котором они расположены). При каком наименьшем N фокусник может договориться с помощником так, чтобы фокус гарантированно удался? (К. Кноп, О. Леонтьева) 10.5. Дан набор из п > 2 векторов. Назовем вектор набора длинным, если его длина не меньше длины суммы остальных векторов набора. Докажите, что если каждый вектор набора — длинный, то сумма всех векторов набора равна нулю. (Н.Агаханов) V ЭТАП. 2006/2007 учебный год 169
10.6. Две окружности Шх и со2 пересекаются в точках А и Б. Пусть PQ и RS — отрезки общих внешних касательных к этим окружностям (точки Р и R лежат на co^ точки Q и S — на со2). Оказалось, что RB || PQ. Луч RB вторично пересекает со2 в точке W. Найдите отношение RB : BW. (С. Берлов) 10.7. У выпуклого многогранника одна вершина А имеет степень 5, а все остальные — степень 3 (степенью вершины называется количество выходящих из нее ребер). Назовем раскраску ребер многогранника в синий, красный и лиловый цвета хорошей, если для любой вершины степени 3 все выходящие из нее ребра покрашены в разные цвета. Оказалось, что количество хороших раскрасок не делится на 5. Докажите, что в одной из хороших раскрасок какие-то три последовательных ребра, выходящие из А, покрашены в один цвет. (Д. Карпов) 10.8. Дима посчитал факториалы всех натуральных чисел от 80 до 99, нашел числа, обратные к ним, и напечатал получившиеся десятичные дроби на 20 бесконечных ленточках (например, на последней ленточке было напечатано число j—: = 0,00... 0010715...). 99! v v ' 155 нулей Саша хочет вырезать из одной ленточки кусок, на котором записано N цифр подряд и нет запятой. При каком наибольшем N он сможет это сделать так, чтобы Дима не смог определить по этому куску, какую ленточку испортил Саша? (А. Голованов) 0 11 класс 11.1. Докажите, что при k > 10 в произведении / (х) = cos х - cos 2x • cos Зх • ... • cos 2kx можно заменить один cos на sin так, что получится функция f1 (х), удовлетворяющая при всех действи- Q тельных х неравенству | fx (х) | ^ k + im (H. Агаханов) 11.2. Вписанная окружность треугольника ABC касается сторон ВС, АС, АВ в точках Al9 Bl9 Сг соответственно. Отрезок ААг вторично пересекает вписанную окружность в точке Q. Прямая I параллельна ВС и 170 XXXIII Всероссийская математическая олимпиада школьников
проходит через А. Прямые А1С1 и А1В1 пересекают I в точках Р и R соответственно. Докажите, что ZPQR = Z.BXQCX. (А. Полянский) 11.3- Фокусник с помощником собираются показать такой фокус. Зритель пишет на доске последовательность из N цифр. Помощник фокусника закрывает две соседние цифры черным кружком. Затем входит фокусник. Его задача — отгадать обе закрытые цифры (и порядок, в котором они расположены). При каком наименьшем N фокусник может договориться с помощником так, чтобы фокус гарантированно удался? (К. КноПу О. Леонтьева) 11.4. В бесконечной последовательности (хп) первый член хх — рациональное число, большее 1, и хп + г = = хп + -—- при всех натуральных п. Докажите, что \-Хп\ в этой последовательности есть целое число. (А. Голованов) 11.5. В каждой вершине выпуклого 100-угольника написано по два различных числа. Докажите, что можно вычеркнуть по одному числу в каждой вершине так, чтобы оставшиеся числа в любых двух соседних вершинах были различными. (Ф. Петров) 11.6. Существуют ли ненулевые числа а, Ь, с, такие, что при любом п > 3 можно найти многочлен вида Рп (х) = хп + ... + ах2 + Ъх + с, имеющий ровно п (не обязательно различных) целых корней? (Н.Агаха- нову И. Богданов) 11.7. Дана треугольная пирамида. Леша хочет выбрать два ее скрещивающихся ребра и на них, как на диаметрах, построить шары. Всегда ли он может выбрать такую пару, что любая точка пирамиды лежит хотя бы в одном из этих шаров? (А. Заславский) 11.8. В стране есть N городов. Некоторые пары из них соединены беспосадочными двусторонними авиалиниями. Оказалось, что для любого k (2 ^ k ^ N) при любом выборе k городов количество авиалиний между этими городами не будет превосходить 2k - 2. Докажите, что все авиалинии можно распределить между двумя авиакомпаниями так, что не будет замкнутого авиамаршрута, в котором все авиалинии принадлежат одной компании. (И. Богданову Г. Челноков) V ЭТАП. 2006/2007 учебный год 171
РЕШЕНИЯ ЗАДАЧ Ф 8 класс 8-1- Так как (6 - с) + (с - а) + (а - Ь) = О, то одно из слагаемых неположительно; пусть для определенности это Ь — с. Тогда дискриминант первого уравнения (а - Ь)2 - 4 (Ь - с) ^ 0, т. е. оно имеет решение. 8.2. Разделим таблицу вертикальной линией т пополам. В одной из половин, например в правой, окажется не более 25 четных чисел. Такое же количество нечетных чисел окажется в левой половине. Меняя местами пары таких чисел разной четности, не более чем за 25 операций можно получить таблицу, у которой в правой половине все числа нечетные, а в левой — четные. Сумма чисел в каждой паре соседних клеток в каждой из половин четное (и большее 2), а потому составное число. Простыми могут оказаться только суммы чисел в соседних клетках lj и гу из разных половин, примыкающих к линии т. Будем теперь менять местами числа только из правой половины так, чтобы суммы чисел в парах клеток (Zy, г,) (у = 1, 2, ..., 10) стали делиться на три. Это можно сделать, так как в правой половине не менее чем по 16 чисел дают остатки 0, 1 и 2 при делении на три, а для требуемой перестановки может потребоваться не более чем по 10 чисел, дающих эти остатки. Полученная не более чем за 25 + 10 = 35 операций таблица — искомая. 8.3. Ответ. \. Обозначим через R точку пересечения РВ9 QC и AM (рис. 70). Заметим, что РМ \\ AC, MQ || В£>, поэтому четырехугольники РМСА и QMBD — параллелограммы. Значит, МС = РАУ ВМ = DQnPQ = PA + AD + DQ = = МС + AD + ВМ = 2ВС. Так как ВС \\PQ и ВС = \ PQ, то ВС — средняя линия треугольника PRQ. Значит, и ВМ — средняя линия треугольника ARP. Тогда МС = РА = 2ВМ. 172 XXXIII Всероссийская математическая олимпиада школьников
8.4. Приведем один из возможных вариантов договоренности. Рассмотрим 2007 дуг, на которые разбили окружность отмеченные точки. Пусть АВ — наибольшая из них (если их несколько, то возьмем любую), и пусть эта дуга лежит по часовой стрелке от точки А (и против часовой — от точки Б). Тогда помощник должен стереть точку А. Покажем, что фокусник сможет указать полуокружность, на которой находилась стертая точка. Войдя в комнату, он увидит окружность, разбитую на 2006 дуг. Ясно, что стертая точка будет находиться на наибольшей из дуг (она уже единственна, так как наибольшая дуга после стирания ее конца увеличилась). Более того, если сейчас наибольшая дуга — СВ (и она находится по часовой стрелке от точки С), то АВ ^ СА (рис. 71). Поэтому, если точка X — середина дуги СВ9 то точка А лежит на дуге СХ. Поэтому фокусник может выделить полуокружность, находящуюся по часовой стрелке от С (она содержит дугу СХ и, следовательно, стертую точку А). 8.5. В течение 24 : (18 + 6) = 1 часа все движутся навстречу друг другу (первый — на велосипеде), при этом первый и третий друзья встретятся и первый передаст велосипед третьему (рис. 72). В этот момент второй, прошедший 6 км, должен остановиться и дожидаться третьего, едущего к нему навстречу на велосипеде. Первый в это время тоже может отдохнуть. Третий через (24 - 6 - 6) : 18 = - часа доедет до стоя- о щего второго и передаст ему велосипед. После этого второй доедет до Белореченска, третий дойдет до Майкопа, а первый — до Белореченска за 1 час. Все- -2—1 Рис. 72 V ЭТАП. 2006/2007 учебный год 173
о го с начала движения прошло 1 + - + 1 часов, т. е. о 2 часа 40 минут. Замечание. Можно показать, что за меньшее время все трое добраться не смогут. 8.6. Опустим из точки / на стороны АВ, ВС, С А перпендикуляры 1Сг, IAU IBi соответственно (рис. 73). Очевидно, эти перпендикуляры равны по длине. Кроме того, АСг = АВг и САг = СВг как отрезки касательных ко вписанной окружности, проведенных из одной Рис. 73 А К Вг L С точки. Тогда и прямоугольные треугольники 1КВ1 и INA1 равны по катету и противолежащему острому углу, поэтому ВгК = AXN. Аналогично, ВгЬ = СгМ. Следовательно, AM + KL + CN = AM + МСг + NAX + + CN = АСг + САХ = АВХ + СВ1 = АС. Замечание. Утверждение задачи верно и в случае, когда треугольник BMN тупоугольный. 8.7. Ответ, п = 7. Нам надо решить уравнение п? = 2п + 16 или равносильное ему nl - 32 = 2 (п - 8). (1) Так как п? не делится на 4, то из (1) следует, что /г-8 нечетно. Пусть п > 9, тогда п - 8 имеет нечетный простой делитель р. Так как р < п, то п? делится на р. Значит, и 32 делится на р, что невозможно. Мы получили, что п < 9 и нечетно. При п = 9 имеем /г? = 210 > 2 • 9 + 16. Число /г = 7 — корень нашего уравнения, а при п = 5 имеем /г? = 6 < 26 = 2/г + 16. 8.8. Пронумеруем прямоугольники разбиения. Пусть в i-м прямоугольнике лежат числа at и bt. Заметим, что _ af + bf (а,-Ь,)2 __ аД = ——J—. Просуммировав эти равенства 174 XXXIII Всероссийская математическая олимпиада школьников
по всем прямоугольникам, получаем, что сумма всех 50 произведений равна Ь\ Ь|о (а50-Ь50)2 Заметим, что первая дробь равна не зависит от разбиения. В числителе же второй дроби каждый квадрат равен либо I2, либо 102 в зависимости от того, горизонтален или вертикален i-й прямоугольник. Поэтому вторая дробь будет максимальна (а итоговая сумма — минимальна) тогда, когда все слагаемые в числителе будут равняться 100, т. е. когда все прямоугольники будут вертикальны (рис. 74). I2 + ... + 1002 т. е. Рис. 74 ф 9 класс 9.1. Если какой-то из трехчленов / (х) или g (x) (скажем, / (х)) не имеет корней, то f (х) > 0 для любого х> поэтому и / (/ (х)) > 0 для любого х> и утверждение доказано. Пусть оба трехчлена имеют корни. Не умаляя общности, можно считать, что минимальное значение / (х) не превосходит минимального значения g (x). Из условия на многочлен g (f (x)) следует, что минимальное значение / (х) больше любого корня уравнения g (х) = 0 (действительно, если g (а) = 0 и в некоторой точке f (хх) < ау то найдется х2, такое, что / (х2) = а; тогда g (f (x2)) = 0, что невозможно). Тогда и минимальное значение g (x) больше любого корня уравнения g (х) = 0. Поэтому уравнение g (g (x)) = 0 не может иметь действительных корней. 9.2. Первое решение. Пусть вначале в сумму входила дробь ^. Докажем, что в исходной сумме найдется такая дробь - с нечетным знаменателем с, что числа а и Ь имеют разную четность. Действительно, дробей с нечетными знаменателями ровно 50, и число а не является числителем ни одной из них. Поэтому V ЭТАП. 2006/2007 учебный год 175
среди числителей таких дробей не больше 49 имеют ту же четность, что и а. Поменяем теперь местами числители a vs. Ъ. Сделаем это в два приема: сначала поменяем числитель у дроби со знаменателем 2 (сумма изменилась на нечетное число а — Ъ половинок и, значит, превратилась в целое число), а затем — числитель дроби со знаменателем с (сумма изменилась на дробь с нечетным знаменателем, т. е. стала дробью с нечетным знаменателем). Второе решение. Пусть вначале в сумму входила дробь ■£-.. Тогда среди числителей встречается либо число 64 х - 32, либо число х + 32; обозначим этот числитель через z/, а соответствующий ему знаменатель через г. Поменяем местами х и у. Тогда дробь со знаменателем 64 изменилась на -, а дробь со знаменателем z 32 изменилась на —; несократимая запись последней дроби имеет нечетный знаменатель. Отсюда, как и в предыдущем решении, получаем, что новая сумма дробей имеет нечетный знаменатель в несократимой записи. 9.3. Ответ. При нечетных п выигрывает второй игрок, при четных п — первый игрок. Заметим, что число вершин по одну сторону от любой диагонали (2/г + 1)-угольника четно, а по другую — нечетно. Поэтому любую диагональ пересекает четное число других диагоналей (2/г + 1)-угольника. Пусть в некоторый момент игры невозможно сделать ход, тогда каждая непроведенная диагональ пересекает нечетное число уже проведенных. Так как любая диагональ пересекает четное число диагоналей, то каждая непроведенная диагональ пересекала бы также нечетное число непроведенных диагоналей. Такая ситуация возможна только тогда, когда непроведенных диагоналей четное число. Действительно, посчитаем для каждой непроведенной диагонали число непроведенных диагоналей, пересекающих ее. В этой сумме каждая пара пересекающихся диагоналей учтена два раза, поэтому сумма четна, а все ее слагаемые нечетны. Значит, их четное число. Таким образом, если общее количество диагоналей 176 XXXIII Всероссийская математическая олимпиада школьников
в многоугольнике нечетно, то выигрывает первый, а если четно — то выигрывает второй. Легко видеть, что в (2/г + 1)-угольнике число диагоналей нечетно при четном п (тогда выигрывает второй) и четно при нечетном п (тогда выигрывает первый). 9.4. Пусть S и Т — основания перпендикуляров, опущенных из точек Вх и В соответственно на ВС и АК (рис. 75). В прямоугольных треугольниках ALT и BSK имеем ZSBK = ZLAT = а как опирающиеся на одну дугу КС; поэтому ZBXLB = ZALT = 90° - а = = ZBKS = ZBKBU т. е. точки В, Bl9 L, К лежат на одной окружности. Отсюда ZBBXK = ZBLK = р, и из прямоугольных треугольников BBXS и KLT получаем ZAKL = ZTKL = 90° - Р = ZB.BS = \ ZABC = \ ZAKC, с* с* что и означает, что KL проходит через середину АС. Рис. 75 9.5. Пусть в любой вершине стоят одни и те же числа а и Ь; тогда достаточно оставить в вершинах с четными номерами число а, а в вершинах с нечетными — число Ъ. Пусть это не так. Тогда пронумеруем вершины по порядку от 1 до 100 так, чтобы в вершинах 1 и 100 стояли разные пары чисел. Покрасим все поставленные числа в красный и синий цвета следующим образом. Числа в первой вершине окрасим в разные цвета. Пусть в fe-й вершине числа а и Ъ окрашены в красный и синий цвета соответственно. Тогда, как нетрудно видеть, в (k + 1)-й вершине можно покрасить числа так, чтобы одноцветные числа в fe-й и (k + 1)-й вершинах различались. Таким образом мы покрасим все числа. При этом в любой паре соседних вершин, кроме (1, 100), одноцветные числа будут различны. Рассмотрим вершины 1 и 100. Если в них равны красные числа и равны синие чис- V ЭТАП. 2006/2007 учебный год 177
ла, то в этих вершинах стоит одна и та же пара чисел, что не так. Пусть синие числа в этих вершинах различны. Тогда, стерев во всех вершинах красные числа, мы получим требуемое. 9.6. Пусть прямая MN вторично пересекает описанные окружности cOj и со2 треугольников AHN и СНМ в точках D и Е> а прямую РН в точке S (рис. 76). Поскольку HN — медиана прямоугольного треугольника ВНС, то HN = CN и ZNHC = ZNCH. Из параллельности хорд ME и НС окружности со2 следует, что четырехугольник МНСЕ — равнобокая трапеция, поэтому НМ = СЕ и /.МНС = ZECH. Следовательно, ZMHN = ZMHC - ZNHC = ZECH - ZNCH = ZECN. СО Рис. 76 Значит, AMHN = AECN по двум сторонам и углу между ними, откуда NE = MN. Аналогично, DM = MN. Обозначим длину отрезков MN9 NE и DM через а, а длины отрезков MS и NS через х и у соответственно. Из вписанности четырехугольников DHNP и МНЕР получаем MS • SE = PS • SH = NS • SD, откуда х (а + у) = у (а + x)y т. е. ах = ay. Таким образом, S — середина MN, что и требовалось доказать. Замечание. Утверждение задачи остается в силе, даже если отказаться от требования остроугольности ААВС. Доказательство в этом случае полностью аналогично изложенному. 9.7. Ответ. На 50 вертикальных прямоугольников (см. рис. 74). См. решение задачи 8.8. 9.8. Ответ. N = 155. Пусть на ленточках, на которых записаны числа — и — (k < 1), нашлось по одинаковому куску из N под- XXXIII Всероссийская математическая олимпиада школьников
ряд стоящих цифр. Домножим числа — и — на степё- К! II ни числа 10 так, чтобы одинаковые куски оказались сразу после десятичной запятой. Дробные части по- лучившихся дробей —— и ±^г- не могут совпадать тт „ 10° 10Ь Действительно, в противном случае число — — = = — целое. Следовательно, числитель последней дроби делится на I. Тогда на I делится и число 10ь. С другой стороны, ни одно число от 81 до 99 не является делителем числа вида 10ь, так как каждое из этих чисел содержит в своем разложении на множители хотя бы одно простое число, отличное от 2 и 5. Рассматриваемые нами дробные части | . *х , , могут быть записаны как обыкновенные дроби со знаменателями ftl и Я, а потому — и как дроби со знаменателем 99!, который делится на все числа 80!, 81!, ..., 99!; следовательно, их разность есть разность двух неравных дробей со знаменателем 99!, и она не мень- ше9§г С другой стороны, две правильные десятичные дроби, у которых совпадают первые N цифр после запятой, отличаются меньше чем на —ттг- Таким образом, 99! К То^- ИЗ Условия слеДУет> что glj > Y^' поэто" му N < 156. Таким образом, куска из 156 знаков всегда достаточно для того, чтобы определить, из какой полоски он вырезан. С другой стороны, на полосках с числами ^у и ^-у есть одинаковые куски по 155 знаков: gg! 155 нулей = Q,00...0010715... -0,00...0000107... = 0,00...00106..., 153 нуля 153 нуля 153 нуля т. е. на обеих полосках есть кусок 00... 0010. 153 нуля V ЭТАП. 2006/2007 учебный год 179
0 10 класс 10.1. Покрасим клетки каждой грани куба в шахматном порядке так, чтобы угловые клетки были черными. При этом каждая грань содержит 41 черную и 40 белых клеток. Заметим, что все согнутые полоски будут одноцветными, а все остальные нет. Так как количество черных клеток на 6 больше, чем количество белых, то число черных согнутых полосок на 3 больше, чем число белых. Следовательно, эти числа разной четности и их сумма нечетна. 10.2. Поскольку а0 + аг + ... + ak> m, то все суммы вида * —т + а0 + аг + ... + ak неотрицательны. Поэтому при х ^ 1 имеем цепочку неравенств Р (х) - тхп = (-т + а0) (хп - хп~1) + + (-т + ао + аг) (хп~1 - хп~2) + ... + + (-т + а0 + ... + ап_1) (х - 1) + (-т + а0 + ... + ап) > 0, так как каждое слагаемое неотрицательно. 10.3. См. решение задачи 9.4. 10.4. Ответ. При N = 101. Предположим, что при каком-то значении N фокус удастся. Тогда по каждому варианту последовательности с двумя закрытыми цифрами (пусть их количество равно kx) фокусник может восстановить исходную; значит, каждой последовательности с двумя закрытыми цифрами фокусник однозначно может поставить в соответствие восстановленную последовательность из N цифр (пусть их количество равно k2). Следовательно, kx ^ fe2. Отметим, что kx = (N — 1) ■ 10^~2 (есть N - 1 вариант вычеркнуть две цифры, а на остальные N - 2 позиции есть по 10 вариантов на каждую). Нетрудно видеть, что k2 = 10N. Тогда из kx ^ k2 следует, что N - 1 > 100, T.e.N> 101. Покажем, как выполнить фокус при N = 101. Пусть сумма всех цифр на нечетных позициях имеет остаток s от деления на 10, а сумма всех цифр на четных позициях имеет остаток t от деления на 10 (позиции нумеруются слева направо числами от 0 до 100). Положим р = 10s + t. Пусть помощник закроет цифры, стоящие на позициях рир + 1. Увидев, какие цифры закрыты, фокусник определит р, а следовательно, определит s и t. Отметим, что одна закрытая цифра 180 XXXIII Всероссийская математическая олимпиада школьников
стоит на нечетной позиции, а другая — на четной. Таким образом, вычислив сумму открытых цифр на нечетных позициях и зная s, фокусник определит закрытую цифру, стоящую на нечетной позиции. Аналогично определяется закрытая цифра, стоящая на четной позиции. Замечание. Применяя теорему Холла, можно доказать существование алгоритма действий фокусника и его помощника при N = 101, не приводя явно самого алгоритма. 10.5, Первое решение. Пусть сумма с всех векторов отлична от нуля (| а | = s > 0). Введем прямоугольную систему координат Оху, в которой ось Ох сонаправлена с а. Пусть а — длинный вектор набора, т. е. он не коро- че, чем Ъ = с — а. Поскольку г/-координаты векторов а и о равны по модулю, то дг-координата ах вектора а по модулю не меньше, чем дг-координата bx = s — ах вектора Ь. Отсюда получаем, что ах ^ ^. Теперь, если все векторы набора длинные, то сумма их дг-координат не меньше ^ > s, но эта сумма равна s. Противоречие. Второе решение. Обозначим данные векторы через ак -*■ (k = 1, ..., п)у а их сумму через а. По условию I ak I ^ IG — ak I- Возведем это неравенство в квадрат: а\ ^ с2 - 2с - ак + а\ . Просуммировав такие неравенства по всем k от 1 до пу получаем 0> пс2-2с • (ах + а2 + ... + ал), т. е. 0 > (п - 2) а2. Значит, с = 0. 10.6. Ответ. 1:3. Пусть X — точка пересечения прямых АВ и PQ (рис. 77 на с. 182). Тогда ХР2 = ХА - ХВ = = XQ2> т. е. X — середина PQ. Прямые АВ и PR параллельны, так как обе эти прямые перпендикулярны линии центров окружностей cOj и со2- Из условия теперь получаем, что четырехугольник PXBR — параллелограмм, откуда BR = XP=- PQ-- RS (послед- нее — из симметрии PQ и RS). Далее, так как RS — отрезок касательной к со2> то RB • RW= RS2 = (2RB)2> откуда RW= 4RB. Значит, RB : BW = 1 : 3. V ЭТАП. 2006/2007 учебный год 181
Р ЛI Q R Рис. 77 10.7, Рассмотрим произвольную хорошую раскраску. Заметим, что общее число концов ребер каждого цвета четно. При этом в каждой вершине степени 3 количество концов каждого цвета имеет одинаковую четность (их там по одному). Поэтому и в вершине А четность их количеств также одинакова. Тогда все они нечетны, и в ней сходятся три ребра одного цвета и по одному ребру остальных. Предположим, что утверждение задачи не выполнено, т. е. нет хорошей раскраски с тремя последовательными ребрами одного цвета, выходящими из одной вершины. Докажем, что количество хороших раскрасок, в которых из А выходит три синих ребра, делится на 5. Тогда, очевидно, и общее количество раскрасок будет делиться на 5, что противоречит условию. Пусть из вершины А последовательно выходят ребра АВ19 АВ2у АВ3у АВ4 и АВъ (далее по циклу опять идет ребро АВг). В любой раскраске красное и лиловое ребра из вершины А идут не подряд (иначе и три синих ребра идут подряд). Следовательно, для концов красного и лилового ребер есть 5 вариантов: (В19 Б4), (Б2, Б5), (Б3, Вх), (Б4, Б2), (Б5, Б3). Обозначим соответствующие количества раскрасок через й14, й25, ksi, k42J k5B. Мы докажем, что k14 ^ k42 ^ k25 < k53 ^ k31 ^ ft14, откуда будет следовать, что все 5 чисел равны, а общее количество раскрасок делится на 5. Покажем, что k25 ^ й53 (остальные неравенства аналогичны). Пусть в некоторой раскраске ребра АБ2 и АВ5 не синие (пусть для определенности АБ2 красное). Рассмотрим граф, вершинами которого являются вершины многогранника, а ребрами — синие и красные ребра. Тогда степень вершины А равна 4, а степени остальных вершин — 2. Отсюда сразу сле- 182 XXXIII Всероссийская математическая олимпиада школьников
дует, что граф распался на несколько циклов, причем два из них пересекаются только по вершине А, а остальные не пересекаются вовсе. Рассмотрим цикл, проходящий через А и содержащий АВ2. Тогда он содержит еще и синее ребро, выходящее из А. Перекрасим синие ребра этого цикла в красные и наоборот. Тогда мы получили другую хорошую раскраску. в Bl в вь Рис. 78 При этом возможны три случая (рис. 78). Если цикл содержит синее ребро АВг или АВ4, то после перекраски три последовательных ребра (АВ2, АВ3, АВ4 или АВ19 АВ2у АВ3) окрашены в синий цвет; это невозможно по предположению. Значит, в цикле есть ребро АВЗУ и после перекрашивания получилась раскраска, в которой красное и лиловое ребра — АВ3 и АВ5. При этом из разных раскрасок после перекрашивания получались разные, так как исходная раскраска восстанавливается по новой аналогичной процедурой. Поэтому k25 ^ й53, что и требовалось. Замечание 1. Легко видеть, что мы нигде не пользовались специфическими особенностями многогранника. Зафиксировав некоторую (вообще говоря, произвольную) циклическую нумерацию В19 В2У В3, В4, В5 соседей вершины А у мы доказали существование хорошей раскраски, в которой в один цвет покрашены три ребра, идущих из А к последовательным в этой нумерации вершинам. Замечание 2. Вернемся к задаче с многогранником. Предположим, что в нашем многограннике есть хорошие раскраски, но откажемся от условия, что их количество не кратно 5. Сделаем еще более смелое предположение, что нам в таких условиях удалось доказать наличие хорошей раскраски, в которой три последовательных ребра, выходящие из А, покрашены в один и тот же цвет. Отсюда без труда выводится самое из- V ЭТАП. 2006/2007 учебный год 183
вестное (без преувеличения!) утверждение в теории графов — гипотеза четырех красок. 10,8, Ответ. При N = 155. См. решение задачи 9.8. ф 11 класс 11.1. Заметим, что | sin Зх | = | 3 sin х - 4 sin3 x \ = = | 3 - 4 sin2 х | | sin х | ^ 3 | sin x |. Поэтому для функции fl9 полученной из / заменой cos Зх на sin 3xy выполняется неравенство | fx (х) | ^ 3 | sin х | | cos х | | cos 2x | x x |cos 4x| |cos 8л;| • ... • |cos 2kx\. (Мы опустили все множители |cos nx\9 в которых п > 3 и не является степенью двойки; каждый из этих множителей не превосходит 1.) Утверждение задачи теперь следует из тождества sin х - cos x - cos 2x • cos 4x • cos 8x • ... • cos 2kx = = 2-k~1 sin 2k + 1x. 11.2. Так как ВС — касательная к вписанной окружности (рис. 79), то ZBArQ = \ A&Q = £AXBXQ\ с другой стороны, ZBAXA = ZAXAR как внутренние накрест лежащие. Поэтому Z.QAR = /.АХВ^У и четырехугольник ARBXQ — вписанный. Аналогично, вписанным является и четырехугольник PAQCU поэтому ZPQR = = ZPQA + ZRQA = ZPCXA + ZRBXA = ZAXCXB + ZAXBXC. Поскольку оба слагаемых — углы между касательными и хордами, имеем ZPQR = - (А1С1 + АХВ^) = = - В1А1С1 = ZBXQCX9 что и требовалось доказать. Рис. 79 184 XXXIII Всероссийская математическая олимпиада школьников
11.3. Ответ. При N = 101. См. решение задачи 10.4. 11.4. Назовем особым членом последовательности такой член хпУ для которого [хп] > [хп_1]. Очевидно, особых членов бесконечно много (так как если [хп] = й, то [^п + л] > [хп\)- Для каждого особого члена представим его дробную часть в виде несократимой дроби (если особый член — целое число, будем считать числитель его дробной части равным 0). Мы докажем, что числитель этой дроби у каждого следующего особого члена не больше, чем у предыдущего; кроме того, если этот числитель больше 0, то найдется особый член, у которого соответствующий числитель меньше. Пусть хк — особый член. Обозначим его целую часть через т (очевидно, т ^ 2), а дробную — через г. Поскольку [лг/г-i] = т - 1, то имеет место неравенство 12 1 г < ^ —. Если г < —, то следующие т членов по- следовательности будут равны xk + 1 = xk + —, xk + 2 = т 2 т — 1 т — 1 = xk + —, ..., хк + т_1 = хк + —— = m + r + —^- <m+l, xk + m = xk + 1- Как видно, xk + m — очередной особый член, по сравнению с предыдущим его дробная часть не изменилась, а целая часть увеличилась на 1. Продолжая так же, мы придем, наконец, к особому члену xtJ целая часть которого р = [хД и дробная часть s = ixi) удовлетворяют условию — ^ s < -. Тогда Р Р + 1 следующие члены последовательности будут равны _ 1 _ 2 _^~2_ Х1+1 — Х1 + ~> Х1 + 2 — Х1 + ~у ---у Х1+р-2 — Х1 ^ ~ — р-2 2 Р-2 <Р + ~ +-у Р=Р+ 1, xi + P-i = Новый особый член xl + D_1 имеет дробную часть s . Р Докажем, что числитель у нее меньше, чем у s. Дей- а 1 ар -Ъ ствительно, если s = -, то s = —, и, поскольку о р ро а 1 * - < -, выполняется неравенство ар - а < Ь <=> о р — 1 <=> ар - Ь < а. V ЭТАП. 2006/2007 учебный год 185
Итак, для каждого особого члена последовательности, дробная часть которого имеет ненулевой числитель, мы нашли особый член с меньшим числителем дробной части. Так как числитель — целое неотрицательное число, которое не может уменьшаться бесконечно много раз, в последовательности встретится член, дробная часть которого равна 0, что и требовалось доказать. 11.5. См. решение задачи 9.5. 11.6. Ответ. Не существуют. Первое решение. Пусть такие числа а, 6, с нашлись. Тогда они целые в силу теоремы Виета; кроме того, при каждом п > 3 одно из чисел с или —с является произведением всех корней многочлена Рп (х), т. е. произведением п целых чисел. Для каждого построенного многочлена Рп (х) рассмотрим все его корни, отличные от ±1; их произведение равно ±с. У чисел с и -с конечное число разложений на множители, отличные по модулю от единицы; значит, какое-то из таких разложений встретится бесконечное число раз. Рассмотрим последовательность многочленов с таким разложением. Они различаются между собой только дополнительными корнями, равными +1 и -1. Рассмотрим два таких многочлена Рт (х) и Pk (x) степеней т > k. Из теоремы Виета получаем, что сумма чисел, обратных к корням каждого из этих многочленов, равна —. Эти суммы различаются лишь наличием нескольких слагаемых вида 1 и —1; ясно, что у многочлена Рт по сравнению с многочленом Рк добавились равные количества таких корней, т. е. Рт (х) = Pk (х) (х - l)d (х + \у = Pk (х) (х2 - l)d. (*) Заметим, что (х2 - l)d = x2d - ... + (-l)d- xdx2 + (-l)d. Тогда, сравнив свободные члены и коэффициенты при х2 в левой и правой частях равенства (*), получаем с = с • (-l)d (откуда d — четно) и а = а - (-l)d + + с • (-l)d~ * d = a - cd; последнее невозможно, так как cd Ф О. Противоречие. Второе решение. Опять же предположим, что такие а, Ь, с нашлись. Пусть k — максимальное число сомножителей (больших 1 по модулю), на которые раскладывается число с. Тогда у каждого многочлена Рп (х) не больше k корней, отличных от ±1. 186 XXXIII Всероссийская математическая олимпиада школьников
Пусть xl9 ..., хп — корни этого многочлена. Рассмотрим сумму S = —г- + ... н——. С одной стороны, в эту х\ х2п сумму входит хотя бы п - k единиц, поэтому S ^ п — k. С другой стороны, X±XZ x\xs хп - \Xn ( \2 ( > что по теореме Виета равно S = - -2-1. Это не- возможно, если п — k>\-\ — 2 — . 11.7. Пусть дана пирамида ABCD. Выберем пару ее скрещивающихся ребер с наибольшей суммой квадратов — пусть это АВ и CD. Покажем, что шары с диаметрами АВ и CD покрывают каждое ребро пирамиды. Ясно, что достаточно доказать это для ребра ВС. Рассмотрим основания Ах и Dx перпендикуляров, опущенных соответственно из А и В на ВС (рис. 80). Тогда АВ2 + CD2 = АА\ + ВА\ + DD\ + CD2 > > AC2 + BD2 = AA\ + С A2 + DD\ + BD2, откуда BA\ + CD\ > CA\ + BD\. Это означает, что отрезки ВАг и CDr перекрываются, а значит, они покрывают весь отрезок ВС. Но наши шары как раз покрывают оба этих отрезка. Поскольку шары покрывают все ребра, то они покрывают и все грани. Пусть теперь какая- то точка X тетраэдра не покрыта шарами. Тогда из нее можно выпустить луч, не имеющий общих точек с шарами. Однако он пересечет поверхность в точке, принадлежащей одному из Рис. 80 шаров. Противоречие. 11.8. Первое решение. Рассмотрим граф, вершины которого — города, а ребра — авиалинии. Обобщим задачу — разрешим графу иметь кратные ребра. Тогда для любого набора, скажем, из k вершин, количество ребер между ними не превосходит 2й - 2. Требуется дока- V ЭТАП. 2006/2007 учебный год 187
зать, что можно покрасить ребра графа в два цвета так, чтобы не было одноцветных циклов. Назовем непустое подмножество А вершин графа критическим, если количество ребер графа между вершинами множества А ровно 2 |А\ — 2. Лемма. Если А и В — критические подмножества, причем А П В Ф0У то A U В тоже критическое. Доказательство. Пусть С = А Г) В, D = A U В и D не критическое. Пусть | А | = a, | В | = Ь, | С | = с, \D\ = d = = а + b — с. Так как количество ребер в А равно 2а - 2, а количество ребер в D меньше, чем 2d - 2, то число ребер, соединяющих вершины из D, у которых не оба конца лежат в А, меньше (2d — 2) — - (2а - 2) = 2 (d - а) = 2 (Ъ - с). В частности, в число этих ребер входят все ребра, соединяющие вершины В, не обе из которых лежат в С. Поэтому их число также меньше 2 (Ь — с), а число остальных ребер среди вершин В больше (2Ь — 2) — 2 (Ь — с) = 2с — 2. Но это в точности ребра, соединяющие вершины множества С; значит, С не удовлетворяет условию задачи. Противоречие. Лемма доказана. Замечание. Заметим, что в условиях леммы множество С также будет критическим. Перейдем к решению задачи. Предположим противное. Рассмотрим граф с минимальным числом вершин п, для которого утверждение задачи не выполняется. Рассмотрим все его вершины. Число ребер между ними не больше 2п — 2. Если степень каждой вершины не меньше 4, то общее количество ребер не меньше 4-^ = 2п>2п — 2, что невозможно. Значит, найдется вершина а степени не больше 3. Если ее степень меньше 3, то выкинем ее; ребра оставшегося графа можно покрасить требуемым образом, так как он, очевидно, удовлетворяет условию. Покрасив после этого ребра из вершины а в разные цвета, мы, очевидно, не образуем одноцветных циклов, и требуемая раскраска получена. Итак, степень а равна 3, и она соединена с вершинами by с, d. Все три вершины Ь, с, d не могут совпадать, так как иначе между двумя вершинами а и Ь было бы больше (2-2-2) ребер, что невозможно. Тогда среди Ь, с и d есть вершина, отличная от обеих остальных — пусть это вершина с. 188 XXXIII Всероссийская математическая олимпиада школьников
Рис.81 IXV Выбросим из графа вершину а. Если в оставшемся графе пара вершин Ь и с не принадлежит одновременно никакому критическому подмножеству, то после добавления «фиктивного» ребра (Ь, с) мы получим граф, удовлетворяющий условию задачи, число вершин в котором меньше, чем в нашем (рис. 81). Покрасим его ребра требуемым образом, потом удалим добавленное ребро, вернем вершину а и покрасим ребра (а, Ь) и (а, с) в цвет «фиктивного» ребра, а ребро (a, d) в другой цвет. Очевидно, одноцветных циклов не появится. Аналогично можем поступить, если end одновременно не принадлежат критическому множеству. Если же вершины бис принадлежат критическому множеству А19 а вершины cud — критическому множеству А2, то Ах U А2 тоже критическое (ибо с е Аг П А2). Но тогда, добавив к этому множеству вершину а, мы добавим к его внутренним ребрам три ребра; следовательно, полученное множество противоречит условию задачи. Второе решение. Покажем, как можно по-другому (без использования леммы) завершить решение задачи. Так же как и в первом решении, рассмотрим вершину а степени 3 и критическое подмножество V9 содержащее двух ее соседей, но не ее саму. Выкинем из нашего графа G все вершины множества V и добавим новую вершину v9 причем для каждого ребра, соединяющего вершину V с вершиной не из V, соединим i; с этой внешней вершиной (рис. 82). (Такая операция называется стягиванием подграфа V.) Покажем, что новый граф G' также удовлетворяет условию. Рассмотрим произвольное множество D Рис. 82 V ЭТАП. 2006/2007 учебный год 189
из d его вершин. Если v £ Z>, то число ребер между ними такое же, как было в графе G, т. е. не болыпе 2d - 2. Если же v e D> то рассмотрим множество D U F\{i>} из (d + k — 1) вершин графа G; среди них не больше (2d + 2k - 4) ребер, из них ровно (2k - 2) ребер между вершинами V; значит, остальных ребер не больше (2d - 2), что и требовалось. Ребра графа на вершинах множества V можно покрасить требуемым образом. Покрасим также таким образом ребра G' (в обоих полученных графах меньше, чем по п вершин!). В графе G каждое ребро соответствует ребру в одном из двух графов — G' или V. Покрасим это ребро так же, как соответствующее ему ребро в этих графах. Покажем, что не появилось одноцветных циклов. Пусть это не так. Ясно, что каждый цикл проходит как по вершинам множества F, так и по другим вершинам. Поэтому можно выйти по ребру из вершины множества F, пройти по нескольким (болыпе одного!) одноцветным ребрам и впервые прийти снова в вершину множества V. Это снова означает наличие цикла в новом графе. Противоречие. Замечание. Заметим, что если между какими-то k вершинами число ребер больше (2k - 2), то требуемая покраска невозможна. Таким образом, верно и утверждение, обратное утверждению задачи.
ПИСОК ЛИТЕРАТУРЫ Агаханов Н. X., Купцов Л. П., Нестеренко Ю. В., Резничен- коС.В., СлинъкоA.M. Математические олимпиады. 9 класс. — М.: Просвещение: Учеб. лит., 1997. — 208 с. Агаханов Н.Х., ПодлипскийО.К. Математические олимпиады Московской области. — М.: Физматкнига, 2006. — 320 с. Агаханов Н. X., Подлипский О. К. Всероссийская олимпиада школьников по математике: Методическое пособие / Науч. ред. Э. М. Никитин. — М.: АПК и ППРО, 2005. — 140 с. АгахановН.Х., ПодлипскийО.К. Всероссийская олимпиада школьников по математике в 2006 году / Науч. ред. Э. М. Никитин. — М.: АПК и ППРО, 2006. — 161 с. Агаханов Н. X., Терешин Д. А., Кузнецова Г. М. Школьные математические олимпиады. — М.: Дрофа, 1999. — 128 с. Берлов С. Л., Иванов С. В., Кохасъ К. П. Петербургские математические олимпиады. — Санкт-Петербург, Москва, Краснодар: Лань, 2005. — 606 с. Васильев Н. Б., Егоров А. А. Задачи Всесоюзных математических олимпиад. — М.: Наука, 1988. — 288 с. Виленкин Н. Я., Виленкин А. Н., Виленкин П. А. Комбинаторика. — М.: ФИМА, МЦНМО, 2006. — 400 с. Всероссийские олимпиады школьников по математике 1993— 2006 / под ред. Н. X. Агаханова. — М.: МЦНМО, 2007. — 468 с. Гальперин Г. А., Толпыго А. К. Московские математические олимпиады. — М.: Просвещение, 1986. — 303 с. ГенкинС.А., Итенберг И.В., Фомин Д. В. Ленинградские математические кружки. — Киров: Аса, 1994. — 272 с. Горбачев Н. В. Сборник олимпиадных задач по математике. — М.: МЦНМО, 2005. — 560 с. Зарубежные математические олимпиады / под ред. И. Н. Сергеева. — М.: Наука, 1987. — 416 с. Канель-Белов А. Я., Ковалъджи А. К. Как решают нестандартные задачи / под ред. В. О. Бугаенко. — М.: МЦНМО, 2004. — 96 с. Купцов Л. П., Нестеренко Ю. В., РезниченкоС.В., Слинько А.М. Математические олимпиады. 10 класс. — М.: Просвещение: Учеб. лит., 1998. — 256 с. Купцов Л. П., Нестеренко Ю. В., Резниченко С. В., Слинько A.M. Математические олимпиады. 11 класс. — М.: Просвещение: Учеб. лит., 1999. — 254 с. Список литературы 191
Купцов Л. П., РезниченкоС.В., ТерешинД.А. Российские математические олимпиады школьников. Книга для учащихся. — Ростов-на-Дону: Феникс, 1996. — 640 с. КюршакЙ., НейкоммД., ХайошД., ШураниЯ. Венгерские математические олимпиады. — М.: Мир, 1976. — 543 с. Леман А. А. Сборник задач Московских математических олимпиад. — М.: Просвещение, 1965. — 384 с. МуштариД.Х. Подготовка к математическим олимпиадам. — Казань: Изд-во Казан, матем. об-ва, 2000. — 239 с. ОреО. Графы и их применение. —М.: КомКнига, 2006. — 168 с. Прасолов В. В. Задачи по планиметрии. —М.: МЦНМО, 2006. — 640 с. Прасолов В. В., ШарыгинИ.Ф. Задачи по стереометрии. — М.: Наука, 1989. — 288 с. Савин А. П. и др. Физико-математические олимпиады. Сборник. — М.: Знание, 1977. — 160 с. СедракянН.М., Авоян A.M. Неравенства. Методы доказательства. — М.: Физматлит, 2002. — 256 с. Серпинский В. 250 задач по теории чисел. — М.: НИЦ РХД, 2004. — 160 с. Фомин Д. В. Санкт-Петербургские математические олимпиады. — СПб.: Политехника, 1994. — 309 с. Федоров Р. М., Канелъ-Белов А. Я., Ковалъджи А. К., Ящен- коИ.В. Московские математические олимпиады 1993—2005 г. / под ред. В. М. Тихомирова. — М.: МЦНМО, 2006. — 456 с. ШарыгинИ.Ф. Геометрия. Планиметрия. 9—11 кл. — М.: Дрофа, 2001. — 398 с. ЭвнинА.Ю. Элементарная теория чисел. Сборник олимпиад- ных задач. — Челябинск: Изд-во ЧГТУ, 1996. — 76 с. Яковлев Т.Н.у Купцов Л. П., РезниченкоС.В., Гусятников П. Б. Всероссийские математические олимпиады школьников. — М.: Просвещение, 1992. — 383 с.